You are on page 1of 358

INTRODUCTION

Electrical Occupation

 Organization of the Industry

The electrical industry is one of the largest in the United States and Canada. In 2008,
electricians held about 692,000 jobs. Electrical contracting firms employed about
65% of the wage and salaried workers. The remainder worked as electricians in other
related industries. About 9% of the electricians were self-employed. The
opportunity for employment and advancement as an electrician is one of the highest
of any industry. Basically, the entire country runs on electricity. Industry,
commercial locations, and homes all employ electricity as the main source of power.
It has been estimated that between 2008 and 2018 the need for qualified electricians
will increase of over 8000 electricians over the next 10 years.ertyuio90p-The lay-off
rate of electricians is one the l `5432Wowest of any occupation. If the industry
operates, it will require electricians to keep it running.

 Electrical Personnel

Electricians can generally be divided into several categories, depending on their


specific area of employment. Each of these categories may require special skills.

 Construction

 Industrial Electricians

 Instrumentation Technicians

 Related Industries

The fields related to the electrical industry are too numerous to mention
but include air conditioning and refrigeration, aircraft electronics,
automotive, cable TV, broadcast media, energy and utilities, and home
appliance and repair, as well as many, many others. The opportunity for
employment in the electrical field is almost unlimited.

 Ethics

 Appearance

 Communication
2

 Speaking
 Listening
 Writing

 Working on a Team

 Building Code

The 15 Most Dangerous Jobs In America

1: Fishers and related fishing workers


2: Logging workers
3: Aircraft pilots and flight engineers
4: Farmers and ranchers
5: Coal mining
6: Roofers
7: Refuse and recyclable material collectors
8: Driver/sales workers and truck drivers
9: Police and sheriff's patrol officers
10: Electrical power-line installers and repairers
11: Construction laborers
12: Taxi drivers and chauffeurs
13: Grounds maintenance workers
14: Athletes, coaches, umpires, and related workers
15: Operating engineers and construction
equipment operators
3

SAFETY OVERVIEW

Why You Need to Know

Safety is the job of each individual. You should be concerned not only with your own
safety but also with the safety of others around you. This is especially true for persons
employed in the electrical field. Some general rules should be followed when working
with electrical equipment or circuits.

General Safety Rules

 Never Work on an Energized Circuit If the Power Can Be Disconnected

When possible, use the following three-step check to make certain that power is
turned off:

1. Test the meter on a known live circuit to make sure the meter is operating.
2. Test the circuit that is to become the de-energized circuit with the meter.
3. Test the meter on the known live circuit again to make certain the meter is
still operating.

Install a warning tag at the point of disconnection so people will not restore
power to the circuit. If possible, use a lock to prevent anyone from turning
the power back on.

 Think

Of all the rules concerning safety, this one is probably the most important. No
amount of safeguarding or idiot proofing a piece of equipment can protect a
person as well as taking time to think before acting. Many technicians have
been killed by supposedly “dead circuit”. Do not depend on circuit breakers,
fuses, or someone else to open a circuit. Test it yourself before you touch it. If
you are working on high-voltage equipment, use insulated gloves and meter
probes to measure the voltage being tested. Think before you touch something
the could you your life.

 Avoid Horseplay

Jokes and horseplay have a time and place, but not when someone is working
on an electric circuit or a piece of moving machinery. Do not be the cause of
4

someone’s being injured or killed, and do not let someone else be the cause of
your being injured or killed.

 Do Not Work Alone

This is especially true when working in a hazardous location or on a live


circuit. Have someone with you can turn off the power or give artificial
respiration and/or cardiopulmonary resuscitation (CPR). Several electrical
shocks can cause breathing difficulties and can cause heart to go into
fibrillation.

 Work with One Hand When Possible

The worst kind of electric circuit occurs when the current path is from one
hand to the other, which permits the current to pass directly through the heart.
A person can survive a severe shock between the hand and foot that would
cause death if the current path from one hand to the other.

 Learn First Aid

Anyone working on electric equipment, especially those working with


voltages greater than 50 volts, should make an effort to learn first aid. A
knowledge of first aid, especially CPR, ay save your own or someone else’s life.

 Avoid Alcohol and Drugs

The use of alcohol and drugs has no place on a work site. Alcohol and drugs
are not only dangerous to users and those who work around them; they also
cost industry millions of dollars a year. Alcohol and drug abusers kill
thousands of people on the highways each year and are just as dangerous on a
work site as they left behind the wheel of a vehicle. Many industries have
institutes testing policies to screen for alcohol and drugs. A person who tests
positive generally receives a warning the first time and is fired the second
time.

Effects of Electric Current on the Body


5

0.100-0.200 (DEATH) THIS RANGE


AMPERES GENERALLY CAUSES
FIBRILLATION OF THE HEART.
WHEN THE HEART IS IN THIS
CONDITION, IT VIBRATES A
FAST RATE LIKE A “QUIVER”
AND CEASES TO PUMP BLOOD
TO THE REST OF THE BODY.

0.060-0.100 (EXTREME DIIFICULT IN


AMPERES BREATHING)

0.040-0.060 (DIFFICULTY IN BREATHING)


AMPERES

0.030-0.040 (MUSCULAR PARALYSIS)


AMPERES

0.020-0.030 (UNABLE TO LET GO OF THE


AMPERES CIRCUIT)

0.010-0.020 (VERY PAINFUL)


AMPERES

0.009-0.010 (MODERATE SENSATION)


AMPERES

0.002-0.003 (SLIGHT TINGLING SENSATION)


AMPERES

.
6

Unit 1

INTRODUCTION TO ELECTRICITY

LEARNING OUTCOMES

After completing this


unit, A.
you are expected to:
B.

1. define electricity.
2. name some scientists who contributed to the
development of electricity and electronics.
3. discuss the scientist’s contributions to electricity and
electronics.
4. quote some applications of electricity and electronics.
5. identify various electrical components.
6. use metric prefixes in simplifying large and small
numbers.
7. perform mathematical operations involving powers of
ten and metric prefixes.
8. identify the SI units of mass, force, weight, work, power
and energy.
9. discuss the difference between direct current and
alternating current.
7

Important Terms

electricity semiconductor
static electricity active element
dynamic electricity passive element
resistor electrical quantities
resistance metric prefixes
inductor direct current
inductance alternating current
transformer

1.1 Definition of Electricity

Electricity is a physical phenomenon arising from the


existence and interaction of electric charge. It is a form of
energy generated by friction, heat, light, magnetism, chemical
reaction, etc.

1.1.1 Two Types of Electricity:

a. Static electricity – electricity at rest. It cannot flow from one place to


another.
b. Dynamic electricity – also known as current electricity. Electricity in
motion. It can be transmitted from one place to the other.

1.1 .2 Methods of Producing Electricity

There are six methods for producing electricity:


1. Magnetism
2. Chemical reaction
3. Pressure
4. Heat
5. Friction
6. Light
8

1.1.3 Electrical Effects

With the exception of friction, electricity can be used to cause the same effects
that cause it.
1. Magnetism
2. Chemical action
3. Pressure
4. Heat
5. Light

1.2 History of Electricity and Electronics

Prehistoric people experienced the properties of magnetite – permanently


magnetized pieces of ore, often called lodestones. These magnetic stones were strong
enough to lift pieces of iron.
The philosopher Thales of Miletus (640 – 546 B.C.) is thought to have been the first
person who observed the electrical properties of amber. He noted that when amber
was rubbed, it acquired the ability to pick up light objects such as straw and dry grass.
He also experimented with the lodestones and knew of its power to attract iron. By
the thirteenth century, floating magnets were used for compasses.
One of the first important discoveries about static electricity is attributed to
William Gilbert (1540-1603). Gilbert was an English Physician who, in a book
published in 1600, described how amber differs from magnetic loadstones in its
attraction of certain materials. He found that when amber was rubbed with a cloth,
it attracted only lightweight materials, whereas loadstones attracted only iron.
Gilbert also discovered that other substances, such as sulfur, glass, and resin, behave
as amber does. He used the Latin word elektron for amber and originated the word
electrica for the other substances that acted similar to amber. The word electricity
was used for the first time by Sir Thomas Browne (1605-82), an English physician.
Following Gilbert’s lead, Robert Boyle published his many experimental
results in 1675. Boyle was one of the early experiments with electricity in a vacuum.
Otto von Guiricke (1602 – 1686) built an electrical generator and reported it
in his Experimenta Nova of 1672. This device was a sulfur globe on a shaft that could
be turned on its bearing . When the shaft was turned with a dry hand held on the
surface, an electrical charge gathered on the globe’s surface. Guiricke also noted small
sparks when the globe was discharges. In his studies of attraction and gravitation,
Guiricke devised the first electrical generator. When a hand was held on a sulfur ball
revolving in its frame, the ball attracted paper, feathers, chaff, and other light objects.
Another Englishman, Stephen Gray (1696-1736), discovered that some
substances conduct electricity and some do not. Following Gray’s Lead, a Frenchman
named Charles du Fay experimented with the conduction of electricity. These
experiments led him, to believe that there were two kinds of electricity. He called one
type vitreous electricity and the other type resinous electricity. He found that objects
having vitreous electricity repelled each other and those having resinous electrify
9

attracted each other. It is known today that two types of electrical charge exist,
positive and negative. Negative charge results from an excess of electrons in a
material and positive charge results from a deficiency of electrons.
A major advance in electrical science was made in Leyden, Holland, in 1746,
when Peter van Musschenbroek introduces a jar that served as a storage apparatus
for electricity. The jar was coated inside and out with a tinfoil, and a metallic rod was
attached to the inner foil lining and passed through the lid. Leyden jar were gathered
in groups (called batteries) and arranged with multiple connections, thereby further
improving the discharge energy.
Benjamin Franklin (1706-90) conducted studies in electricity in the mid-
1700s. He theorized that electricity consisted of a single fluid, and he was the first to
use the terms positive and negative. In his famous kite experiment, Franklin showed
that lightning is electricity.
Charles Augustin de Coulomb (1736-1806), a French physicist, in 1785
proposed the laws that govern the attraction and repulsion between electrically
charged bodies. Today, the unit of electrical charge is called the coulomb.
Luigi Galvani (1737-98) experimented with current electricity in 1786.
Galvani was a professor of anatomy at the University of Bologna in Italy. Electrical
current was once known as galvanism in his honor.
In 1800, Alessandro Volta (1745-1827), an Italian professor of physics,
discovered that the chemical action between moisture and two different metals
produced electricity. Volta constructed the first battery, using copper and zinc plates
separated by paper that had been moistened with a salt solution. This battery, called
the voltaic pile, was the first source of steady electric current. Today, the unit of
electrical potential energy is called the volt in honor of Professor Volta.
A Danish scientist, Hans Christian Oersted (1777-1851), is credited with the
discovery of electromagnetism, in 1820. He found that electrical current flowing
through a wire caused the needle of a compass to move. This finding showed that a
magnetic field exists around a current-carrying conductors and that the field is
produced by the current.
The modern unit of electrical current is the ampere (also called amp) in honor
of the French physicist André Ampère (1775-1836). In 1820, Ampère measured the
magnetic effect of an electrical current. He found that two wires carrying current can
attract and repel each other, just as magnets can. By 1822, Ampère had developed
the fundamental laws that are basic to the study of electricity.
One of the most well known and widely used laws in electrical circuits today
is Ohm’s law. It was formulated by Georg Simon Ohm (1787-1854), a German teacher,
in 1826. Ohm’s law gives us the relationship among the three important electrical
quantities of resistance, voltage, and current.
Although it was Oersted who discovered electromagnetism, it was Michael
Faraday (1791-1867) who carried the study further. Faraday was an English
physicist who believed that electricity could produce magnetic effects, then
magnetism could produce electricity. In 1831 he found that a moving magnet cause
an electric current in a coil of wire placed within the field of the magnet. This effect,
known today as electromagnetic induction, is the basic principle of electric
generators ands transformers.
10

Joseph Henry (1797-1878), an American physicist, independently discovered


the same principle in 1831, and it is in his honor that the unit of inductance is called
the henry. The unit of capacitance, the farad, is named in honor of Michael Faraday.
A paper published by James Prescott Joule in 1841 claimed the discovery of the
relationship between a current and the heat or energy produced, which today we call
Joule’s law. The unit of energy is called the joule in his honor.
In the 1860s, James Clerk Maxwell (1831-79), a Scottish Physicist, produced a
set of mathematical equations that expressed the laws governing electricity and
magnetism. These formulas are known as Maxwell’s equations. Maxwell also
predicted that electromagnetic waves (radio waves) that travel at the speed of light
in space could be produced.
It was left to Heinrich Rudolph Hertz (1857-94), a German physicist, to actually
produce these waves that Maxwell predicted. Hertz performed this work in the late
1880s. Today, the unit of frequency is called the hertz.

The Beginning of Electronics

The early experiments in electronics involved electric currents in glass tubes. One of
the first to conduct such experiments was a German named Heinrich Geissler (1814-
79). Geissler found that when he removed most of the air from a glass tube, the tube
glowed when an electrical potential was placed across it.
Around 1878, Sir William Crookes (1832-1919), a British scientist,
experimented with tubes similar to those of Geissler. In his experiments, Crookes
found that the current in the tubes seemed to consist of particles.
Thomas Edison (1847 – 1931), experimenting with the carbon-filament light
bulb that he had invented, made another important finding. He inserted a small metal
plate in the bulb. When the plate was positively charged, a current flowed from the
filament to the plate. This device was the first thermionic diode. Edison patented it
but never used it.
The electron was discovered in the 1890s. The French physicist Jean Baptiste
Perrin (1870 – 1942) demonstrated that the current in a vacuum tube consists of
negatively charged particles. Some of the properties of these particles were
measured by Sir Joseph Thomson (1856 – 1940), a British physicist, in experiments
he performed between 1895 and 1897. These negatively charged particles later
became known as electrons. The charge on the electron was accurately measured by
an American physicist, Robert A. Millikan (1868 – 1953), in 1909. As a result of these
discoveries, electrons could be controlled, and the electronic age was ushered in.

Putting the Electron to Work

A vacuum tube that allowed electrical current in only one direction was constructed
in 1904 by John A. Fleming, a British scientist. The tube was used to detect
electromagnetic waves. Called the Fleming valve, it was the forerunner of the more
recent vacuum diode tubes.
Major progress in electronics, however, awaited the development of a device
that could boost, or amplify, a weak electromagnetic wave or radio signal. This device
11

was the audion, patented in 1907 by Lee de Forest, an American. It was a triode
vacuum tube capable of amplifying small electrical signals.
Two other Americans, Harold Arnold and Irving Langmuir, made great
improvements in the triode tube between 1912 and 1914. About the same time, de
Forest and Edwin Armstrong, an electrical engineer, used the triode tube in an
oscillator circuit. In 1914, the triode was incorporated in the telephone system and
made the transcontinental telephone network possible.
The tetrode tube was invented in 1916 by Walter Schottky, a German. The
tetrode, along with the pentode (invented in 1926 by Tellegen, a Dutch engineer),
provided great improvements over the triode. The first television picture tube, called
the kinescope, was developed in the 1920s by Vladimir Zworykin, an American
researcher.
During World War II, several types of microwave tubes were developed that
made possible modern microwave radar and other communications systems. In
1939, the magnetron was invented in Britain by Henry Boot and John Randall. In the
same year, the klystron microwave tube was developed by two Americans, Russell
Varian and his brother Sigurd Varian. The traveling-wave tube was invented in
1943 by Rudolf Komphner, an Austrian-American.
The Computer

The computer probably has had more impact on modern technology than any other
single type of electronic system. The first electronic digital computer was completed
in 1946 at the University of Pennsylvania. It was called the Electronic Numerical
Integrator and Computer (ENIAC). One of the most significant developments in
computers was the stored program concept, developed in the 1940s by John von
Neumann, an American mathematician.

Solid State Electronics

The crystal detectors used in the early radios were the forerunners of modern solid
state devices. However, the era of solid state electronics began with the invention of
the transistor in 1947 at Bell Labs. The inventors were Walter Brattain, John Bardeen,
and William Shockley.
In the early 1960s, the integrated circuit was developed. It incorporated many
transistors and other components on a single small chip of semiconductor material.
Integrated circuit technology continues to be developed and improved, allowing more
complex circuits to be built on smaller chips.
The introduction of the microprocessor in the early 1970s created another
electronics revolution: the entire processing portion of a computer placed on a single,
small, silicon chip. Continued development brought about complete computers on a
single chip by the late 1970s.

Major Events in Electrical Sciences and Engineering


12

1672 Ottoo von I published I Nova.


1675 Robert Boyle was published Production of Electricity.
1746 The Leyden jar was demonstrated in Holland.
1750 Benjamin Franklin invented the lightning conductor.
1767 Joseph Priestley published the Present State of Electricity.
1786 Luigi Galvani observed electrical convulsion in the legs of dead
frogs.
1800 Alessandro Volta announced the voltaic pile.
1801 Henry Moyes was the first to observe an electric arc between
carbon rods.
1820 Hans Oersted discovered the deflection of a magnetic needle by
current on a wire.
1821 Michael Faraday produced magnetic rotation of a conductor and
magnet- the first electric motor.
1825 Andre-Marie Ampere defined electrodynamics.
1828 Joseph Henry produced silk-covered wire and more powerful
electromagnets.
1831 Michael Faraday discovered electromagnetic induction and carried
out experiments with an iron ring and core. He also experimented
with a magnet and rotating disk.
1836 Samuel Morse devised a simple relay.
1836 Electric light from batteries was shown at Paris Opera.
1841 James Joule stated the relation between current and energy
produced.
1843 Morse transmitted telegraph signals from England to France.
1850 First channel telegraph signals from Baltimore to Washington, D.C.
1858 Atlantic telegraph cable was completed and the first message sent.
1861 Western Union established telegraph service from New York to San
Francisco.
1862 James Clerk Maxwell determined the ohm.
1873 Maxwell published Treatise on Electricity and Magnetism.
1874 Alexander Graham Bell invented the telephone.
1877 Thomas Edison invented the telephone.
1877 Edison Electric Light Company was formed.
1881 First hydropower station was brought into use at Niagara, New
York.
1881 Edison constructed the first electric power station at Pearl Street,
New York.
1883 Overhead trolley electric railways were started at Portrush and
Richmond, Virginia.
1884 Philadelphia electrical exhibition was held.
1885 The American Telephone and Telegraph Company was organized.
1886 H. Hallerith introduced his tabulating machine.
1897 J.J. Thomson discovered the electron.
1898 Guglielmo Marconi transmitted radio signals from South Foreland
to Wimereux, England.
13

1904 John Ambrose Fleming invented the thermionic diode.

1.3 Applications of Electricity and Electronics

Computers Communications

Medicine Automation

Consumer Products
14

1.4 Circuit Components and Measuring Instruments

Resistors

These can be the carbon-composition type or wound with special resistance wire.
Their function is to limit the amount of current or divide the voltage in a circuit.

Capacitors

A capacitor is constructed of two conductor plates separated by an insulator (called


a dielectric). Its basic function is to concentrate the electric field of voltage across the
dielectric. As a result, the capacitor can accumulate and store electric charge from the
voltage source. Furthermore, the dielectric can discharge the stored energy when the
charging source is replaced by a conducting path.

When ac voltage is applied, the capacitor charges and discharges as the voltage varies.
The practical application of this effect is the use of capacitors to pass an ac signal but
to block a steady dc voltage.

Capacitors Inductors

Inductors

An inductor is just a coil of wire. Its basic function is to concentrate the magnetic field
of electric current in the coil. Most important, an induced voltage is generated when
15

the current with its associated magnetic field changes in value or direction. Inductors
are often called chokes.

In the practical application of a choke, the inductor can pass a steady current better
than alternating current. The reason is that a steady current cannot produce induced
voltage. Note that the effect of a choke, passing a steady current, is the opposite of
that of a coupling capacitor, which blocks dc voltage.

Transformers

A transformer consists of two or more coil windings in the same magnetic field.
Induced voltage is produced when the current changing in any winding. The purpose
of a transformer is to increase or decrease the amount of ac voltage coupled between
the windings. However, the transformer operates only with alternating current.

Transformers Semiconductor Devices

Semiconductor Devices

These include diode rectifiers and transistor amplifiers, either as separate, discrete
components or as part of an IC chip. A diode has two electrodes; the transistor has
three. In addition, the silicon controlled rectifier (SCR) and triac are used for power-
control circuits.

Active and Passive Elements

Active elements - are capable of delivering power to some external device.


Examples: dependent and independent voltage and current sources
.
.
16

Passive elements – are capable of receiving power. They are able to store to store
finite amounts of energy and then return that energy later to various external devices.
Examples are resistors, inductors, and capacitors.

Electronic Instruments

Typical instruments include the power supply, for providing voltage and current; the
voltmeter, for measuring voltage; the ammeter, for measuring current; the ohmmeter,
for measuring resistance; the wattmeter, for measuring power; and the oscilloscope
for observing and measuring AC voltages.
17

1.5 Electrical Quantities and units with SI symbols.

Table 1.1
Electrical Quantities and Units with SI Symbols

Quantity Symbol Unit Symbol


Capacitance C farad F
Charge Q coulomb C
Conductance G siemen S
Current I ampere A
Energy W joule J
Frequency f hertz Hz
Impedance Z ohm 
Inductance L henry H
Power P watt W
Reactance X ohm 
Resistance R ohm 
Voltage V volt V

1.6 Metric Prefixes

Table 1.2
Commonly Used Metric Prefixes in Electricity

Power of Ten Value Metric Prefix Metric Symbol


109 one billion giga G
106 one million mega M
103 one thousand kilo k
10-3 one-thousandth milli m
10-6 one-millionth micro 
10-9 one-billionth nano n
10-12 one-trillionth pico p

1.7 SI Units
.
18

1. Mass. It is quantity of matter contained in a body.


Unit of mass is kilogram (kg). Other multiples commonly used are :
1 quintal = 100 kg, 1 tonne = 10 quintals = 1000 kg

2. Force. Unit of force is newton (N). Its definition may be obtained from Newton’s
Second Law of Motion i.e. F = ma.
If m = 1 kg ; a = 1m/s2, then F = 1 newton.
Hence, one newton is that force which can give an acceleration of 1 m/s2 to a
mass of 1 kg.
Gravitational unit of force is kilogram-weight (kg-wt). It may be defined as
follows :
It is the force which can impart an acceleration of 9.8 m/s2 to a mass of 1 kg.
or,
It is the force which can impart an acceleration of 1 m/s2 to a mass of 9.8 kg.
Obviously, 1 kg-wt. = 9.8 N

3. Weight. It is the force with which earth pulls a body downwards. Obviously, its
units are the same as for force.
(a) Unit of weight is newton (N)
(b) Gravitational unit of weight is kg-wt.*
Note. If a body has a mass of m kg, then its weight, W = mg newtons = 9.8
newtons.

4. Work. If a force F moves a body through a distance S in its direction of


application, then
Work done W = F × S
(a) Unit of work is joule (J).
If, in the above equation, F = 1 N : S = 1 m ; then work done = 1 Nm or joule.
Hence, one joule is the work done when a force of 1 N moves a body through
a distance of 1 m in the direction of its application.
(b) Gravitational unit of work is m-kg. wt or m-kg**.
-----------------------------------------------------------------
* Often it is referred to as a force of 1 kg, the word ‘wt’ being omitted. To
avoid confusion with mass of 1 kg, the force of 1 kg is written in engineering
literature as kgf instead of kg. wt.
** Generally the work ‘wt’ is omitted and the unit is simply written as m-kg.

If F = 1 kg-wt; S = 1 m; then W.D. = 1 m-kg. Wt = 1 m-kg.


Hence, one m-kg is the work done by a force of one kg-wt when applied over
a distance of one meter.
19

Obviously, 1 m-kg = 9.8 m-N or J.

5. Power. It is the rate of doing work. Its units is watt (W) which represents 1 joule
per second.
1 W = 1 J/s
If a force of F newton moves a body with a velocity of ν m./s then power = F
× ν watt
If the velocity ν is in km/s, then power = F × ν kilowatt

6. Kilowatt-hour (kWh) and kilocalorie (kcal)


1 kWh = 1000 × 1 J/s × 3600 s = 36 × 105 J
1 kcal = 4,186 J ∴ 1 kWh = 36 × 105/4, 186 = 860 kcal

7. Miscellaneous Units
(i) 1 watt hour (Wh) = 1 J/s × 3600 s = 3600 J
(ii) 1 horse power (metric) = 75 m-kg/s = 75 × 9.8 = 735.5 J/s or watt
(iii) 1 kilowatt (kW) = 1000 W and 1 megawatt (MW) = 106 W

1.8 Six Basic SI Units

Quantity Basic Unit Symbol


Length meter m
Mass kilogram kg
Time second s
Electric Current ampere A
Thermodynamic kelvin K
Luminous
temperatureIntensity candela cd
Charge coulomb C

1.9 Calculating Power


If torque and angular speed are known, the power may be calculated. The
relationship when using a coherent system of units (such as SI) is simply
P = τω
where P is power, τ is torque, and ω is angular speed. But when using other
units or if the speed is in revolutions per unit time rather than radians, a
conversion factor has to be included. When torque is in pound-
foot units, rotational speed f is in rpm and power is required in horsepower:

τ(ft − lbf) x f(rpm)


P= hp
5252
.
20

The constant 5252 is the rounded value of (33,000 ft·lbf/min)/(2π rad/rev).


When torque is in inch pounds:

τ(ft − lbf) x f(rpm)


P= hp
63,025

The constant 63,025 is the rounded value of (33,000 ft·lbf/min) ×


(12 in/ft)/(2π rad/rev).

Mechanical horsepower ≡ 33,000 ft-lbf/min


hp(I)
= 550 ft·lbf/s
≈ 17696 lbm·ft2/s3
= 745.69987158227022 W

Metric horsepower ≡ 75 kgf·m/s


hp(M) - also PS,
≡ 735.49875 W
''cv, hk, pk, ks or ch
Electrical horsepower
≡ 746 W
hp(E)
Boiler horsepower ≡ 33,475 BTU/h
hp(S) = 9,812.5 W

Hydraulic horsepower = flow rate (US gal/min) × pressure (psi) ×


7/12,000
or
= flow rate (US gal/min) × pressure (psi) /
1714
= 550 ft·lbf/s
= 745.69988145 W

1.10 Comparison of AC and DC


21

100
V
50
v
o 0
l 0 1 2 3 4 5 6 7
t -50
s
-100
. time (ms)

E
80 Vpk
E DC 60 Hz AC
80 V

Figure 1.1 Graphs of DC and AC Voltages

Direct Current

The DC electricity, flows in one direction. The flow is said to be from negative to
positive. The normal source of a DC electricity, is the dry cell or storage battery.

Alternating Current.

The AC electricity constantly reverses its direction of flow. It is generated by machine


called generator. This type of current is universally accepted because of its limited
number of applications with the following advantages.
1. It is easily produced.
2. It is cheaper to maintain.
3. It could be transformed into higher voltage.
4. It could be distributed to far distance with low voltage drop.
5. It is more efficient compared with the direct current.

Comparison of DC Voltage and AC Voltage

DC Voltage AC Voltage
Fixed Polarity Reverses polarity
Can be steady or vary in magnitude Varies between reversals in polarity
Steady value cannot be stepped up or Can be stepped up or down for
down by a transformer electric power distribution
Easier to measure Easier to amplify

Heating effect is the same in AC and DC


22

The War of Currents


Thomas Edison, a prolific inventor in the 19th and 20th centuries, holds the record
for the most U.S. patents by one person. When it came to technology, he was usually
a winner. But there was one important area where Edison lost - and lost badly. It
was called the 'War of Currents,' and it pitted Edison and his support for direct
current (DC) electricity against engineers like George Westinghouse and Nikola
Tesla, who supported alternating current (AC).

In the 1880s, incandescent lighting was the main goal, and DC was just as good as
AC. But a storm was brewing, and it centered on which type of power, AC or DC,
would be best for power generation, electric motors, and power transmission.
The war came to a head in 1893, when the contract to provide electricity to the
Chicago World's Fair was awarded to Westinghouse, whose proposal, using AC,
came in over 30% cheaper than Edison's. In that same year, the Niagara Falls Power
Company decided to go with AC power generation for the city of Buffalo and signed
with Westinghouse and Tesla as well. These two major victories were part of the
changes taking place rapidly in the 1890s that set our country on a path toward AC
power.
23

Assessment No. 1

Name: Last, First, MI Score: _________ Rating: ______

How Much Have You Learned?

Direction: Search for names of scientist who contributed to the development of


electricity, electrical components and instruments found in this puzzle. Encircle the
name or word vertically, horizontally, backward, upward or downward.

C A B A D E T S R E O N A I T S I R H C S N A H O Q W E
H R T T U C Y R Y Q O O N P H A S D F G H J U K H W L Z
A L E S S A N D R O V O L T A Z X C V B N M Q V M W E R
R L L Y V P M U I P A S D T L U I O P T P A Q S M D F I
L E U C G A I J O S E P H H E N R Y H J K Y K L E Z T N
E W O D S C C B D F G H J K S L R E S I S T O R T A R A
S X J Z R I H E Z X C V B N O I M Q C E D R T Y E U A V
D A T I T T A N O P T G I H F J E K J Z X C N V R B N L
U M T Q W O E J R Y M U B I M I O P A S D F G H J K S A
F K O Z X R L A C V H B N M I R O B E R T B O Y L E F G
A R C U M Q F M W R O Y U M L I P O T A S K A S D F O I
Y E S H V J A I K P N R E T E M T L O V K L Z A C V R G
B L E N W M R N Y O O Z J F T W A X Q E W E R T T Y M I
A C R A W B A F N U M I P O U M U T C D R E Z X C V E U
N S P Q W E D R R X I T K Y S E M I C O N D U C T O R L
W E S I R O A A M P S A S D F S T E P H E N G R A Y G H
O M E L Z X Y N C V G B W A T T M E T E R B N M Q W E R
R A M L U I O K E O R B N E H C S S U M N A V R E T E P
B J A P O G A L S D O F G H J K Q K L M N I B J C K X Z
S X J C S V B I B N E P M E N O F B C K F X P L M L R G
A C X V B Q N N B M G Q D W E R T Y H U I O P A S F E R
M Y A G S D F G H M J W I L L I A M G I L B E R T Y T Y
O P O O Z I U Y N G T R E W Q J A S D F G H H J K L E M
H D H G C H A R L E S A U G U S T I N D E C O U L O M B
T A S D F G H J K L A A W R I T Y U I O P P A S D F M G
R A N D R E A M P E R E H G H J Z D Y Q W C E D R T A E
I B K L C M N B V E K C I R I U G N O V O T T O B V C X
S A H E I N R I C H R U D O L P H H E R T Z A Z X C V B
24

QUESTIONS: (not part of Assessment No. 1)

1. What is electricity?
2. Discuss the probable changes in your lifestyle if the electric service in your
home were disconnected for one week.
3. Name some scientists who contributed to the development of electricity and
electronic and their contributions to the development of electricity.
4. Discuss the scientist’s contributions to electricity and electronics.
5. What are some applications of electricity and electronics?
6. Identify various electrical components.
7. What is force?
8. What is one newton?
9. What is work?
10. What is the difference between direct current and alternating current?
25

Problem Set No. 1

METRIC PREFIXES

I. Express each of the following as quantity having a metric prefix:

1) 31 x 10−3 A
A. 0.31 mA
B. 3.1 mA
C. 31 mA
D. 310 mA

2) 5.5 x 103 V
A. 5.5 kV
B. 55 kV
C. 550 kV
D. 5.5 MV

3) 200 x 10−12 F
A. 200 pF
B. 200 nF
C. 200 µF
D. 2000 pF

4) 0.000003 F
A. 3 µF
B. 30 µF
C. 3 nF
D. 30 nF

5) 3,300,000 
A. 3.3 kΩ
B. 33 kΩ
C. 3.3 MΩ
D. 33 MΩ

6) 350 x 10−9 A
A. 35 nF
B. 350 nF
C. 3.5 pF
26

D. 35 pF

II. Express each quantity as a power of ten:

7) 5 A
A. 5 x 10-3 A
B. 5 x 10-6 A
C. 5 x 10-9 A
D. 50 x 10-3 A

8) 43 mV
A. 43 x 10-3 V
B. 43 x 10-6 V
C. 43 x 10-9 V
D. 43 x 10-12 V

9) 275 k
A. 275 x 106 V
B. 275 x 103 V
C. 275 x 10-3 V
D. 275 x 10-6 V

10) 10 MW
A. 10 x 1012 W
B. 10 x 109 W
C. 10 x 106 W
D. 10 x 103 W

III. Add the following quantities:

11) 6 mA + 3 A = _________ mA
A. 6.03
B. 60.03
C. 6.003
D. 6.3

12) 550 mV + 3.2 V = _________ V


27

A. 375
B. 37.5
C. 3.75
D. 0.375

13) 12 k + 6800  = ________ k


A. 1880
B. 188
C. 18.8
D. 1.88

14) 15 MW + 7500 kW = __________ MW


A. 0.0225
B. 0.225
C. 2.25
D. 22.5
.
.
.
.
.
.
.
.
.
.
.
.
.
.
.
.
.
.
.
.
.
.
.
.
.
.
.
.
.

.
28

Objective Test No. 1

INTRODUCTION TO ELECTRICITY

1. Which of the following is not an electrical quantity?


A. time
B. power
C. current
D. voltage

2. The unit of current is


A. volt
B. watt
C. joule
D. ampere

3. The unit of voltage is


A. ohm
B. volt
C. watt

4. The unit of resistance is


A. ohm
B. hertz
C. henry
D. ampere

5. 15,000 W is the same as


A. 15 W
B. 15 mW
C. 15 kW
D. 15 MW

6. The quantity 4.7 x 103 is the same as


A. 0.0047
B. 470
C. 4700
D. 47,000

7. The quantity 56 x 10-3 is the same as


A. 0.056
B. 0.560
C. 560
29

D. 56,000

8. The number 3,300,000 can be expressed as


A. 3.3 x 10-6
B. 3.3 x 106
C. 3.3 x 109
D. 3.3 x 1012

9. Ten milliamperes can be expressed as


A. 10 A
B. 10 mA
C. 10 kA
D. 10 MA

10. Five thousand volts can be expressed as


A. 5 kV
B. 50 MV
C. 500 kV
D. 5000 kV

11. Twenty million ohms can be expressed as


A. 20 
B. 20 m
C. 20 M
D. 20 MW

12. Hertz is the unit of


A. time
B. power
C. frequency
D. inductance

13. One of the following parameters cannot be changed by a transformer? Which


one is it?
A. Power
B. Voltage
C. Current
D. impedance

14. The prefix pico means


A. 10-12 of a unit
B. 10-9 of a unit
30

Objective Test No. 2

INTRODUCTION TO ELECTRICITY

Note: Answers of some items may not be found in this text. Look for answers in
other references.

1. Discovered the difference between conductors and insulators in 1729.


A. Luigi Galvani
B. Stephen Gray
C. Alessandro Volta
D. Gottfried Wilhelm Leibniz

2. Discovered Galvanic action in 1780.


A. Luigi Galvani
B. Stephen Gray
C. Alessandro Volta
D. Gottfried Wilhelm Leibniz

3. Invented the electric dry cell in 1800.


A. Luigi Galvani
B. Stephen Gray
C. Alessandro Volta
D. Gottfried Wilhelm Leibniz

4. Discovered electromagnetism and invented galvanometer in 1820.


A. J W Ritter
B. Stephen Gray
C. William Herschel
D. Hans Christian Oersted

5. Discovered thermoelectricity in 1821.


A. J W Ritter
B. T J Seebeck
C. Stephen Gray
D. Hans Christian Oersted

6. Formulated Ohm’s Law in 1826.


A. T J Seebeck
B. George S. Ohm
C. Hans Christian Oersted
D. Jean-Baptiste-Joseph Fourier

7. Discovered electromagnetic induction in 1831.


31

TJ Seebeck
A. George S. Ohm
B. Michael Faraday
C. Jean-Baptiste-Joseph Fourier

8. Invented the magnetohydrodynamic battery in 1839.


A. Samuel Morse
B. Charles Babbage
C. Michael Faraday
D. Thomas Davenport

9. Discovered the photovoltaic effect in 1839.


A. Samuel Morse
B. Michael Faraday
C. Edmond Becquerel
D. Thomas Davenport

10. Invented the fuel cell in 1839.


A. William Grove
B. Michael Faraday
C. Edmond Becquerel
D. Thomas Davenport

11. Invented the differential resistance measurer in 1843.


A. John Herschel
B. William Grove
C. Charles Wheatstone
D. Edmond Becquerel

12. Formulated KCL and KVL in 1845.


John Herschel
A. George S. Ohm
B. Gustav Kirchhoff
C. Charles Wheatstone

13. Invented submarine cable insulation in 1847.


A. Charles Wheatstone
B. Gustav Kirchhoff
C. George S. Ohm
D. Werner Siemens

14. Invented the lead acid cell in 1860.


A. Plante
B. Michael Faraday
32

C. George Boole
D. J P Reis

15. Invented the Leclanche cell in 1868.


A. Plante
B. Georges Leclanche
C. James Clerk Maxwell
D. J P Reis

16. Discovered electron in 1897.


A. Joseph John Thomson
B. Guglielmo Marconi
C. Wilhelm Rontgen
D. Almon Brown Strowger

17. Invented Nickel-iron cell in 1900.


A. Joseph John Thomson
B. Guglielmo Marconi
C. Thomas Alva Edison
D. Almon Brown Strowger

18. Discovered superconductivity in 1911.


A. Lee De Forest
B. Thadius Cahill
C. Alessandro Artom
D. Kamerlingh Onnes

19. Discovered neutron in 1932.


A. James Chadwick
B. Tellegen and Hoist
C. H S Black
D. lius Lilienfield

20. Invented Digital Voltmeter in 1952.


A. M V Wilkes
B. J I Nishizawa
C. Yoshire Nakamats
D. Andy Kay
.
.
.
.
.
.
.
33

Assessment No. 2

INTRODUCTION TO ELECTRICITY

Name: Ybañez, Eric L. Score: _________ Rating: ______

Answer the questions in your own words:

1. What is electricity?

2. Write at least five scientists who contributed to the development of electricity?


Include their contributions to electricity.

3. What are the applications of electricity and electronics?

4. Express the following using metric prefixes:


a. 0.00000625 F
b. 23000000000 Ω
c. 0.0000000000234 A
d. 126000 V

5. Add the following without using a calculator. Use metric prefixes to simplify.
a. 0.00000625 F + 0.0000835 F
b. 23000000000 Ω + 45 GΩ
c. 0.0000000000234 A + 0.000000002446 A
d. 126000 V + 923800 V

6. Write the SI units of mass, force, weight, work, power and energy.

7. What is the difference between AC and DC?


.
.
.
.
.
.
.
34

Unit 2

ATOMS, ELECTRICAL CHARGE, VOLTAGE, AND CURRENT

LEARNING
C. OUTCOMES
D.
After completing this
unit, you are expected to:

1. describe the structure of an atom.


2. discuss the behavior of free electrons.
3. calculate the number of free electrons in a material.
4. define electric charges.
5. Calculate the amount of charge in a given number of electrons.
6. define voltage and current.
7. discuss how voltage produces current.
8. solve the current for a given charge and time.
9. solve the voltage for a given energy and amount of charge.
10. identify various types of voltage sources.
11. classify the three basic types of materials.
12. explain the difference among conductors, insulators, and
semiconductors
13. explain the difference between dependent and independent
voltage and current sources.
14. discuss electron flow and conventional current direction.
15. draw symbols of voltage and current sources.
16. Discuss the sign conventions of voltage and current.
35

Important Terms

Matter conductor battery


Element semiconductor primary cell
compound insulator secondary cell
Mixture ion wet cell
Molecule cation dry cell
Atom anion ampere-hour
Proton charge ideal current source
electron coulomb dependent current source
valence electron potential difference electric current
atomic number voltage conventional current flow
atomic mass volt electron flow
saturated orbit ideal voltage source
unsaturated orbit dependent voltage source
Copper cell

2.1 Structure of Matter

Matter – anything that occupies space and has mass.


Element – a substance that cannot be decomposed
any further by chemical action
Compound – a combination of two or more elements in a fixed
proportion,
e.g., H2O, SO2, NaCl, etc.
Mixture – is a collection of two or more elements in any proportion.
Molecule – smallest particle that a compound can be reduced to
before it breaks down into its elements.
Atom – smallest part of an element can be reduced to and still
keeping the properties of the element.

Parts of an atom:

Name Charge Mass (kg) Diameter (m)


Proton Positive charge: +1.602 x 1.672 x 10-27 1/3 of the diameter of
10-19 coulomb electron

Electron Negative charge: - 1.602 x 9.107 x 10-31 10-15


10-19 coulomb
Neutron No charge 1.672 x 10-27 approximately the
same as proton
.
.
36

The structure of an atom:

Nucleus of an atom – center


of the atom. It is where the
protons and the neutrons are
located.

The electrons revolve around


the orbits or shells around
the nucleus.

To determine the number of electrons in each shell this formula applies:

N = 2 n2

where: N = total number of electrons on a given shell


n = nth shell of the atom

Valence Electrons – electrons found in the outermost shell or orbit of an atom.

Atomic Number – represents the number of electrons or protons of an atom.

Atomic mass – represents the sum of protons and neutrons of an atom.

Saturated orbit – the orbit which acquires that much electrons as it should have.

Unsaturated orbit – the orbit which is not satisfied or not having that much number
of electrons which it should possess.

The Copper Atom

Figure 2.1. The copper atom


.
37

Figure 2.1 shows the atomic structure of copper. There are two electrons in the first
shell, eight in the second, eighteen in the third, and only one in the last shell which is
called the valence shell. The following discusses why copper is considered as a
conductor.

The electron energy level:

Rule: Although all electrons have the same negative charges, not all electrons share
the same energy level. The further an electron orbits from the nucleus, the greater its
energy.

Energy Added

1. The energy added to a valence shell is distributed among the valence electrons.
Thus for a given energy, the more valence electrons, the less energy each will get.

2. If enough energy is added to an electron, the electron will move out from its orbit
and move to the next higher orbit. That is, if enough energy is added to a valence
electron, the electron will move out from its atom and becomes a free electron since
there is no more higher orbit.

2.2 Positive and Negative Ions

When an electron is removed from a neutral atom, this atom becomes


positively charged and is called positive ion or cation. However if an electron is
added to a neutral atom, it becomes negatively charged and is called a negative ion
or anion. Thus an atom becomes an ion by the gain or loss of an electron.

2.3 Classification of Materials According to Conductivity


38

Conductors – are substances or materials used to convey or allow the flow of electric
current.
-has 3 or less valence electrons

Materials Considered as Good Electric Conductors are:

1. Silver 7. Zinc
2. Copper 8. Platinum
3. Gold 9. Iron
4. Aluminum 10. Lead
5. Nickel 11. Tin
6. Brass

Semiconductors – are classed below the conductors in their ability to carry current.
- has exactly 4 valence electrons
Silicon and germanium are semiconductor materials.

Insulators – are substances or materials that resist the flow of electric current.
- has 5 or more valence electrons

Various Kinds of Insulators:


1. Rubber 7. Latex
2. Porcelain 8. Asbestos
3. Varnish 9. Paper
4. Slate 10. Oil
5. Glass 11. Wax
6. Mica 12. Thermoplastic

Number of electrons, protons, neutrons, and valence


electrons of some elements.

Element No. of No. of No. of Valence


Name electrons protons neutrons electron(s)
Copper 29 29 34 1
Aluminum 13 13 14 3
Germanium 32 32 41 4
Phosphorus 15 15 16 5

2.4 Number of Free Electrons of Some Common Materials

Silver – 1.68 x 1024 free electrons/cubic inch


Copper – 1.64 x 1024 free electrons/cubic inch
Aluminum – 1024 free electrons/cubic inch
Hard Rubber – 3 free electrons/cubic inch
39

Objective Test No. 3

ATOMS, ELECTRICAL CHARGE, VOLTAGE, AND CURRENT

1. A neutral atom with an atomic number of three has how many electrons?
A. 1
B. 3
C. 4
D. 5

2. Electrons orbits are called ________.


A. waves
B. shells
C. nuclei
D. valences

3. Materials in which no current can flow are called ____________.


A. filters
B. conductors
C. insulators
D. semiconductors

4. Electrons in the last orbit of an atom are called ___________.


A. free electrons
B. bound electrons
C. valence electrons
D. stationary electrons

5. Component of an atom that doesn’t have any electrical charge is a/an ________.
A. proton
B. neutron
C. electron
D. positron

6. If the number of valence electrons is exactly four, the material is called _______.
A. a conductor
B. an insulator
C. a superconductor
D. a semi-conductor

7. An atom that acquires additional electrons is called ___________.


.
.
.
.
.
40

A. anion
B. anode
C. cation
D. cathode

8. In order for a material to be called a conductor, what is the maximum number


of valence electrons it can have?
A. one
B. two
C. three
D. four

9. Of the following materials the only conductor is _____________.


A. slate
B. latex
C. brass
D. asbestos

10. A three-layer semiconductor device


A. diode
B. transistor
C. vacuum tube
D. potentiometer

11. Copper when exposed to ordinary atmosphere becomes oxidized turning into
a _____ color
A. black
B. brown black
C. light gray
D. light orange

12. At absolute zero temperature a semi-conductor behave as a __________.


A. good insulator
B. good conductor
C. super conductor
D. variable resistor

13. Copper is a highly malleable and ductile metal with ______ color.
A. amber
B. brown
C. reddish brown
D. bluish-white

14. If an atom has less than 4 valence electrons, the material is _______.
.
.
.
41

A. a conductor
B. an insulator
C. a superconductor
D. a semi-conductor

15. Which of the following is NOT an insulator?


A. mica
B. porcelain
C. ceramic
D. carbon

16. A material with atoms in which the electrons tend to stay in their orbits is
called _____.
A. inductor
B. intrinsic
C. insulator
D. conductor

17. Which of the following is the poorest conductor of electricity?


A. steel
B. silver
C. carbon
D. aluminum

18. Which of the following metals has the highest electrical and thermal
conductivity?
A. gold
B. silver
C. platinum
D. palladium

19. A substance that cannot be decomposed any further by a chemical reaction is


called ___
A. ion
B. alloy
C. element
D. molecule

20. Which one is a semi-conductor?


A. diamond
B. arsenic
C. phosphorous
D. gallium arsenide

21. If an atom losses some of its electron or accepts extra electrons from another
atom, the atom will be called _____________.
42

A. an ion
B. a lattice
C. a neutron
D. an element

22. Most common semi-conductor in use today is __________.


A. arsenic
B. silicon
C. germanium
D. gallium arsenide

23. Which of the following metals has the highest melting point?
A. gold
B. silver
C. copper
D. tungsten

24. The atomic number of an element is determined by:


A. The number of neutrons.
B. The number of protons.
C. The number of neutrons plus the number of protons.

25. The atomic weight of an element is approximately determined by:


A. The number of neutrons.
B. The number of protons.
C. The number of neutrons plus the number of protons.
D. The number of electrons.

26. An ion:
A. Is electrically neutral.
B. Has positive electric charge.
C. Has negative electric charge.
D. Might have either a positive or negative charge.

27. In a compound:
A. There can be just a single atom of an element.
B. There must always be two or more elements.
C. The atoms are mixed in with each other but not joined.
D. There is always a shortage of electrons.

28. An electrical insulator can be made a conductor:


A. By heating.
B. By cooling.
C. By ionizing.
D. By oxidizing.
43

29. Of the following substances, the worst conductor is:


A. Air.
B. Copper.
C. Iron.
D. Salt water.

30. Of the following substances, the best conductor is:


A. Air.
B. Copper.
C. Iron.
D. Salt water.

31. If a material has low resistance:


A. It is a good conductor.
B. It is a poor conductor.
C. The current flows mainly in the form of holes.
D. Current can flow only in one direction.

32. The cation has ____.


A. more number of electrons than protons.
B. more number of protons than electrons.
C. equal number of protons and neutrons.
D. equal number of protons , electrons and neutrons.

33. The charged atom is called ______.


A. molecule
B. ion
C. nucleus
D. orbit

34. The mass of the electron is _____.


A. 1.602 x 10-31 kg
B. 9.11 x 10-31 kg
C. 6.24 x 10-18 kg
D. 1.371 x 10-31 kg

35. The proton is ________ times more than the mass of one electron.
A. 1836
B. 1736
C. 1386
D. 1639
36. Which is the best conductor?
44

A. iron
B. copper
C. silver
D. aluminum

37. Which is the best insulator?


A. copper
B. dry air
C. moist air
D. mica

38. The lightest particle of an atom ______


A. neutron
B. proton
C. electron
D. nucleus

39. The heavy particle of the atomic configuration is ________.


A. electron
B. proton
C. free electron
D. neutron

40. If any electron is removed from an atom, it becomes _____.


A. positive ion
B. negative ion
C. conductor
D. insulator

41. Metals are good conductors of electricity because they have


A. their valence orbit causes less attracting force between electrons and
nucleus.
B. their valence orbit is completely filled
C. their valence orbit causes great force between free electrons and nucleus
45

2.5 Electric Charge

Charge is an electrical property of the atomic particles of which matter consists,


measured in coulombs (C).

A body is said to be charged, if it has either an excess or deficit of electrons from


its normal values due to sharing.

Coulomb (C) – unit of electric charge which is equivalent to 6.24 x 1018 electrons or
protons.
- named after the French physicist, Charles A. Coulomb (1736 – 1806).

The following points should be noted about electric charge:

1. The coulomb is a large unit for charges. In 1 C of charge, there are 1/(1.602 x
1018) = 6.24 x 1018 electrons. Thus realistic laboratory values of charges are on
the order of pC, nC, or µC.
2. According to experimental observation, the only charges that occur in nature are
integral multiples of the electronic charge e = -1.602 x 10-19 C.
3. The law of conservation of charge states that charge can neither be created nor
destroyed, only transferred. Thus the algebraic sum of the electric charges in a
system does not change.

Example 2.1 How many coulombs do 93.75 x 1016 electrons represent?

Solution:
number of electrons
Q = number of electrons in one coulomb

93.75 x 1016 electrons


= 6.25 x 1018 electrons/C

= 0.15 C
46

Problem Set No. 2

CHARGES

1. What is the symbol for charge?


A. C
B. G
C. Q
D. E

2. What is the unit of charge, and what is its symbol?


A. Coulomb, C
B. Coulomb, Q
C. Farad, F
D. Volt, V

3. How much charge, in coulombs, is there in 10 x 1012 electrons?


A. 1.6 µC
B. 16 µC
C. 160 mC
D. 1.6 nC

4. How many coulombs of charge do 50 x 1013 electrons possess?


A. 80 µC
B. 8.0 µC
C. 180 mC
D. 8.0 nC

5. How many electrons does it take to make 60 C of charge?


A. 3.75 x 1011
B. 3.75 x 1012
C. 3.75 x 1013
D. 3.75 x 1014
.
47

2.6 Potential Difference (Voltage)

Potential – the capability of doing work

Any charge had the capability of doing work of moving another charge either by
attraction or repulsion.

The net number of electrons moved in the direction of the positive charge plate
depends upon the potential difference between the two charges.

Volt (V) – unit of potential difference which is equal to one joule of work done per
coulomb of charge.

Potential difference in electrical terms is more commonly called voltage (V) and is
expressed as energy (W) per unit charge (Q):
W
V = Q

where: W is expressed in Joules (J) and Q is in coulombs(C).

The unit of voltage is the volt, symbolized by V.


- named after the Italian physicist, Alessandro C. Volta (1754 – 1827)
who invented the first electric battery.

One volt is the potential difference (voltage) between two points when
one joule of energy is used to move one coulomb of charge from one
point to the other.

We assume that we are dealing with a differential amount of charge and energy,
then

dw
v = dq

Example 2.2 If 50 J of energy are available for every 10 C of charge, what is the
voltage?

Given: W = 50 J
Q = 10 C

Find: voltage

W
Known: V = Q
.
48

50 J
Solution: V= = 5V
10 C

Answer: 5V

Example 2.3 An energy of 20 Joules is required in moving a 2-coulomb charge from


point A to B. What is the potential difference between point A and B?

Given: W = 20 J
Q=2C

Find: p.d. or voltage between two points


W
Known: V = Q

20 J
Solution: V= = 10 V
2C

Answer: 5V

2.7 The Idea of Electric Potential

LOAD

Direction of electron
flow

zinc
copper

H2SO4

Figure 2.2. A Simple Voltaic Cell


49

In Fig.2.2, a simple voltaic cell is shown. It consists of copper plate (known as anode)
and a zinc rod (i.e. cathode) immersed in dilute sulphuric acid (H2SO4) contained in a
suitable vessel. The chemical action taking place within the cell causes the electrons
to be removed from copper plate and to be deposited on the zinc rod at the same time.
This transfer of electrons is accomplished through the agency of the diluted H2SO4
which is known as the electrolyte. The result is that zinc rod becomes negative due to
the deposition of electrons on it and the copper plate becomes positive due to the
removal of electrons from it. The large number of electrons collected on the zinc rod
is being attracted by anode but is prevented from returning to it by the force set up
by the chemical action within the cell.
But if the two electrodes are joined by a wire externally, then electrons rush to
the anode thereby equalizing the charges of the two electrodes. However, due to the
continuity of chemical action, a continuous difference in the number of electrons on
the two electrodes is maintained which keeps up a continuous flow of current through
the external circuit.

2.8 Analogy of Electrical Potential Difference


50

2.9 Voltage Sources and Their Symbols

Ideal Voltage Source

A voltage source which has zero resistance.

The Ideal Independent Voltage Source

This is a circuit element that maintains a prescribed voltage across its


terminals regardless of the current through it.

E
V or
+ 12V
-

Battery

The Ideal Dependent Voltage Source

This is a voltage source in which a voltage or a current at some other part of


the circuit determines the voltage across its terminals.

+
V -

Sources of Voltage

1. The Battery

A voltage source is a source of potential energy that is also called electromotive force
(emf). The battery is one type of voltage source that converts chemical energy into
electrical energy. A voltage exists between the electrodes (terminals) of a battery, as
shown by a voltaic cell in the figure. One electrode is positive and the other negative
as result of the separation of charges caused by the chemical action when two
different conducting materials are dissolved in the electrolyte.

Difference Between Cell and a Battery

Cell – is composed of two dissimilar metals, which are immersed in a conductive


liquid or paste called an electrolyte. (Electrolysis is the process of converting
chemical energy to electrical energy).
51

Battery – a combination of cells

Classification of Chemical cells (or battery)


a. Primary cells are ordinarily not usable after a certain period of time.
After this period of time its chemicals can no longer produce
electrical energy.

b. Secondary cells can be renewed after they are used, by reactivating


the chemical process that is used to produce electrical energy. This
reactivation is known as charging.

Classification of Cells According to Type of Chemicals Used:

a. Wet Cell – uses liquid chemicals


b. Dry Cell – contains a chemical paste

Types of Primary and Secondary Cells


Cell Name Open Circuit Cell Type
Voltage
Carbon-zinc 1.5 V Primary
Alkaline Primary
Zinc-chloride 1.5 V Primary
Zinc air cells
Manganese-zinc 1.5 V Primary or Secondary
Mercury-oxide 1.35 V Primary
Silver-oxide 1.5 V Primary
Lithium 3.0 V Primary
Rechargeable alkaline Secondary
Nickel metal hydride
Lead-acid 2.1 V Secondary
Nickel-cadmium 1.25 V Secondary
Nickel-iron 1.2 V Secondary
Nickel ion Secondary
Lead-acid Secondary
Silver-zinc 1.5 V Secondary
Silver-cadmium 1.1 V Secondary
52

Sizes for Popular Types of Dry Cells

Size Height (inch) Diameter (inch)


D 2¼ 1¼
C 1¾ 1
AA 1 7/8 9/16
AAA 1¾ 3/8

2.10 Ampere-Hour Rating of Secondary Cells

The capacity of a battery composed of lead-acid cells is given by an ampere-


hour rating. A 60-ampere hour battery could theoretically deliver 60 amperes
for 1 hour, 30 amperes for 2 hours, or 15 amperes for 4 hours. However, this
is an approximate rating dependent upon the rate of discharge and the
operating temperature of the battery. The normal operating temperature is
considered to be 80 F.

The ampere-hour rating is a measure of the energy the battery stores;


consequently, the energy transferred for total discharge is the same whether
it is transferred in 10 hours or 20 hours. Since power is the rate of energy
transfer, the power for a 10-hour discharge is twice that in a 20-hour
discharge.

2. The Electronic Power Supply


3. The Solar Cell
4. The Generator
53

Problem Set No. 3

ELECTRICAL POTENTIAL

1. How much is the voltage when there are 4 J of energy for 10 C of charge?
A. 0.004 V
B. 0.25 V
C. 0.4 V
D. 2.5 V

2. How much energy does a 12-V battery use to move 2.5 C through a circuit?
A. 0.4 J
B. 2.5 J
C. 30 J
D. 120 J

3. Work equal to 136.0 joules is expended in moving 8.5 x 1018 electrons between
two points in an electric circuit. What potential difference does this establish
between the two points?
A. 1.0 V
B. 10 V
C. 100 V
D. 1000 V

4. Electrical energy is converted to heat at the rate of 7.56kJ/min in a resistor which


has 270 C/min through it. What is the voltage difference across the terminals?
A. 35.71 V
B. 30.8 V
C. 28 V
D. 21.5 V

5. A typical 12-V auto battery is rated according to ampere-hours. A 70-Ah battery,


for example, at a discharge rate of 3.5 A has a life of 20 h. Assuming the voltage
remains constant, obtain the energy and power delivered in a complete discharge
of the preceding battery.
A. 800 J, 40 W
B. 820 J, 42 W
C. 800 J, 45 W
D. 840 J, 42 W

6. Repeat for a discharge rate of 7.0 A.


A. 840 J, 42 W
B. 820 J, 84 W
C. 840 J, 84 W
D. 840 J, 48 W
54

2.11 Electric Current: Charge in Motion

Random motion of free electrons in a material.

When a potential difference between two charges forces a third charge to move, the
charge in motion is called an electric current.

_ + +

Electrons flow from negative to positive when a voltage is applied across a


conductive material.

The movement of free electrons from the negative end of the material to the positive
end is the electrical current, symbolized by I.

Electrical current is defined as the rate of flow of electrons in a


conductive material.

Current is measured by the number of electrons (amount of charge, Q) that flows


past a point in a unit of time:
Q
I = t
where: I is the current, Q is the charge of the electrons, and t is
the time.

Ampere (A) – unit of charge flow equal to one coulomb of charge past a given point
in one second.
-named after the French physicist and mathematician Andre M.
Ampere (1175 – 1836)

If there a non-linear relationship between charge and time, the current is

dq(t)
i(t) =
dt
.
.
55

t
and q(t) = ∫ i(t)dt or q(t) = ∫t 2 i(t)dt
1

Electric current is the time rate of change of charge, measures in amperes (A).

Example 2.4 Ten coulombs of charge flow past a given point in a wire in 2 s. What
is the current?

Given: Q = 10 C , t = 2 sec

Find: current I

Q
Known: I =
t

𝟏𝟎 𝐂
Solution: 𝐈= = 𝟓𝐀
𝟐𝐬

Answer: 5A

Example 2.5 How many electrons pass a given point in 40 seconds in a conductor
carrying 10 amps?

Given: t = 40 s , I = 10 A

Find: no. of electrons

Known: Q = It
charge in coulomb
no. of electrons = 1.602 x 10−19 coulmb/electron

Solution: 𝐐 = 𝐈𝐭 = (𝟏𝟎)(𝟒𝟎) = 𝟒𝟎𝟎 𝐂


𝟒𝟎𝟎
𝐧𝐨. 𝐞𝐥𝐞𝐜𝐭𝐫𝐨𝐧𝐬 = 𝟏.𝟔𝟎𝟐𝐱 𝟏𝟎 −𝟏𝟗 = 𝟐𝟓 𝐱 𝟏𝟎𝟐𝟎 electrons

Answer: 25 x 1020 electrons

Example 2.6 Determine the current flowing through an element if the charge flow is
(a) q(t) = (3t + 8) mC (b) q(t) = (3e-t – 5e-2t) nC

dq(t)
Known: i(t) = dt
.

d(3t+8)
Solution: (a) i(t) = dt
56

3dt
= +0
dt

i(t) = 3 𝑚𝐴

Example 2.7 Determine the total charge transferred over the time interval of 0  t 
1
10 s when i(t) = 2t .

t
Known: q(t) = ∫t 2 i(t)dt
1

10 1
Solution: q(t) = ∫0 t dt
2

10
1 𝑡2
= [ ]
2 2 0
2
1 102 − 0
= [ ]
2 2

q(t) = 25 C

Example 2.8 The charge that enters the BOX is shown below. Calculate and sketch
the current flowing into the BOX between 0 and 10 milliseconds.

i(t) C1
E BOX
12 V 1µF
57

4
q(t) (mC)
3

0
0 1 2 3 4 5 6 7 8 9 10
-1
t (ms)

-2

-3

dq(t)
Solution: Recall that current is related to charge by i(t) = . The
dt
current is equal to the slope of the charge waveform.

𝐢(𝐭) = 𝟎 𝟎  𝐭  𝟏 𝐦𝐬

𝟑 𝐱 𝟏𝟎−𝟑 − 𝟏 𝐱 𝟏𝟎−𝟑 𝟏  𝐭  𝟐 𝐦𝐬
𝐢(𝐭) = = 𝟐𝐀
𝟐 𝐱 𝟏𝟎−𝟑 − 𝟏 𝐱 𝟏𝟎−𝟑

𝐢(𝐭) = 𝟎 𝟐  𝐭  𝟑 𝐦𝐬

−𝟐 𝐱 𝟏𝟎−𝟑 − 𝟑 𝐱 𝟏𝟎−𝟑 𝟑  𝐭  𝟓 𝐦𝐬
𝐢(𝐭) = = −𝟐. 𝟓 𝐀
𝟓 𝐱 𝟏𝟎−𝟑 − 𝟑 𝐱 𝟏𝟎−𝟑

𝐢(𝐭) = 𝟎 𝟓  𝐭  𝟔 𝐦𝐬

𝟐 𝐱 𝟏𝟎−𝟑 − (−𝟐 𝐱 𝟏𝟎−𝟑 ) 𝟔  𝐭  𝟗 𝐦𝐬


𝐢(𝐭) =
𝟗 𝐱 𝟏𝟎−𝟑 − 𝟔 𝐱 𝟏𝟎−𝟑
= 𝟏. 𝟑𝟑 𝐀

𝐢(𝐭) = 𝟎 𝐭  𝟗 𝐦𝐬
.
.
.
.
.
.
58

0
0 1 2 3 4 5 6 7 8 9 10
-1
t (ms)
-2

-3

Example 2.9 The current in a conductor varies as follows: during the first 2 sec there
is a linear change from zero to 5 amp; during the next 4 sec the current is constant at
5 amp; during the third period of 6 sec the current decreases linearly to 2 amp.
Determine the total charge transferred in the elapsed time of 12 sec.
6
i (A)

0
0 1 2 3 4 5 6 7 8 9 10 11 12 t (s)

Solution: Since q(t) = ∫ i(t)dt . Charge is equal to the area bounded by the x and y
axes.
1 1
q = (2)(5) + (4)(5) + (6)(5 + 2) = 46 C
2 2
59

2.12 Electron Drift Velocity

Suppose that in a conductor, the


number of free electrons available per m3 The electron moves at the Fermi speed,
of the conductor material is n and let and has only a tiny drift velocity
their superimposed by the applied electric field.
axial drift velocity be ν metres/second. In
time dt, distance travelled would be ν ×
dt. If A is area of cross-section of the
conductor, then the volume is νAdt and
the number of electrons contained in this
volume is νA dt. Obviously, all these
electrons will cross the conductor cross- Drift
section in time dt. If e is the charge of vd velocity
each electron, then total charge which Electric Field
crosses
the section in time dt is dq = nAeν dt.
Since current is the rate of flow of charge, it is given as

dq nAevdt
i = = i = nAev
dt dt

Current density; J = i/A = nev ampere/meter2

Assuming a normal current density J = 1.55 x 106 A/m2 , n = 1029 for a copper
conductor and e = 1.6 x 10−19 coulomb, we get

1.55 x 106 = 1029 x 1.6 x 10−19 x v v = 9.7 x 10−5 m/s = 0.58 cm/min

It is seen that contrary to the common but mistaken view, the electron drift velocity
is rather very slow and is independent of the current flowing and the area of the
conductor.

Example 2.10 A conductor material has a free-electron density of 1024 electrons per
metre3. When a voltage is applied, a constant drift velocity of 1.5 × 10−2 meter/second
is attained by the electrons. If the cross-sectional area of the material is 1 cm2,
calculate the magnitude of the current. Electronic charge is 1.6 × 10−19 coulomb.

Solution: The magnitude of the current is


i = nAeν amperes
Here, n = 1024 ; A = 1 cm2 = 10−4 m2
e = 1.6 × 10 − 19 C ; v = 1.5 × 10−2 m/s

 i = 1024 × 10 −4 × 1.6 × 10−19 × 1.5 × 10−2 = 0.24 A

2.13 Charge Velocity and Velocity of Field Propagation


60

The speed with which charge drifts in a conductor is called the velocity of
charge. As seen from above, its value is quite low, typically fraction of a meter per
second.
However, the speed with which the effect of e.m.f. is experienced at all parts of
the conductor resulting in the flow of current is called the velocity of propagation of
electrical field. It is independent of current and voltage and has high but constant
value of nearly 3 × 108 m/s.

Example 2.11 Find the velocity of charge leading to 1 A current which flows in a
copper conductor of cross-section 1 cm2 and length 10 km. Free electron density of
copper = 8.5 × 1028 per m3. How long will it take the electric charge to travel from
one end of the conductor to the other?

i
Solution. i = neAν or ν = neA

1
ν = = 7.35 × 10−7 m/s = 0.735 μm/s
(8.5 × 1028 × 1.6 × 10−19 × 1 × 10−4)

Time taken by the charge to travel conductor length of 10 km is

distance 10 x 103
t = velocity = 7.35 × 10−7 = 1.36 x 1010 s

Now, 1 year = 365 × 24 × 3600 = 31,536,000 s

1.36 × 1010
t = 31,536,000 = 431 years Answer

2.14 Current Sources

Ideal Current Source

- a current source which has a very high resistance.

The Ideal Independent Current Source

This is a circuit element that maintains a prescribed current in its terminals


regardless of the voltage across it.

I
61

The Ideal Dependent Current Source

This is a current source in which either a voltage or a current at some other


part of the circuit determines the current in its terminals.

The Conventional Direction of Current and Electron Flow

V R V R
10ohm 10ohm
12V 12V

Conventional Direction of Current Electron Flow

Conventions of Voltage

_
+ a a

V1 V1
12V -12 V
12 V
12V

_ b + b

vab = - vba
.
Since the polarity is reversed the value becomes –12 V .

3A -3 A

The current becomes –3 A because the direction of the original current is


reversed.
62

Assessment No. 3

Name: Ybañez, Eric L. Score: _________ Rating: ______

How Much Have You Learned?

Direction: Solve the crossword puzzle. Use the given clues to arrive at the right
answer.
1 2 5 6

3 4
2 7
5

8
3 9
7
10
6
11
8 12 9 13

14 10 15

11

12

13

DOWN ACROSS
1 negative to positive current flow 1 a substance that cannot be decomposed further
2 smallest particle of a compound 2 no charge at all
3 positive charge 3 central part of an atom
4 quantity of electricity 4 unit of charge
5 potential difference 5 smallest particle of an element
6 H20, NaCl, etc. 6 negative charge
I
7 7 excess or deficiency of electrons
1A

8 cell that is not chargeable 8 outermost orbit


9 carbon-zinc 9 composed of many cells
10 unit of voltage 10 a blend
11 anything that occupies space 11 constituted with current
12 copper, silver, gold, sulfur, etc. 12 an insulator
13 positive ion 13 a cell that can be recharged
14 conductor with the highest
conductivity
15 used as a bulb filament
63

QUESTIONS: (not part of Assessment No. 3)

1. How do you compare the structure of an atom?


2. What are free electrons?
3. What is an electric charge.
4. What is voltage?
5. How does voltage exists between two points?
6. What is an electric current?
7. How does voltage produces current?
8. What are the types of voltage sources.
9. What are the three basic types of materials.
10. Explain the difference among conductors, insulators, and semiconductors
11. Explain the difference between dependent and independent voltage and
current sources.
12. What is the difference between electron flow and conventional current
direction?
13. Discuss the sign conventions of voltage and current.
64

Assessment No. 4

CHARGE AND CURRENT

Name: Ybañez, Eric L. Score: _________ Rating: ______

1. For t ≥ 0, q = (4.0 x 10 −4 )(1 – e− 250t ) C. Obtain the current at t = 3 ms.

2. The current equation the flows in a circuit is i = 10e-t. Find the charge transferred
from 0 ms to 5 ms.
65

3. Find the charge that has entered the terminal of an element from t = 0 s to t = 3 s
when the current entering the element as shown in the figure below.

i (A)

0 1 2 3 t (s)
-1

4. The charge that enters the BOX is shown in the figure below. Calculate and sketch
the current flow into the BOX between 0 and 9 milliseconds.
66

i(t) C1
E BOX
2412VV 1µF

q(t) (mC)
2.5

1.5

0.5

0
0 1 2 3 4 5 6 7 8 9 t (ms)
-0.5

-1

-1.5
67

Practical Application No. 1

Name: Ybañez, Eric L. Score: _________ Rating: ______

1. Suppose that your car will not start. To determine whether the battery is faulty,
you turn on the light switch and find the lights are dim, indicating a weak
battery. You borrow a friend’s car and a set of jumper cables. However, how do
you connect his car’s battery to yours? What do you want his battery to do?

2. A Universal Serial Bus (USB) port is a common feature on both desktop and
notebook computers as well as many handheld devices such as MP3 players,
digital cameras, and cell phones. The USB 2.0 specification (www.usb.org)
permits data transfer between a computer and a peripheral device at rates up to
480 megabits per second. One important feature of USB is the ability to swap
peripherals without having to power down a computer. USB ports are also
capable of supplying power to external peripherals. A USB cable is a four-
conductor cable with two signal conductors and two conductors for providing
power. The amount of current that can be provided over a USB port is defined in
the USB specification in terms of unit loads, where one unit load is specified to
be 100 mA. All USB ports default to low-power ports at one unit load, but can be
changed under soft-ware control to high-power ports capable of supplying up to
five unit loads or 500 mA.
a. A 680 mAh lithium-ion battery is standard in a Motorola RZZR®. If this
battery is completely discharged (i.e., 0 mAh), how long will it take to recharge
68

the battery to its full capacity of 680 mAh from a low-power USB port? How
much charge is stored in the battery at the end of the charging process?
b. A third-generation iPod® with a 630 mAh lithium-ion battery is to be
recharged from a high-power USB port supplying 150 mA of current. At the
beginning of the recharge, 7.8 C of charge are stored in the battery. The
recharging process halts when the stored charge reaches 35.9 C. How long does
it take to recharge the battery?
69

Problem Set No. 4

ELECTRIC CHARGE AND CURRENT

1. What is the current in amperes when 20 C flow past a point in a wire in 4 s?


A. 5 A
B. 10 A
C. 15 A
D. 20 A

5. How long does it take 10 C to flow past a point if the current is 5 A?


A. 0.5 sec
B. sec
C. sec
D. 50 sec

6. How many coulombs pass a point in 0.1 s when the current is 1.5 A?
A. 0.15 C
B. 1.5 C
C. 15 C
D. 150 C

7. How many electrons pass a given point in 40 sec in a conductor carrying 10 A?


A. 1.2 x 1020
B. 2.1 x 1020
C. 2.5 x 1020
D. 2.5 x1021

8. A charge of 45 nC passes through a circuit during a particular interval of time that


is 5 ms in duration. Determine the average current in this circuit element during
the interval of time.
A. 9 mA
B. 9 µA
C. 9 nA
D. 9 pA

9. Ten billion electrons per second pass through a particular circuit element. What
is the average current in that circuit element?
A. 1.6 mA
B. 1.6 µA
C. 1.6 nA
D. 1.6 pA

10. Calculate the number of electrons in a copper conductor having a diameter of


0.064 in. and a length of 1,000 ft.
A. 6.22 x1024
70

B. 6.33 x 1025
C. 7.33 x 1026
D. 8.12 x 1026

11. The current in a conductor varies as follows: during the first 8 sec there is a linear
change from zero to 4 amp; during the next 15 sec the current is constant at 4
amp; during the third period of 20 sec the current decreases linearly to 3 amp.
Determine (a) the total charge transferred in the elapsed time of 43 sec, (b) the
average current.
A. 126 C, 3.3 A
B. 136 C, 3.2 A
C. 146 C, 3.4 A
D. 156 C, 3.4 A

12. The current density in the wire of an electric machine is not to exceed 4,500 amp
per sq in. What is the maximum permissible current in each conductor if its
diameter is 0.032 in.?
A. 3.62 A
B. 3.89 A
C. 4.22 A
D. 5.72 A
Objective Test No. 4
POTENTIAL DIFFERENCE

1. The charge on a single electron is


A. 3.14 x 10-6 C
B. 6.25 x 10-18 C
C. 1.6 x 10-19 C
D. 1.6 x 10-19 J

2. Potential difference is another term for


A. charge
B. voltage
C. energy

3. Electrical current is defined as


A. free electrons
B. the charge on free electrons
C. the rate of flow of free electrons
D. the energy required to move electrons

4. Another name for a secondary cell


A. wet cell
B. dry cell
71

C. storage cell
D. disposable cell

5. The energy stored in an electrolytic cell is _________________.


A. a chemical
B. a magnetic
C. an electrical
D. a mechanical

6. SI unit of potential difference.


A. volt
B. statvolt
C. volt per ohm
D. coulomb per volt

7. A battery is a group of cells connected in ___________.


A. series only
B. parallel only
C. series-parallel only
D. series, parallel and series-parallel

8. Voltage across an electric circuit, acts as


A. force
B. mass of electrons
C. negative ions
D. component of current

9. Electromotive force is measured by a ___________.


A. megger
B. voltmeter
C. galvanometer
D. clamp ammeter

10. The presence of current is only made known by the effect it produces. Three
important effects are:
A. heating, magnetic and chemical
B. heating, magnetic and electric shock
C. heating, electric shock and generation
D. generation, chemical and electric shock

11. Most commonly used cell.


A. lithium cell
B. lead-acid cell
72

C. silver-zinc cell
D. nickel-iron cell

12. The nominal open circuit voltage of a carbon-zinc cell is


A. 1.35 V.
B. 1.5 V.
C. 2.1 V.
D. 3.0 V.

13. The smallest size of a dry cell.


A. size C
B. size D
C. size AA
D. size AAA

14. A Leclanche cell is a __________________.


A. lead-acid cell
B. zinc-silver oxide cell
C. carbon-zinc dry cell
D. cadmium-mercuric oxide

15. Volt is the same as


A. watt per ohm
B. joule per second
C. ampere per ohm
D. joule per coulomb

16. ________ is a quantity of electricity transported in one second by a current of one


ampere.

A. Watt
B. Joule
C. Coulomb
D. Electron-volt

17. An active element on a circuit is one that ____ to the circuit.


A. supplies energy
B. does not supply energy
C. receives the energy supplies

18. The potential difference that exists across the space between two electrically
connected materials is called contact potential difference or _______.
A. Volta effect
B. Seebeck effect
73

C. Joulian effect
D. Flywheel effect

19. An ideal current source has a _________ internal resistance.


A. zero
B. negative
C. very low
D. very high

20. The rating of a storage battery is expressed in


A. watts
B. volt-amperes
C. ampere-hours
D. kilowatt-hours

21. Cells are connected in parallel to increase


A. the voltage capacity of the cells
B. the current capacity of the cells
C. the resistance capacity of the cells

22. A battery is rated 200 Ah. If it is used to supply a constant current of 8 A, how
long can the battery last until it becomes unusable?
A. 15 hrs
B. 20 hrs
C. 25 hrs

23. The average dry cell gives an approximate voltage of


A. 1.1 V
B. 1.3 V
C. 1.5 V
D. 1.7 V

24. A cell which cannot be recharged.


A. primary
B. secondary

25. The negative plate of a nickel-iron storage battery is ____.


A. iron
B. lead
C. steel
D. nickel

26. A battery can supply 10 joules of energy to move 5 coulombs of charge. What is
the voltage of the battery?
74

A. 2V
B. 5V
C. 15 V
D. 50 V

27. A cell supplies a load current of 0.5 A for a period of 20 hours until its terminal
voltage falls to an unacceptable level. How long it could be expected to supply
a current of 100 mA?
A. 50 hours
B. 60 hours
C. 70 hours
D. 100 hours

28. A coulomb:
A. Is one ampere in a second.
B. Flows through a 100-watt light bulb.
C. Represents a current of one ampere.
D. Is an extremely large number of charge carriers.

29. A stroke of lightning:


A. Has a very low current.
B. Builds up between clouds.
C. Is a discharge of static electricity.
D. Is caused by a movement of holes in an insulator.

30. The volt is the standard unit of:


A. Charge.
B. Current.
C. Resistance.
D. Electromotive force.

31. In some batteries, chemical energy can be replenished by:


A. Charging it.
B. Discharging it.
C. Connecting it to a light bulb.

32. Light is converted into electricity:


A. In a dry cell.
B. In a wet cell.
C. In a photovoltaic cell.

.
75

Assessment No. 5

ATOMS, ELECTRICAL CHARGE, VOLTAGE, AND CURRENT

Name: Ybañez, Eric L. Score: _________ Rating: ______

Answer the questions in your own words:

1. What are free electrons?

2. What is the importance of free electrons?

3. How many free electrons are there in a 200- m copper wire with a diameter of 5
mm?

4. What are electric charges?

5. What is the charge of the wire in number 3?

6. What is voltage? current?

7. How many coulombs of electric charge pass through a lamp in 1 min if the current
is a steady at 300 mA?

8. What PD must be developed across the terminals of a lamp in order for a flow of
0.05 C from one terminal to the other to release 6 J of energy?

9. What are the various types of voltage sources


76

10. .What are the three types of materials and discuss each type?

11. Explain the difference between dependent and independent voltage and current
sources.

12. Discuss the difference between electron flow and conventional current direction.

13. Discuss the sign conventions of voltage and current.


77

.
Unit 3

RESISTANCE

B.
LEARNING
C. OUTCOMES

After completing this


unit, you are expected to:

1. define resistance.
2. discuss how resistance restricts current.
3. identify the factors that affect the resistance of conductors.
4. find the resistance of conductors.
5. identify various types of fixed and variable resistors.
6. determine resistance value by color code.
7. define conductance.
8. calculate the cross-sectional area of a wire or cable in terms of
circular mils.
9. convert circular mils to other units.
10. compute the insulation resistance of high voltage cables.
11. select the proper size or wattage of a resistor.
12. discuss the effect of temperature on the resistance of a
conductor.
78

Important Terms

Resistance wire
ohm cable
resistor stranded wire
Conductance solid wire
mil resistivity
circular mil

3.1 The Meaning of Resistance

When current flows in a material, the free electrons move through the material and
occasionally collide with atoms. These collisions cause the electrons to lose their
energy, and thus their movement is restricted. The more collisions, the more flow of
electrons is restricted. This restriction varies and is determined by the type of
material. The property of a material that restricts the flow of electrons is resistance,
designated R.

Resistance is the opposition to current.

1kohm

Symbol of resistor or any resistive load with resistance R.

Ohm: The unit of Resistance

Resistance, R, is expressed in the unit of ohms, named after Georg Simon Ohm
symbolized by the Greek letter omega ()

There is one ohm (1) of resistance when there is one ampere (1 A) of


current through a material with one volt (1 V) applied.
79

Why does a conductor heats up when electric current flows through it?

Current

3.2 Resistors

Circuit elements that are specially designed to have a certain amount of


resistance are called resistors.

3.2.1 Linear and Non-linear Resistors

A linear resistor is one which obeys Ohm’s law. A circuit which contains only
linear components is called a linear circuit.
Such elements in which the V/I (volt-ampere) plots are not straight lines but
curves are called non-linear resistors or non-linear elements.

A resistor entails the following two main characteristics:


1. It resistance (R) in ohms. The resistors are available from a fraction of an
ohm to many mega ohms
2. The wattage rating. The power rating may be as high as several hundred
watts or as low as 1/10 watt. Power rating indicates the maximum wattage
the resistor can dissipate without excessive heat. (Too much heat can make
the resistor burn open).

3.2.2 Classification of Resistors

The resistors are classified as follows:

 Fixed resistors
 Tapped resistors
 Variable resistors
 Special resistors

Fixed Resistors
80

Carbon-Composition Type
Carbon film
Metal Oxide
Metal film
Metal glaze
Wire-wound

Variable Resistors – can either be wire wound or carbon composition.

Potentiometer - a variable resistance with 3 terminals (the purpose is


to vary the voltage between the center terminals and the ends).

Rheostat – a variable resistance with 2 terminals connected in series


with the load (the purpose of which is to vary the current).

The following types of resistors are used in electric circuits:

1. Carbon resistors
2. Wire-wound resistors on ceramic or plastic forms (as in case of rheostat, etc)
3. Deposited carbon resistors on ceramic base
4. Deposited metal resistors on ceramic base
5. Printed, painted or etched circuit resistors

3.3 Resistance of Conductors

The resistance R offered by a conductor depends on the following factors:


(i) It varies directly as its length, l.
(ii) It varies inversely as the cross-section A of the conductor.
(iii) It depends on the nature of the material.
(iv) It also depends on the temperature of the conductor.

Current

l
l
R=
1m

1m
A 1m
A
Smaller l Larger l
Larger A smaller A
Low R Greater R
Figure 3.1 Figure 3.2
81

Neglecting the last factor for the time being, we can say that

l
R = A

In terms of volume

l2 V
R = = 2
V A

where:

R - the resistance in ohms, 


 - the resistivity or specific resistance of the conductor
L - the length of the conductor
A - the area of the conductor
V - the volume of the conductor

3.4 Conductance

The ability of a material to allow the flow of current.


1 1
G = R  = 
where:

G − the conductance of the conductor in Siemens


 − the conductivity of the conductor

Units of Resistivity and Corresponding Units of Length and Area

Resistivity Length Area


 L A
-m m sq.m.
-cm cm sq.cm
-CM/ft ft CM

3.5 The Cross sectional Area in Circular Mils

CM = circular mils

Area = d2 circular mils where d is the diameter expressed in mils


Area = r2 square mils where r is the radius in mils
82

1 mil = 1/1000 in
Conversion Factor

Square mil = Square inch x 0.000001


Square inch = Square mil x 1,000,000
Square mil = Circular mil x 0.7854
Circular mil = Square mil x 1.273
Square millimeter = Circular mil x 0.0005067

3.6 Wire and Cables

Wires are those electrical conductors 8 mm2 (AWG No. 8) and smaller sizes.
Cables on the other hand, are those which are larger than wires.

Wires and cables are either:

A. Stranded
D. Solid

Example 3.1 What is the equivalent size in square millimeter of a cable 250 MCM?

Solution:
MCM stands for thousand circular mils
250 MCM = 250,000 circular mils

Square millimeter = Circular mil x 0.0005067


= 250,000 x 0.0005067
= 126.67 mm2
83

Example 3.2 What is the area in circular mils of a wire with a diameter of 0.125 in.?

Solution:
1 mil
0.125 in x = 125 mils
0.001 in

Area = d2 = (125)2 = 15,625 circular mils

3.7 Resistivity or Specific Resistance


l
If in Eq. R = ,we put
A

l = 1 meter and A = 1 meter2, then R = ρ (Fig. 3.2)

Hence, specific resistance of a material may be defined as the resistance between the
opposite faces of a meter cube of that material.

Resistivity of Some Common Materials at 20C

Material Resistivity ()


-m -CM/ft
Silver 1.629 x 10−8 9.805
Copper (annealed) 1.724 x 10−8 10.371
Gold 2.44 x 10−8 14.676
Aluminum 2.83 x 10−8 17.02
Iron 98 x 10-8 589.4

Example 3.3 What is the resistance of a copper wire having a diameter of 10 mm


and a length of 10 m?

𝐥
Solution: From the equation : 𝐑 = 𝐀

10
R = 1.724 x 10-8 . 
(10 x 10−3 )2
4

R = 2.19 m

Example 3.4 A 50-m conductor has a cross-sectional area of 1.5 mm2 and having a
resistance of 15 . Calculate its conductivity.
Solution:
l
Conductivity is:  = RA
84

50 m
 = 1 m2
(15 Ω)[1.5 mm2 x ]
(1000 mm)2
 = 2.22 MS/m

Example 3.5 A wire measuring 1.5 m3 has a resistance of 0.955  at 20C;


its length is 1.5 m and an area of 1 m2. Calculate the resistance of this wire at 20C if
the length is changed to 150 m and its area to 450 cm2.

Solution: By ratio and proportion:

R2 l 2 A1
=
R1 l 1 A2

From which:

l 2 A1
R 2 = R1 x l 1 A2
(150 m)(1 m2 )
R 2 = 0.955 Ω x 1 m2
(1.5 m)[450 cm2 x ]
(100 cm)2
R 2 = 2.12 m

Example 3.6 A piece of wire has a resistance of 0.5 . The length is doubled and
the area is increased four times. What is its resistance?

Solution:
 l1  l2
R1 = and R 2 =
A1 A2

By ratio and proportion:

R2 l A 2l1 A1
= l2 A1 =
R1 1 2 l1 4A2

R1 0.5
R2 = =
2 2

R 2 = 0.25 

Example 3.7 A 1-km cable consists of 12 identical strands of aluminum each 3 mm


in diameter. What is the resistance of the cable?

Solution:
l
R1 strand = A
1 000
= 2.8 x 10 −8 x 
(3 x 10−3 )2
4
R1 strand = 3.961 
85

3.961 
R1 strand = = 0.33 
12

Example 3.8. The resistivity of a ferric-chromium-aluminum alloy is 51 × 10−8 Ω-


m. A sheet of the material is 15 cm long, 6 cm wide and 0.014 cm thick. Determine
resistance between (a) opposite ends and (b) opposite sides.

15 cm

6 cm 0.014 cm

(a) (b)

Figure 3.3
Solution:
(a) As seen from Fig. 3.3 (a) in this case,

l = 15 cm = 0.15 m
A = 6 × 0.014 = 0.084 cm2 = 0.084 × 10−4 m2

l 51 × 10−8 x 0.15
R = = = 9.1 × 10−3 Ω
A 0.084 ×10−4

(b) As seen from Figure 3.3(b) here

l = 0.014 cm = 14 × 10−5 m
A = 15 × 6 = 90 cm2 = 9 × 10−3 m2

l 51 × 10−8 x 14 × 10−5
R = = = 79.3 × 10−10 Ω
A 9 × 10−3

3.8 Insulation Resistance of High-Voltage Cables


86

r2
cable

Insulator

r1
Cross-section of a Cable

The insulation resistance of a high-voltage cable is


 r2
R = ln
2l r1

where :  = resistivity of the insulating material (-m)


l = length of the cable (m)
r1 = radius of the conductor
r2 = radius of the insulating material

Example 3.9 A cable has a diameter of 1 in. and the conductor has a diameter of 0.8
in. If the insulating material has an average resistivity of about 1.6 x 1014 ohm-cm,
calculate the insulation resistance per mile of cable.

Solution: 1 mile = 160,934.72 cm

1.6 x 1014 ohm−cm 0.5 in


R = ln
2(160,934.72 cm) 0.4 in

R = 3.53 M
87

Problem Set No. 5

RESISTANCE

1. A wire in the American Wire Gauge has conductor radius of 0.1823 inch. What is
its area in circular mils?
A. 0.3323 MCM
B. 33.23 MCM
C. 333 MCM
D. 3323 MCM

2. If the resistance of a given wire is 32 ohms. If its length is 800 meters, what
would be the resistance of the same size wire which is only 250 meters in
length?
A. 10 ohms
B. 11.25 ohms
C. 15 ohms
D. 25 ohms

3. Determine the length of a copper wire ( = 10.37 -CM/ft) whose diameter is


0.30 inch and resistance of 0.5 ohm at 20C.
A. 4,339 ft
B. 5,255 ft
C. 6,125 ft
D. 6,780 ft

4. Determine the resistance of a bus bar made of copper if the length is 10 meters
long and the cross-sectional area is 4 x 4 sq. cm. The resistivity is 1.724 -cm.
A. 2.121 x 10-3 
B. 1.0775 x 10 –4 
C. 4.312 x 10-4 
D. 3.431 x 10-5 

5. A coil of annealed copper wire has 820 turns, the average diameter of which is 9
in. If the diameter of the wire is 32 mils, calculate the total resistance of the coil
at 20C.
A. 16.46 Ω
B. 17.33 Ω
C. 18.57 Ω
D. 19.57 Ω
88

6. A single layer of No. 24 AWG ( d = 0.051 cm.) commercial iron wire is wound over
a ceramic tube whose diameter is 8.255 cm. If the total wire resistance is 41 ohms,
determine the number of turns.
A. 33 turns
B. 35 turns
C. 38 turns
D. 40 turns

7. A copper wire of unknown length has a diameter of 0.635 cm, and a resistance of
0.28 ohm. By several successive passes through drawing dies the diameter is
reduced to 0.127 cm. Assuming that the resistivity of the copper remains
unchanged in the drawing process, calculate the resistance of the reduced-size
wire.
A. 145 Ω
B. 175 Ω
C. 180 Ω
D. 189 Ω

8. A rectangular bus bar has a cross section of ½ by 1 ¾ in. and is 25 ft long. What is
its resistance at 20C if it made of (a) copper? (b) aluminum?
A. 1.3277 x10-4 Ω, 3.82 x10-4 Ω
B. 2.3277 x10-4 Ω, 3.82 x10-4 Ω
C. 2.3277 x10-4 Ω, 3.82 x10-4 Ω
D. 2.5347 x10-4 Ω, 4.74 x 10-4 Ω

9. A certain batch of copper wire was found to have a conductivity of 95.5 per cent
at 20C. What is its resistivity (a) in ohm-circulars per foot, (b) in microhm-
centimeters?
A. 10.26 Ω-CM/ft, 1.805 x 10-7 Ω-m
B. 10.86 Ω-CM/ft, 1.889 x 10-8 Ω-m
C. 10.96 Ω-CM/ft, 1.826 x 10-7 Ω-m
D. 10.86 Ω-CM/ft, 1.805 x 10-8 Ω-m

10. How many strands are there in a 1,500,000 cir-mil cable, if each strand has a
diameter of 0.094 in.?
A. 170
B. 172
C. 175
D. 182
89

3.9 Effect of Temperature on Resistance of Conductors

The effect of rise in temperature is :

(i) to increase the resistance of pure metals. The increase is large and fairly regular
for normal ranges of temperature. The temperature/resistance graph is a straight line
(Fig. 3.6). As would be presently clarified, metals have a positive temperature co-
efficient of resistance.

(ii) to increase the resistance of alloys, though in their case, the increase is relatively
small and irregular. For some high-resistance alloys like Eureka (60% Cu and 40%
Ni) and manganin, the increase in resistance is (or can be made) negligible over a
considerable range of temperature.

(iii) to decrease the resistance of electrolytes, insulators (such as paper, rubber, glass,
mica etc.) and partial conductors such as carbon. Hence, insulators are said to possess
a negative temperature-coefficient of resistance.

R2

R1

T 0 t1 t2

Graph of Relationship between Resistance and Temperature


R1 R
= T+t2
T+t 1 2

R 2 = R1 [ 1 + 1 (t 2 – t1 )]
1
1 = T+t1

where:

R 2 - resistance at temperature t2
R1 - resistance of temperature t1
90

T - inferred absolute zero temperature


1 - temperature coefficient of resistance at temperature t1

Temperature coefficient of resistance is defined as the ohmic change per


degree per ohm at some specified temperature.

Inferred Absolute Zero Temperature and Temperature Coefficient of


resistance of Some Common Materials

Material T (C) Temperature Coefficient


of Resistance at 20C
Aluminum 228 4.03 x 10-3
Annealed Copper 234.5 3.93 x 10-3
Hard-drawn Copper 241.5 3.82 x 10-3
Iron 180 5.00 x 10-3
Silver 243 3.80 x 10-3

Example 3.10 The resistance of a copper wire measures 25 m at 25C. Calculate


its resistance when heated to 75 C.

Solution:

From the equation:

R1 R2
=
T+t1 T+t2

T+t2 234.5 + 75
R 2 = R1 = 25 mΩ
T+t1 234.5 + 25

R2 = 29.82 m

Alternate solution:

We can also solve this problem using the formula,

R 2 = R1 [ 1 + 1 (t 2 – t1 )]

where R2 is the unknown at t2 = 75C

let R1 = 25 mΩ t1 = 25C and 1 = 25


1 1
to find  1 = 25 = 1 = T+t = = 3.85 x 10−3
1 234.5 + 25
91

Then, R 2 = 25 m [ 1 + 3.85 x 10 −3 (75 – 25)]


= 29.82 m

Example 3.11 A conductor has a resistance of 7  at 0C. At 20C, the resistance


has become 7.5 . Calculate the temperature coefficient of the conductor at 20 C.

Solution:
From the equation:

R 2 = R1 [ 1 + 1 (t 2 – t1 )]

= 7 [ 1 + 1 (20 – 0)]

 0 = 0.003571

1
T = = 280C
0.003571
1 1
20 = =
T+20 280 + 20

20 = 0.0033
92

Problem Set No. 6

EFFECT OF TEMPERATURE ON RESISTANCE OF CONDUCTORS

1. The annealed copper field winding of an electric machine has resistance of 46


ohms at a temperature of 22C. What will be its resistance at a temperature of
75 C?
A. 55.5 ohms
B. 65.9 ohms
C. 72.5 ohms
D. 77.6 ohms

2. Calculate the temperature coefficient of resistance of aluminum at 2 C. Using the


value thus obtained, determine the resistance of an aluminum conductor at 62 C
if its resistance at 2 C is 7.5 ohms.
A. 0.005348/°C, 9.42 ohms
B. 0.004348/°C, 9.46 ohms
C. 0.002348/°C, 9.54 ohms
D. 0.001348/°C, 9.76 ohms

3. The resistance of a given electric device is 46 ohms at 25 C. If the temperature


coefficient of resistance of the material is 0.00454 at 20 C, determine the
temperature of the device when its resistance is 92 ohms.
A. 225.26 °C
B. 235.56 °C
C. 250.26 °C
D. 267.26 °C

4. A certain annealed copper winding has a resistance of 0.25 ohm at a


temperature of 18 C. Calculate the temperature rise in the winding when, after
a period of operation, the resistance increases to 0.31 ohm.
A. 60.6 °C
B. 62.8 °C
C. 65.9 °C
D. 68.33 °C

5. The resistivity of a annealed copper rod 50 ft long and 0.25 in., in diameter is
1.76 microhm-cm at 20 C. What is its resistance at – 20 C?
A. 7.138 x 10-6 ohm
B. 7.138 x 10-5 ohm
C. 7.138 x 10-4 ohm
D. 7.138 x 10-3 ohm
93

3.10 Color Coding of Resistors

Resistor Color Bands

Resistor Color Bands and Their Corresponding Value

Color Number Multiplier Tolerance


Band 1, Band Band 3 Band 4
2 %
Black 0 100 -
Brown 1 101 1
Red 2 102 2
Orange 3 103 -
Yellow 4 104 -
Green 5 105 0.5
Blue 6 106 0.25
Violet 7 107 0.1
Gray 8 108 0.05
White 9 109 -
Gold - 10-1 5
Silver - 10-2 10
None - 20

Example 3.12 A resistor is color coded as red-red-orange-silver. What is the value


of the resistor?

Solution: Red – 2 Red – 2 Orange – 000 Silver -  10%

The value of the resistor is:


R = 22,000   10%
94

3.11 Power Rating of Resistors

The power rating is the maximum amount of power that a resistor can
dissipate without being damaged by excessive heat build-up. The power rating is not
related to the ohmic value (resistance) but rather is determined mainly by the
physical size and shape of the resistor. The larger the surface are of a resistor, the
more power it dissipate. Carbon-composition resistors have power ratings of 1/8 W,
¼ W, ½ W, 1 W, and 2 W.
95

Assessment No. 6

Name: Ybañez, Eric L. Score: _________ Rating: ______

How Much Have You Learned?

Directions: Solve the crossword puzzle. Use the given clues to arrive at the right
answer.
1 2 3 4

2 5

6
3

7
8 9

10 5
6

11
12 7 13

8 14
9

10

DOWN ACROSS
1 a factor affecting resistance 1 causes heat in a conductor
2 composed of many strips of wire 2 universal unit of cross-sectional area cable
3 a single wire 3 a variable resistor used to vary current
4 ______ -composition resistor 4 _______ resistor used in power circuits
5 has negative temperature coefficient 5 zero resistance at 228°C
6 a variable resistor used to vary voltage 6 carbon, wire wound resistors are _____ resistors
7 smaller than 8 mm2 7 _____-able resistors
8 ability to allow the flow of current 8 component used to limit current
9 reciprocal of resistivity 9 with a unit of Siemens
10 equal to 0.001 in. 10 °C, °F,°K
11 larger than a wire
12 tolerance of 5%
13 zero resistance at 180°C
14 resistance of conductors ________ as the temperature increases
96

QUESTIONS: (not part of Assessment No. 6)

1. What is resistance?
2. How does a material restricts current?
3. What are the factors that affect the resistance of conductors?
4. What are the various types of fixed and variable resistors?
5. What is conductance?
97

Practical Application No. 2

Name: Prado, John Bryan Neil Y. Score: _________ Rating: ______

1. You are working on an electronic circuit. The current is 5 mA. A resistor is


marked with the following bands: brown, black, red, gold. A voltmeter measures
a voltage drop of 6.5 V across the resistor. Is this resistor within its tolerance
rating?

2. A homeowner uses a 100-watt incandescent lamps as a heater in an outside well


pump house to protect the pump from freezing in cold weather. Unfortunately,
however, the lamp can burn out and leave pump unprotected. You have been
asked to install a hearth that will not burn out and leave the pump unprotected.
You have available a 100-watt, 150-ohm wire-wound resistor. Can this resistor
be connected to the 120-volt source without damage to the resistor? If so, what
would be the power output of the resistor?

3. You have determined that a 4700-ohm, ½ -watt resistor on an electronic circuit


board is defective. Assuming room permits, can the resistor be replaced with a
4700-ohm, 1-watt resistor without damage to the rest of the board, or will the
higher wattage resistor generate excessive heat that could damage other
components?
98

Objective Test No. 5

RESISTANCE

1. The resistance of a conductor when its temperature is increased,


A. varies.
B. increases.
C. decreases.
D. becomes zero.

2. A 400 MCM cable has 37 strands. What is the diameter of each strand in mils?
A. 10.81
B. 104
C. 108
D. 1,081

3. What is the resistance of a component having no continuity?


A. low resistance
B. infinite resistance
C. no or zero resistance

4. The insulation resistance of the winding of an electric motor is measured by


A. ammeter
B. voltmeter
C. megohmeter
D. galvanometer

5. The resistance of 120 meters of wire is 12 . What is its conductance?


A. 0.0521 Siemens
B. 0.083 Siemens
C. 6 Siemens
D. 12 Siemens

6. The resistance of a material is inversely proportional to its


A. length
B. temperature
C. cross-sectional
D. specific resistance

7. The ability of a conductor to allow current flow.


A. resistance
B. conductance
C. permeability
D. coefficient of resistance
99

8. In making a resistance test, remember that the resistance of a short circuit is


A. infinite
B. approximately zero
C. slightly above the midrange
D. midway between and low range

9. Temperature coefficient of a conductor is defined as the


A. increase in resistance
B. increase in resistance per degree absolute
C. increase in resistance per degree centigrade
D. increase in resistance per ohm per degree centigrade

10. Which of the following statements is TRUE?


A. The diameter of conductor does not affect the resistance
B. The larger the diameter of a conductor, the higher the resistance
C. The smaller the diameter of a conductor, the lesser the resistance
D. The smaller the diameter of a conductor, the higher the resistance

11. An open resistor reads _____ ohms in an ohmmeter


A. zero
B. infinite
C. megohm
D. megohm

12. International ohm is defined in terms of resistance of


A. a cube of copper
B. a cube of carbon
C. a column of mercury
D. a unit length of metal wire

13. Ampere per volt is the same as


A. erg
B. watt
C. siemens
D. Maxwell

14. The former unit of conductance


A. ohm
B. mho
C. henry
D. gauss

15. Resistance commonly used in power circuits.


A. carbon composition
B. wire-wound resistors
C. etched circuit resistors
100

D. deposited film resistors

16. Reciprocal of resistance


A. reluctance
B. admittance
C. susceptance
D. conductance

17. The resistance of a conductor varies ______ when the volume is fixed.
A. directly as the cross sectional area
B. inversely as the cross sectional area
C. directly as the cross sectional area
D. inversely as the square of the cross sectional area

18. A resistor that has an infinite resistance is a sign of ____ resistor.


A. an open
B. a shorted
C. a grounded
D. a burned

19. The resistance of an electrical conductor is inversely proportional to _______.


A. its diameter or cross-sectional area
B. length
C. resistivity
D. square of the volume

20. The resistance of a conductor varies _____ when the volume is fixed.
A. directly as the length
B. directly as the length
C. directly as the square of the length
D. inversely as the square of the length

21. A ground should have a ______ resistance.


A. low
B. high
C. infinite
D. negative

22. In resistance color coding, red color is assigned to a value of


A. 0
B. 1
C. 2
D. 3

23. Which of the following is NOT a common use of resistors in electronic circuits?
A. limit current
101

B. supply power
C. generate heat
D. introduce a voltage drop

24. The physical property of a resistor that determines the ability to dissipate heat is
rated in
A. ohms
B. volts
C. watts
D. amperes

25. In order to show that a resistor has a tolerance of 10%,


A. the fourth band is gold
B. the third band must be silver
C. no color in the fourth band
D. the fourth band must be silver

26. A resistance wire wrapped around an insulating core is called _________.


A. film-type
B. wire wound
C. fusible type
D. carbon composition

27. Which resistor is physically larger in size?


A. 10 ohms, 50 W
B. 100 ohms, 10 W
C. kilohm, 1 W
D. 1 megohm, ½ W

28. The power rating of resistors is determined through their _____.


A. physical size
B. color bands
C. applied voltage

29. A resistor with a color-coded value of 100 ohms and 10% tolerance can have
an expected measured resistance between ______.
A. 90 to 110 ohms
B. 99 to 101 ohms
C. 100 to 110 ohms
D. 110 to 121 ohms

30. Which of the following is a typical resistance and power rating of a carbon
composition?
A. 100 , 5 W
B. 1,000 , 10 W
102

C. 4,700 , 1 W
D. 6,800 , 100 W

31. What is the ohmic value of a resistor having the color bands: brown, green, red,
and gold?
A. 15 
B. 150 
C. 1.5 k
D. 15 k

32. A term universally employed to measure wire diameters.


A. mil
B. meter
C. millimeter
D. circular mil
103

Assessment No. 7

RESISTANCE

Name: Ybañez, Eric L. Score: _________ Rating: ______

Answer the questions in your own words :

1. What is resistance?

2. Discuss how resistance restricts current.

3. What are the factors that affect the resistance of conductors?

4. What is the resistance?

5. What is the resistance of a copper wire having a length of 600 m and diameter of
5 mm?

6. Determine the value of the following resistors:


a. Red, orange, blue, silver
b. Green, red, white, gold

7. What is conductance?

8. Determine the area in circular mils of a wire/cable having the following


diameters:
A. 0.528 in

B. 12 mm

9. An incandescent lamp has a tungsten filament whose resistance is 96 ohms at its


operating temperature of 2900 C. Calculate the filament resistance when the
lamp is disconnected from the electric source, under which condition its
temperature is 24 C.
104

Unit 4

THE ELECTRIC CIRCUIT

E.
LEARNING OUTCOMES
F.
After completing this
unit, you are expected to:

1. describe a basic electric circuit.


2. discuss the function of a fuse or circuit breaker.
3. discuss the causes of overcurrent.
4. determine the rating of fuse or circuit breaker that fits to
a given circuit.
5. identify closed, open, and short circuit .
6. measure voltage, current, and resistance using both
analog and digital multimeters.
7. apply Ohm’s law to determine voltage, current, and
resistance in electric circuits.
8. apply Kirchhoff’s Current Law (KCL) and Kirchhoff’s
Voltage Law (KVL) in a given circuit.
105

Important Terms

electric circuit Closed circuit


energy source Open circuit
Conductors Digital multimeter
Insulators Analog multimeter
load Ohm’s law
Control device overcurrent
Protection
device Short circuit
Fuse overloading
Circuit breaker SPST
SPDT

4.1 The Electric Circuit

Electric Circuit is an interconnection of electrical components in which


there is at least one path for current flow.

Many complete electric circuits contain six parts:

1. An energy source to provide the voltage needed to force current (electrons)


through the circuit.
2. Conductors through which the current can travel.
3. Insulators to confine the current to the desired paths (conductors, resistors,
etc.)
4. A load to control the amount of current and convert the electric energy taken
from the energy source.
5. A control device, often a switch, to start and stop the flow of current.
6. A protection device to interrupt the circuit in case of a circuit malfunction.

The first four of the above six parts are essential parts. All complete circuits use
them. The control device (item 5) is occasionally omitted. Protection devices (item
6) are often omitted from circuits. A complete electric circuit has an uninterrupted
path for current (electrons) to flow from the negative terminal of the energy source
106

through the load and control device to the positive terminal of the energy source.
An example of a circuit is shown in Figure 4.1 and Figure 4.2.

Figure 4.1. A circuit consisting of a battery (voltage source), a switch, a buzzer


(load), and wires used to interconnect the components.

4.2 Protection Devices

Circuit Breakers Fuses

A fuse is inserted into a circuit to protect the device / circuit from receiving too
much current when shorted.

A circuit breaker is the same function. If a surge of current is evident through a line,
the circuit breaker "breaks" the line, opening the flow of current.

A fuse breaks the circuit only once, then, has to be replaced.


107

A circuit breaker is a "switch" that closes and opens the line. A circuit breaker is
usually STRONGER (tolerates more current flow than a fuse.)

4.2.1 Causes of Overcurrent

a. Short Circuit
b. Overloading

Example 4.1 You are an electrician on the job. The electrical blueprint shows that
eight 500-W lamps are to be installed on the same circuit. The circuit voltage is 230
V and is protected by a 20-A circuit breaker. Is a 20-A circuit large enough to carry
this load?

Solution:

IT

20 A

1 Vpk
2X6U2
12
X3X2V
230 V X4 412V1
121kHz
3V 12V
12
1VV
X1
X5
X8 U1
12 V
X7

20 A

total power
Total current (IT ) =
total voltage

8 x 500 W
Total current (IT ) = = 17.39 A
230 V

Since the total current in the circuit is 17.39 A, therefore, a 20-A circuit breaker or
fuse is large enough to carry the load.

Example 4. 2 If the load in example 4.1 is a continuous load, is the 20-A circuit
breaker can carry the load. Note that a continuous-use circuit can be loaded only
80% of its rating.

Solution:
The 20-A circuit breaker can carry only 16 A (80% of 20 A) since the
load is continuous. Since the current in the circuit is 17.39 A, the circuit breaker
cannot carry this load.
108

4.3 Circuit Components and Symbols

Device Symbol

Lamp (light bulb)


X1
12 V

12V1
V
Cell or battery

Fuse
U1
0.5_AMP

Circuit Breaker

Single-Pole-Single KeyOpen
=J1
Space KeyClosed
=J2
Space
Throw (SPST)
Switch
109

Single-Pole-Double
Throw (SPDT)
Switch

Conductor

R
Resistor 1kΩ

L
Inductor (coil) 1mH

C
Capacitor
1µF
110

switch

battery
Key J1
=V
V1 Space
bATTERY
16
12 X1
V

(b) (b)

Figure 4.2. (a) A simple electric circuit. (b) Schematic diagram for the circuit in (a)

4.4 Closed, Open and Short Circuit

Closed Circuit

A closed circuit is a circuit in which the current has a complete path.

Key J1
=V
V1 Space
battery
bATTERY
switch
16
12 X1
V

Open Circuit
When the current path is broken so that current cannot flow, the circuit is
called an open circuit.

Key J1
=V
V1 Space
battery
bATTERY
switch
16
12 X1
V

Short Circuit
111

A short circuit is a zero or abnormally low resistance between two points. It


is usually an inadvertent condition.

Key = Space KeyJ1


= Space
J1
X1battery
bATTERY
switch
12 V 12 VX1 12 VV1
16V1
V

4.5 Basic Circuit Measurement

Three common electrical quantities are voltage, current, and resistance. These
quantities are measured by voltmeter, ammeter, and ohmmeter. They are usually
put into one instrument called multimeter, multitester, or VOm (voltmeter,
ohmmeter, and milliammeter). Two types of multitester are digital and analog.

+ - + -
00.000 A 00.000 V

XMM1
Ammeter VoltmeterOhmmeter

Meter Symbols
How to Measure Current with an Ammeter
112

E R
60ohm
12V

Circuit in which the current is to be measured

(a) Open the circuit either between the resistor and the negative terminal or
between the resistor and the positive terminal of source.

E R
60ohm
12V

(b) Install the ammeter with polarity as shown (negative to negative – positive to
positive)
+ -
0.200 A

E R
60ohm
12V

How to Measure Voltage with a Voltmeter

To measure voltage, connect the voltmeter across the component for which the
voltage is to be found. Such a connection is a parallel connection. The negative
terminal of the meter must be connected to the negative side of the circuit and the
positive terminal of the meter to the positive side of the circuit.
113

R + -
E
12.000 V
60ohm
12V

How to Measure Resistance with an Ohmmeter

To measure resistance, connect the ohmmeter across the resistor. The resistor must
be removed or disconnected from the circuit.

XMM1

E R
60ohm
R
12V 60ohm

(a) Disconnect the resistor from the (b) Measure the resistance.
circuit to avoid damage to the (polarity is not important.)
meter and/or incorrect
measurement.
114

4.6 The Analog Multitester

How to read the measurements in an analog multitester.

1. Select what quantity is to be measured by using the rotary switch.

Reminder: The rotary selector switch below selects what quantity is to be


measured. If you are measuring a voltage of 8 V it must be pointed to the 10-V
range. THE RANGE SHOULD BE HIGHER THAN THE EXPECTED VALUE OF
VOLTAGE, CURRENT, OR RESISTANCE.
115

2. Connect the meter as discussed in 4.4


3. The measured value of the electrical quantity can be read from the needle.
Using the scales below (Figure 4.3) , if you are measuring DC voltage and
selector switch placed at 50-V range, the DC voltage will read 22 V.

Figure 4.3

Example 4.3 Using Figure 4.3, find the correct reading if the selector is placed at
the following:

DC 10V range
DC 50V range
DC 25mA range
AC 10V range

Solution:

DC 10V range: 4.4V (read 0-10 scale directly)


DC 50V range: 22V (read 0-50 scale directly)
DC 25mA range: 11mA (read 0-250 and divide by 10)
AC 10V range: 4.45V (use the red scale, reading 0-10)
116

Objective Test No. 6

ELECTRIC CIRCUIT MEASUREMENT

1. An instrument that measures the voltage or electrical pressure in a circuit.


A. megger
B. ammeter
C. voltmeter
D. galvanometer

2. A multimeter consists of a
A. ammeter and ohmmeter
B. voltmeter and ammeter
C. voltmeter and ohmmeter
D. voltmeter, ammeter, and ohmmeter

3. How is a voltmeter connected in a circuit?


A. Connect in series across the load
B. Connect in shunt across the load
C. Connect in open circuit with the load
D. Connect in short circuit across the load

4. Which of the following is an integrating instrument?


A. ammeter
B. ohmmeter
C. voltmeter
D. wattmeter

5. An ammeter is connected ________.


A. across the load
B. in series with the load
C. in series-parallel across the load

6. Electrical measurement used to measure electrical power


A. wattmeter
B. galvanometer
C. clamp ammeter
D. kilowatt-hour meter

7. Instrument used to measure electrical energy.


A. wattmeter
B. galvanometer
C. clamp ammeter
D. kilowatt-hour meter
117

4.7 Ohm’s Law

For a simple DC circuit, German physicist George Simon Ohm has observed
another relationship related to the resistance of an object. He noted that for a fixed
load in a circuit at a constant temperature, when voltage is increased the current
reading across the load also is increased. Consequently, as the voltage is lowered
across the load, current reading is also lowered.
Therefore, he arrived at the conclusion that current is directly proportional to
voltage. And when the ratio between voltage and current was computed, it has been
found out that it is approximately equal to the resistance of the load. Thus, Ohm’s
Law states that: “The ratio between voltage and current is always constant and is
equal to the resistance of the load. (At a constant temperature!)

E R
60ohm
12V

V
R = I

where:
I = current in amperes
V = voltage in volts
R = resistance in ohms, 

Other Ohm’s Law Formulas


V
V = IR I = R

The Conductance
Conductance is the reciprocal of resistance. It is the property of a conductor
or resistor to allow current flow.

Symbol: G

Unit: Siemens (S)

The formulas are


I I
G = V
I = GV V = G
118

Limitation of Ohm’s Law

Ohm’s law does not apply under the following condition:

1. Electrolytes where enormous gases are produced on either electrode.


2. Non-linear resistors like vacuum radio valve, semi-conductors, gas-filled
tubes, etc.
3. Arc lamps
4. Metals which get heated up due to flow of current through.
5. Appliances like metal rectifiers, crystal detectors, etc. in which operation
depends on the direction of current.

Example 4.4 For the circuit shown below, find the current through the 60-
resistor.

E R
60ohm
12V

Solution:
V 12 V
I = = = 0.2 A
R 60 

Example 4.5 The difference of potential between the terminals of an electric heater
is 110 V when there is a current of 8 A in the heater. What current will be maintained
on the heater if the difference of potential is increased to 180 V?

Solution:
V1 110 V
R = = = 13.75 
I1 8A

V2 180 V
I2 = = = 13.09 A
R 13.75 

Example 4.6 A 6-V battery is connected for 3 hours to a rheostat and a current of
147 mA is noted. (a) What is the resistance of the rheostat? (b) What charge is
take form the battery?

Solution:
V
(a) R = = 40.8 
I

3600 sec
(b) Q = It = (147 mC/s)(3 hours x )= 1.59 x 103
1 hr
119

Assessment No. 8

OHM’S LAW

Name: Ybañez, Eric L. Score: _________ Rating: ______

1. Find the unknown in circuit below.

I = _______ A
+ -
1.000 A

V R +
12.000 V
12ohm -
12V

2. Show the placement of an ammeter and voltmeter to measure the current and
voltage across s R1 and R2.

R1
12ohm

12V

R2
12ohm
120

3. The current in a 5- resistor increases linearly from zero to 10 A in 2 ms. At t = 2


ms the current is again zero, and it increases linearly to 10 A at t = 4 ms. This
pattern repeats each 2 ms. Sketch the corresponding v.

4. A certain electrical device has an unknown resistance. You have available a 12-
V battery and an ammeter. How would you determine the value of the
unknown resistance? Draw the necessary connections.
121

Problem Set No. 7

OHM’S LAW

1. For the circuit shown below, find the current through the 60- resistor.

A. 0.002 A
B. 0.02 A E R
60ohm
C. 0.2 A 12V

D. 2A

2. The difference of potential between the terminals of an electric heater is 110 V


when there is a current of 8 A in the heater. What current will be maintained on
the heater if the difference of potential is increased to 180 V?

A. 13.09 A
B. 0.02 A
C. 0.2 A
D. 2A

3. A 6-V battery is connected for 3 hours to a rheostat and a current of 147 mA is


noted. (a) What is the resistance of the rheostat? (b) What charge is taken from
the battery?

A. 40.8 ohms, 1,587.6 C


B. 32.8 ohms, 1,587.6 C
C. 40.8 ohms, 1,556.6 C
D. 34.8 ohms, 1,487.6 C

4. Find the value of the current measured by the ammeter.


R
30Ω

U1
E -
36 V 0.000 A DC 1e-009 W
+

A. 1.2 A
B. 0.02 A
C. 0.2 A
122

D. 2 A

5. Find R in the circuit below.

A. 12 Ω
B. 13 Ω
C. 14 Ω
D. 16 Ω

+ U1
I -64.000 V DC 10M W
R -
4A
16Ω

6. What is the potential difference across a 15 -  resistor when a current of 6.5 A


passes through it?

A. 67.5 V
B. 77.5 V
C. 87.5 V
D. 97.5 V

Objective Test No. 7


OHM’S LAW

1. When using Ohm’s Law, E divided by I would solve for


A. watts
B. voltage
C. amperage
D. resistance

2. The condition of Ohm’s law is that


A. the temperature should vary
B. ratio V/I should be constant
C. current should be proportional to voltage
D. the temperature should remain constant
123

3. If the resistance of the circuit is doubled while the applied voltage is held
constant. The current will ____________________.
A. be twice as much
B. remain the same
C. increase by half as much
D. decrease to half as much

4. A circuit has a resistance of 8 ohms. If a voltmeter connected across its


terminals reads 10 V, how much current is flowing through the circuit?
A. 0.80 A
B. 1.25 A
C. 1.5 A
D. 2.10 A

5. If the potential across a circuit is 40 V and the current is 5,000 mA, what is the
equivalent resistance of the circuit?
A. 8 
B. 80 
C. 800 
D. 800 k

6. The difference of potential between the terminals of an electric heater is 120 V


when they are at a current of 8 A. What current will be maintained in the
heater if the potential difference is increased to 220 V?
A. 4.4 A
B. 12 A
C. 14.7 A
D. 40 A

7. When using Ohm’s law “IR” would solve for ___________.


A. voltage
B. amperage
C. resistance
D. electrical power

8. A water heater takes 2.5 A at 230 V. What is its hot resistance?


A. 74 ohms
B. 82 ohms
C. 92 ohms
D. 100 ohms
124

4.8 Kirchhoff’s Law


.
.
Important Terminologies

Node – a point in which two or more components have a common connection.

Path- if no node was encountered more than once, then the sets of nodes and
elements that we have passed through is defined as a path.

Closed Path or Loop- If the node at which we started is the same as the node on
which we ended, then the path, is by definition, a closed path or loop.

Branch - is a single path in a network, composed of one simple element and the
node at each end of that element.

R3
1kohm
R1 R2
I 1kohm 1kohm
1A
2
R4
1kohm

(a)

R3
1kohm
R1 R2
I 1kohm 1kohm
1A
2
R4
1kohm

(b)

(a) A circuit containing three nodes and five branches. (b) Node 1 is redrawn to
look like two nodes; it is still one node.
125

4.8.1 Kirchhoff’s Voltage Law (KVL)


The algebraic sum of all voltages in a circuit taken around a closed path
is zero.
Sign Convention a

V
12V

Path b – a : Potential Rise = + V


Path a – b: Potential Drop = -V

+ a

R
1ohm VR

I
_ b

Path b – a: Potential Rise = +VR


Path a – b: Potential Drop = - VR

4.8.2 Kirchhoff’s Current Law (KCL)

The algebraic sum of all currents entering and leaving a node is zero.

Convention: I2

I3

I1
I4

Current entering a node: + sign


Current leaving a node: -sign

I1 – I2 – I3 + I4 = 0
126

Example 4.7 Find V1


- +
14.000 V

V2
a b a
R
2V 30ohm
V1 V3
12V 24V

V4

e d
4V
Solution:

V1 + V2 + VR – V3 – V4 = 0

V1 = − V2 − VR + V3 + V4

V1 = − 2 − 14 + 24 + 4

V1 = 12 V

Example 4.8 Find V2

Solution:
V1 + V2 + VR − V4 = 0

V2 = − V1 − VR + V4

V2 = − 12 − (−10) + 4
V2 = 2 V
127

Example 4.9 Find the current in the circuit.

2VA

+ V30 - 𝑖
-
VA

Solution:

Let the current i be in the clockwise direction.


By applying KVL in a clockwise direction.

120 − V30 − 2 VA + VA = 0
By Ohm’s law
V30 = 30i
VA = −15i

so that 120 − 30i − 2(−15i) − 15i = 0


i=8A

Example 4.10 Find I3

R1
a

15ohm
R2
I1 36ohm I2

3A 5A
I3

Solution: At node a

I1 + I2 − I 3 = 0

I3 = I1 + I2
128

I3 = 3 + 5
I3 = 8 A

Example 4.11 Find i3 and i6.

i3 i6

0.9i3

Solution: By applying KCL at either node.

0.9i3 + 2 − i3 − i6 = 0

v6 3i3 1
where i6 = = = 2 i3 . Substituting to the equation, then
6 6

1
0.9i3 + 2 − i3 − i =0
2 3

i3 = 3.33 A

Example 4.12 Calculate the current flowing in each branch of the circuit shown

R2
V1 V3
20ohm
3V 6V

R1 R3
V2
10ohm 20ohm
4.5V

Solution:

Step 1: Assign branch current direction on a node and form KCL equation. (NOTE:
Do not be over-conscious about the direction. If you wrongly assigned it, it will just
yield negative answer but the magnitude will be the same.)
129

R2
V1 V3
20ohm
3V 6V

I2
R1 R3
V2
10ohm 20ohm
I1 4.5V I3

KCL equation:

I1 + I2 + I3 = 0

Step 2: Assign loop-current direction on each visible loop and form your KVL
equations. (Again, direction is arbitrary)

KVL equation:

For loop 1 (L1):


R2
V1 V3
20ohm
10I1 + 3 − 20I2 + 4.5 = 03V 6V

10I1 − 20I2 = − 7.5 L1 I2


R1 L2 R3
V2
For loop 2 (L2): 10ohm 20ohm
I1 4.5V I3
4.5 + 6 − 20I2 + 30I3 = 0
−20I2 + 30I3 = −10.5

Step 3: Solve for the magnitude of the branch currents (I1, I2 and I3) using any
method.

I1 + I2 + I3 = 0 equation 1
10I1 − 20I2 = − 7.5 equation 2
−20I2 + 30I3 = −10.5 equation 3

The answers are: I1 = −0.15 A

I2 = 0.3 A
130

I3 = 0.15 A
.
Example 4.13 Solve for the currents in all the resistors of the circuit shown below.

R1
50ohm

E1 R3

12V
100ohm
E2
R2 3V
200ohm

Solution:

Step 1: Assign branch-current direction and form KCL equation.

R1
50ohm

I1
E1 R3

12V I3 1kohm
E2
R2 3V
50ohm
I2

KCL equation:

−I1 + I2 − I3 = 0

Step 2: Assign Loop-Current Direction and form KVL equations.


R1
50ohm

Loop 1: I1
E1 R3

12 + 50I1 + 200I2 = 0
12V I3 100ohm
E2
L1 R2 3V
50I1 + 200I2 = −12 200ohm
I2
L2
131

Loop 2:
−3 – 200I2 − 100I3 = 0
−200I2 − 100I3 = 3

Step 3: Solve by matrix and determinants

−I1 + I2 − I3 = 0 equation 1

50I1 + 200I2 = −12 equation 2

−200I2 − 100I3 = 3 equation 3

I1 = 0.086 A; I2 = 0.0385 A; I3 = −0.0475 A


132

Assessment No. 9

KIRCHHOFF’S LAW

Name: Ybañez, Eric L. Score: _________ Rating: ______

1. Find the current through each resistor using Kirchhoff’s law.

R1
50ohm

E1 R3

12V
100ohm
E2
R2 3V
200ohm
133

2. Find the values of I1 and V5.

R1

1ohm

R2
5ohm I2
I1
I1
6A

+
R3 10A
2ohm V5
4A -
R4 1kohm

3ohm

.
134

Problem Set No. 8

KIRCHHOFF’S LAW

1. What is the value of V in the circuit shown below?

6A

+ 4V -
+ + + -
-4A 2V 2A 2V 4A -10 A
2V -2 V
- +
- -
-6 A
i -4 A

+ V -
A. 4V
B. 6V
C. 8V
D. 10 V

2. What is the value of the power received by element B?

-7 A

+ 4V -
- - -
+
B 7A
-2 A -2 V -2 V -2 A v
2V i
+ + +

- 7A

- -9 V +
A. -49 W
B. 49 W
C. 94 W
D. -94 W
135

3. Find I7.

I2 -2 A

+ -1 V - - 3V +
- -
-1 A I7 I6
2V v6
+ +
I3 I5

+ v3 - - -3 V +
- - +

A. 0.5 A
B. 0.75 A
C. -1 A
D. 1.25 A

4. Find vx.

+
Vx

-
A. 2.4 V
B. 4.2 V 4Vx
C. 4.8 V
D. 5.2 V

5. Find iA , iB and iC.


136

iC

A. 0.25 A, 1.35 A, 1.5 A


B. 0.75 A, 1.35 A, 1.5 A
C. 0.25 A, 1.35 A, 1.75 A
D. 0.25 A, 1.85 A, 1.5 A

6. What is the value of V7?

I2
+ V7 -

+ 12 V -
+ + - +
I3 2A I6
V1 3A 3V 4V V6
- -
+ -
I4

+ 6V -

A. 11 V
B. 12 V
C. 13 V
D. 14 V
137

- +
7. Find V1. 14.000 V

V2
a b a
R
A. 22 V 2V 30ohm
V1 V3
B. 23 V 12V 24V
C. 24 V V4
D. 25 V
e d
4V

8. Find V2.

A. 1V
B. 1.5 V
C. 1.75 V
D. 2V

9. Find I3.
R1
a

15ohm
A. 8A R2
I1 36ohm I2
B. 9A
3A 5A
C. 10 A I3
D. 12 A
b

10. Calculate the current flowing in each branch of the circuit shown.

R2
V1 V3
20ohm
3V 6V

R1 R3
V2
10ohm 20ohm
4.5V

A. I1 = 112.27 mA, I2 = 317.96 mA, I3 = 205.68 mA


B. I1 = 112.27 mA, I2 = 217.96 mA, I3 = 205.68 mA
C. I1 = 12.27 mA, I2 = 317.96 mA, I3 = 205.68 mA
138

D. I1 = 112.27 mA, I2 = 317.96 mA, I3 = 105.68 mA

11. Solve for the currents in all the resistors of the circuit shown below.

R1
50ohm

E1 R3

12V
100ohm
E2
R2 3V
200ohm

A. I1 = 55.62 mA, I2 = 38.54 mA, I3 = 47.07 mA


B. I1 = 85.62 mA, I2 = 18.54 mA, I3 = 47.07 mA
C. I1 = 85.62 mA, I2 = 38.54 mA, I3 = 47.07 mA
D. I1 = 85.62 mA, I2 = 38.54 mA, I3 = 37.07 mA

12. Determine I1, I2, and V2.

R3
6A

I1 5ohm
R1
2 R2 V2
V1 1kohm 6ohm 12V
12V
I2

A. I1 = 1A, I2 = 2 A, V2 = 42 V
B. I1 = 2 A, I2 = 2 A, V2 = 62 V
C. I1 = 2 A, I2 = 2 A, V2 = 42 V
D. I1 = 2 A, I2 = 3A, V2 = 42 V
139

13. Determine I1 and V5.


R1

1ohm

R2
5ohm I2
I1
I1
6A

+
R3 10A
2ohm V5
4A -
R4 1kohm

3ohm

A. I1 = -9 A, V5 = 80 V
B. I1 = -8 A, V5 = 80 V
C. I1 = -8 A, V5 = 90 V
D. I1 = -8 A, V5 = 100 V

14. Find the branch currents in the circuit below.

R1

9.1ohm
10 ohms
V1

16V V3
R3

8ohm
4V

V2

10V
R2

2ohm

A. I1 = 1.09 A, I2 = 159.95 mA, I3 = 2.25 A 2 A


B. I1 = 2.09 A, I2 = 157.95 mA, I3 = 2.25 A 2 A
C. I1 = 2.09 A, I2 = 157.95 mA, I3 = 2.35 A 2 A
D. I1 = 1.09 A, I2 = 197.95 mA, I3 = 2.25 A 2 A
140

Assessment No. 10

Name: Ybañez, Eric L. Score: _________ Rating: ______

How Much Have You Learned?

Directions: Solve the crossword puzzle. Use the given clues to arrive at the right
answer.
2
1 1 4
3
2

10
8
6
5
7 3

7
5
8 10

DOWN ACROSS
1 current cannot flow 1 current can flow
2 melts in over current 2 short circuit and overloading
3 protection against current 3 equivalent to volt/ampere
4 interconnection of components 4 part of a circuit that consumes energy
5 used to convey current 5 control device
6 V = IR 6 trips
7 causes overcurrent 7 ____________ multimeter
8 energy source 8 equivalent to ampere/volt
9 single-pole, single-throw 10 equivalent to joule/coulomb
10 connected in series to the load
141

QUESTIONS: (not part of Assessment No. 10)

1. What is an electric circuit?


2. What are the parts of an electric circuit?
3. What is the function of a fuse or circuit breaker?
4. What are the causes of overcurrent?
5. What is the effect of overcurrent?
6. How do you avoid overcurrent?
7. What is a closed circuit? open circuit?
8. How do you connect the voltmeter in measuring voltage?
9. How do you connect the ammeter in measuring current?
10. What is Ohm’s law?

.
142

Practical Application No. 3

Name: Ybañez, Eric L. Score: _________ Rating: ______

1. You are an electrician on the job. The electrical blueprint shows that eight 500-W
lamps are to be installed on the same circuit. The circuit voltage is 277 V and is
protected by a 20-A circuit breaker. Assuming that the load is continuous, is a 20-
A circuit large enough to carry this load?

2. You have been sent to a new home. The homeowner reports that sometimes the
electric furnace trips the 240-V, 60-A circuit breaker connected to it. Upon
examination, you find that the furnace contains three 5000-W heating elements
designed to turn on in stages. For example, when the thermostat calls for heat,
the first 5000-W unit turns on. After some period of time, the second unit will
turn on, and then, after another time delay, the third unit will turn on. What do
you think the problem is, and so would your recommendation for correcting?
Explain your answer.
143

3. You are an electrician installing the wiring in a new home. The homeowner
desires that a ceiling fan with light kits be installed in five different rooms. Each
fan contains a light kit that can accommodate four 60-watt lamps. The voltage
source is 220 V. Each fan motor draws a current of 1.8 amperes when operated
on high speed. It is assumed that each fan can operate more than three hours at a
time and therefore must be considered a continuous-duty service. The fans are to
be connected to a 15-ampere circuit. Because the devices are continuous duty, the
circuit current must be limited to 80% of the continuous connected load. How
many fans can be connected to a single 15-ampere circuit? How many circuits will
be required to supply power to all five fans?

4. A homeowner is installing a swimming pool. You have been asked to install a


circuit to operate a 600-watt underwater light and a circuiting pump. The
motor nameplate reveals that the pump has a current draw of 8.5 amperes.
The devices are considered continuous duty. Can the power to operate both
of these devices be supplied by a single 20-ampere circuit? The voltage source
is 220 V.

.
144

Unit 5

WORK. POWER, AND ENERGY

LEARNING OUTCOMES

After completing this


unit,G.you are expected to:
H.

1. discuss the effect of electricity.


2. explain the difference between power and energy.
3. Discuss the different types of energy conversion.
4. discuss efficiency.
5. determine the amount of power in a resistive circuit.
6. explain how energy is measured in kilowatthours.
7. Compute energy consumption and its cost.
8. Solve the terminal voltage of a battery or any other voltage
source.
9. Analyze electric power transmission and distribution lines.
145

Important Terms

electric current Joule’s law


thermal efficiency energy
Power kilowatthour
Efficiency Terminal potential difference
Power loss

5.1. Effect of Electric Current

It is a matter of common experience that a conductor, when carrying current,


becomes hot after some time. As explained earlier, an electric current is just a
directed flow or drift of electrons through a substance. The moving electrons as they
pass through molecules of atoms of that substance, collide with other electrons. This
electronic collision results in the production of heat. This explains why passage of
current is always accompanied by generation of heat.

5.2. Joule’s Law of Electric Heating

The amount of work required to maintain a current of I amperes through a


resistance of R ohm for t second is

W.D. = I2 Rt joules
= VIt joules
= Pt joules
= V2t/R joules

This work is converted into heat and is dissipated away. The amount of heat
produced is

work done W. D.
H= =
mechanical equivalent of heat J

where J = 4,186 joules/kcal = 4,200 joules / kcal (approx)


∴ H = I2Rt/4,200 kcal = Vlt/4,200 kcal
= Pt/4,200 kcal = V2t/4,200 R kcal
146

5.3. Thermal Efficiency

It is defined as the ratio of the heat actually utilized to the total heat produced
electrically. Consider the case of the electric kettle used for boiling water. Out of the
total heat produced (i) some goes to heat the apparatus itself i.e. kettle (ii) some is
lost by radiation and convection etc.
and (iii) the rest is utilized for heating the water. Out of these, the heat utilized for
useful purpose is that in (iii). Hence, thermal efficiency of this electric apparatus is
the ratio of the heat utilized for heating the water to the total heat produced. Hence,
the relation between heat produced electrically and heat absorbed usefully becomes

VIt
x  = ms (θ2 − θ1 )
J

Example 5.1. The heater element of an electric kettle has a constant resistance of
100 Ω and the applied voltage is 250 V. Calculate the time taken to raise the
temperature of one litre of water from 15ºC to 90ºC assuming that 85% of the
power input to the kettle is usefully employed. If the water equivalent of the kettle is
100 g, find how long will it take to raise a second litre of water through the same
temperature range immediately after the first.

Solution:

Mass of water = 1000 g = 1 kg ( 1 cm3 weight 1 gram)


Heat taken by water = 1 × (90 − 15) = 75 kcal
Heat taken by the kettle = 0.1 × (90 − 15) = 7.5 kcal
Total heat taken = 75 + 7.5 = 82.5 kcal
Heat produced electrically H = I2Rt/J kcal
Now, I = 250/100 = 2/5 A, J = 4,200 J/kcal; H = 2.52 × 100 × t/4200 kcal
Heat actually utilized for heating one litre of water and kettle
= 0.85 × 2.52 × 100 × t/4,200 kcal

0.85 x 6.25 x 100 x t


∴ = 82.5 , ∴ t = 10 min. 52 second
4,200

In the second case, heat would be required only for heating the water
because kettle would be already hot.

0.85 x 6.25 x 100 x t


∴ 75 = = 82.5 , ∴ t = 9 min 53 second
4,200
147

5.4 Electrical Power and Energy

Energy
Energy is the capacity to do work.
Power
Power is the rate at which energy is expended.

W
P = t

where: P = power in watts (W) (1 watt = 1 joule/sec)


W = energy in joules (J)
t = time in sec (s)

If the energy changes non-linearly with time the power is

dw
p = dt

dw dq
If v = and i = . Multiplying v and i yields
dq dt

dw dq dw
vi = ( )= = p
dq dt dt

Example 5.2 In Example 2.6 calculate and sketch the power absorbed by the BOX
between 0 and 10 milliseconds.

Solution: We have found the following current

𝐢(𝐭) = 𝟎 𝟎  𝐭  𝟏 𝐦𝐬

𝐢(𝐭) = 𝟐 𝐀 𝟏  𝐭  𝟐 𝐦𝐬

𝐢(𝐭) = 𝟎 𝟐  𝐭  𝟑 𝐦𝐬

𝐢(𝐭) = −𝟐. 𝟓 𝐀 𝟑  𝐭  𝟓 𝐦𝐬

𝐢(𝐭) = 𝟎 𝟓  𝐭  𝟔 𝐦𝐬

𝐢(𝐭) = 𝟏. 𝟑𝟑 𝐀 𝟔  𝐭  𝟗 𝐦𝐬

𝐢(𝐭) = 𝟎 𝐭  𝟗 𝐦𝐬
148

The power absorbed by the BOX is 12 · i(t).

𝐩(𝐭) = 𝟏𝟐 · 𝟎 = 𝟎 𝟎  𝐭  𝟏 𝐦𝐬

𝐩(𝐭) = 𝟏𝟐 · 𝟐 = 𝟐𝟒 𝐖 𝟏  𝐭  𝟐 𝐦𝐬

𝐩(𝐭) = 𝟏 · 𝟎 = 𝟎 𝟐  𝐭  𝟑 𝐦𝐬

𝐩(𝐭) = 𝟏𝟐 · −𝟐. 𝟓 𝐀 = −𝟑𝟎 𝐖 𝟑  𝐭  𝟓 𝐦𝐬

𝐩(𝐭) = 𝟏𝟐 · 𝟎 = 𝟎 𝟓  𝐭  𝟔 𝐦𝐬

𝐩(𝐭) = 𝟏𝟐 · 𝟏. 𝟑𝟑 = 𝟏𝟔 𝐖 𝟔  𝐭  𝟗 𝐦𝐬

𝐩(𝐭) = 𝟏𝟐 · 𝟎 = 𝟎 𝐭  𝟗 𝐦𝐬

30

20

10

0
0 1 2 3 4 5 6 7 8 9 10
-10

-20

-30

-40

Example 5.3 The power absorbed by the BOX shown in the figure below is p(t) =
2.5e−4t W . Calculate the energy and charge entered to the BOX in the interval
0  t  250 ms.
149

i(t) C1
E BOX
50 𝑒 −𝑡 𝑉 12 V 1µF

Solution:

dw
From = p ; w = ∫ p (t)dt
dt

250 x 10−3
so that w = ∫0 2.5e−4t dt = 395.1 mJ

dw dw
and also from = v ; q= ∫ ; dw = p(t)dt
dq v

250 x 10−3 p(t)dt 250 x 10−3 2.5e−4t dt


and so q = ∫0 = ∫0 = 8.8 mC
v 50 e−t

When current flows through a resistance, the collision of the electrons give
off heat resulting in a loss of energy. There is always a certain amount of power in
an electrical circuit, and it is dependent on the amount of

E L
12V 12V_25W

Power in an electrical circuit is seen as heat given off by the resistance.

resistance and on the amount of current, expressed as follows:

P = I2 R

This is Joule’s law which states that the heat produced in a current carrying
conductor is proportional to square of the current.

We can produce an equivalent expression for power by substituting V = IR (I2 is I x


I)
150

P = I2 R = (I x I)R = I(IR) = (IR)I


P = VI
We obtain another equivalent expression by substitution V/R for I (Ohm’s law) as
follows:
V
P = VI = V(R)
V2
P = R

Example 5.4 Determine P if R = 30  and I = 3 A

Solution: P = I2 R = (3)2(30) = 270 watts

Example 5.5 Determine P if V = 120 V and I = 3 A

Solution:
P = IV = (3)(120) = 360 W

Example 5.6 A bulb having a rating of 220 V, 75 watts, is to be connected to a 110-V


source. What power will it draw if connected across 110-V source. Find the current
through the lamp and the power it dissipates.
Solution:
+ -
0.170 A

Lam p
E 75W 220V
110V

The resistance of the lamp is:

V2rating (220)2
RL = = = 645.33 
Prating 75

Vsource 110 V
Ilamp = = = 170.46 mA
RL 645.33 

The power dissipated by the lamp is

P = IL 2 R L = (0.17046)2 (645.33) = 18.75 W or

V2s 1102
P = RL
= 645.33
= 18.75 W
151

Power Supplied or Absorbed?

i The element absorbs or receives


a b power if the direction of current
+ v - is from positive to negative using
the conventions direction of
current.
i
In this case the element delivers
a b or supplies power.
- v +

5.5 The kilowatt-hour (kWh) Unit of Energy

Since power is the rate of energy usage, power utilized over a period of time
represents energy consumption. If we multiply power and time, we have energy,
symbolized by W:

Energy = Power x Time

W=Pt

Earlier, joule was defined as a unit of energy. However, there is another way
of expressing energy. Since power is expressed in watts and time in seconds, we can
use units of energy called the wattsecond (Ws), watthour (Wh), and kilowatt-hour
(kWh).
When you pay your electric bill, you are charged on the basis of the amount of
energy you use. Because power companies deal in huge amounts of energy, the more
practical unit is the kilowatt-hour. You use a kilowatt-hour of energy when you use
the equivalent of 1000 W of power for I hr. For example, a 100-W light burning for
10 h uses 1 kWh of energy

Example 5.7 Determine the number of kilowatt-hours for each of the following
energy consumptions:

(a) 1400 W for 1 hr (b) 2500 W for 2 hr (c) 100,000 W for 5 hr

Solution:

(a) 1400 W = 1.4 kWhr


Energy = (1.4 kW)(1 hr) = 1.4 kWh

(b) 2500 W = 2.5 kW


152

Energy = (2.5 kW)(2 hr) = 5 kWh

(c) 100,000 W = 100 kW


Energy = (100 kW)(5 hr) = 500 kWh

5.6 Cost of Electricity Usage

To find the cost of electricity usage on a bill, use the following:

watts x hours used x rate per kilowatthour


Cost =
1000

For example, an electric heater that draws 1350 watts is used for 4 hours, and the
cost of electricity for that particular location is Php 10 per kilowatthour. What is the
cost of using the heater?

1350 x 4 x 10
Cost = = Php 54
1000

.
153

Problem Set No. 9

ENERGY AND ELECTRIC POWER

1. Determine P if R = 30  and I = 3 A.
A. 90 W
B. 900 W
C. 270 W
D. 2700 W

2. Determine P if V = 120 V and I = 3 A.


A. 360 W
B. 3600 W
C. 4800 W
D. 1080 W

3. Let us consider the element shown in the figure below when v = 4 V and i = 10 A.
Find the power absorbed by the element and the energy absorbed over a 10-s
interval.
i
a b
+ v -

A. 20 W, 900 J
B. 40 W, 300 J
C. 20 W, 800 J
D. 40 W, 400 J

4. Consider the circuit shown in the figure below with v = 8e-t V and i = 20e-t A for t
≥ 0. Find the power supplied by this element and the energy supplied by the
element over the first second of operation. Assume that v and i are zero for t < 0.
i
a b

- v +

A. 69.17 W, 69.17 J
B. 69.17 W, 21.65 J
C. 21.65 W, 69.17 J
D. 21.65 W, 21.65 J
154

5. The average current in a typical lightning thunderbolt is 2 x 104 A and its typical
duration is 0.1 s. The voltage between the clouds and the ground is 5 x 108 V.
Determine the total charge transmitted to the earth and the energy released.
A. 20,000 C, 1 x 1014 J
B. 2000 C, 1 x 1012 J
C. 200 C, 1 x 1012 J
D. 20 C, 1 x 1021 J

In nos. 6 to 10 refer to the figures below. The four circuit elements are identified by
the letter A, B, C, and D

3A 6A
A A

+ 4V - - 2V +
(A) (B)

2A 4A

+ 6V - - 3V +

(C) (D)

6. Which of the devices supply 12 W?


A. B, C
B. A, D
C. B, D
D. A, C

7. Which of the devices absorb 12 W?


A. B, C
B. A, D
C. B, D
D. A, C

8. What is the value of the power received by device B?


A. -12 W
B. 12 W
C. 3 W
155

D. -3 W

9. What is the value of the power delivered by device B?


A. -12 W
B. 12 W
C. 3 W
D. -3 W

10. What is the value of the power delivered by device D?


A. -12 W
B. 12 W
C. 3 W
D. -3 W

11. Calculate the resistances of a 100-W 120-V incandescent lamp.


A. 12 Ω
B. 14.4 Ω
C. 144 Ω
D. 120 Ω

12. A bulb having a rating of 220 V, 75 watts, is to be connected to a 110-V source.


Find the power it dissipates.
A. 16 W
B. 17.65 W
C. 18.25 W
D. 18.75 W

13. Determine the number of kilowatt-hours if 1400 W is used in 1 hr.


A. 1.5 kWhr
B. 1.4 kWhr
C. 1.3 kWhr
D. 1.2 kWhr

14. A pulse of electricity measures 305 V, 0.15 A, and lasts 500 s. What power and
energy does this represent?
A. 38.25 W, 0.022875 J
B. 40.75 W, 0.022875 J
C. 45.75 W, 0.05689 J
D. 45.75 W, 0.022875 J
156

15. A unit of power used for electric motors is the horsepower (hp), equal to 746
watts. How much energy does a 5-hp motor deliver in 2 hours? Express the
answer in MJ.
A. 22.856 MJ
B. 24.856 MJ
C. 26.856 MJ
D. 36.856 MJ

16. A certain circuit element has the current and voltage

i = 10e-5000t A v = 50(1 – e-5000t) V.

Find the total energy transferred after t = 3 sec.


A. 2.53 x 10-20 J
B. 2.53 x 10-27 J
C. 2.53 x 10-25 J
D. 2.23 x 10-27 J

17. The unit of energy commonly used by electric utility companies is the kilowatt-
hour (kWh). (a) How many joules are in 1 kWh? (b) A color television set rated at
75W is operated from 7:00 p.m. to 11:30 p.m. What total energy does this
represent in kilowatt-hours?
A. 3.6 J, 0.3375 kWhr
B. 360 J, 0.3375 kWhr
C. 3.6 kJ, 0.3375 kWhr
D. 36 kJ, 0.3375 kWhr

18. The power absorbed by the BOX shown in the figure below is p(t) = 2e−2t W .
Calculate the energy absorbed by the BOX and the charge that enters the BOX
between 0.1 and 0.4 seconds.

i(t)
C1
BOX
E
5 𝑒 −𝑡12𝑉V 1µF

A. 0.2565 J
B. 0.3182 J
C. 0.3694 J
D. 0.3567 J
157

Objective Test No. 8

POWER AND ENERGY

1. Commercial unit of electrical energy


A. joule
B. watt-hour
C. megawatt
D. kilowatthour

2. A 25-W incandescent bulb rated 120 V and operated on a 120-V line has burnt
out and has to be replaced as soon as possible. There are several lamps
available but not of the same rating. Which one of the following bulbs should
be used to approximate the power consumption of the busted bulb?
A. 20 watts, 110 volts
B. 50 watts, 240 volts
C. 75 watts, 220 volts
D. 100 watts, 240 volts

3. One horsepower is equivalent is how many watts?


A. 476
B. 674
C. 746
D. 764

4. A carbon composition resistor when connected across a 24-V battery draws a


current of 1 mA. What is the ,minimum required resistance and wattage rating
of the resistor?
A. 24 ohms, 1/8 W
B. 24,000 ohms, ¼ W
C. 24,000 ohms, 1 W
D. 24,000 ohms, 2 W

5. How many kilowatts is the power 200 joules per second equal to?
A. 0.02
B. 0.2
C. 20
D. 200

6. It was experientially found by James Prescott Joule that the heat produced in a
current carrying conductor is proportional to _____________.
A. the current
B. the square of the current
C. the square of the resistance
158

D. the square of the conductance

7. One joule of electrical energy is equivalent to


A. one watt-minute
B. one watt-second
C. one kilowatt-hour
D. one watt per second

8. Find the cost of running 100-W, 220V- lamp for 20 hours at P3.00 per kW-hr.
A. P6.00
B. P10.00
C. P12.00
D. P24.00

9. How much current does a 24-ohm resistance that dissipates 600 watts need?
A. 0.04 A
B. 1.04 A
C. 5 A
D. 25 A
159

5.7 Energy Conversion

One of the fundamental laws of classical physics states that, under ordinary
conditions, energy can neither created nor destroyed. The energy in the universe
exists in various forms, such as heat energy, light energy, and electric energy. When
we say we “use” electric energy, we do not mean that we have destroyed, or lost, the
energy. We mean that we have converted that electric energy into a more useful form
of energy. For example, when we operate an electric lamp, we are converting electric
energy into light energy and heat energy. We have used the electric energy in the
sense that it no longer exists as electric energy, but we have not used up the energy.
It still exists as heat energy and light energy.

5.8 Efficiency

No conversion process is 100 percent efficient. That is, not all the energy put into a
device or a system is converted into the form of energy we desire. When 1000 joules
of electric energy is put into a light bulb, only 200 joules of light energy is produced.
The other 800 joules is converted into heat energy. We could say that the efficiency
of the light bulb is low.

The efficiency of a system is usually expressed as a percentage. It is calculated by


the formula

useful energy out


Percent efficiency = x 100
total energy in

By abbreviating percent efficiency to %eff, and using the symbol W for energy, we
can write this formula as

Wout
% eff. = x 100
Win

Example 5.8 What is the efficiency of a light bulb that uses 1000 joules of electric
energy to produce 200 joules of light energy?

Solution:
Wout
% eff. = x 100
Win

200 joules
% eff. = x 100
1000 joules
efficiency = 20%
160

Example 5.9 The figure below is a nameplate of a motor. What data can you find
regarding the motor?

Example 5.10 A 1-hp motor draws 1000 watts. What is its efficiency?
output
Efficiency = input

Therefore,
746
Efficiency = = 0.746, or 74.6%
1000

Example 5.11 A motor must lift an elevator car weighing 2000 pounds to a height
of 1000 feet in 4 minutes. (a) What is the theoretical size, in horespower, of the
motor required? (b) At 50% efficiency, what is the size, in horespower, of the
motor required?

(a) W = 2000 x 1000 = 2,000,000 ft-lb

2,000,000
= 500,000 ft − lb per minute
4

500,000
= 15.15 hp
33,000

output 15.15
(b) Input = = = 30.3 hp
efficiency 0.50

Example 5.12 What is the overall efficiency of a 5-hp motor that draws 20 amperes
at 240 volts.
Input = 240 x 20 = 4800 watts
Output = 5 x 746 = 3730 watts
3730
Efficiency = 4800 = 0.777, or 77.7%
161

Assessment No. 11

WORK, POWER AND ENERGY

Name: Ybañez, Eric L. Score: _________ Rating: ______

1. What is the efficiency of a motor which requires 914 Joules of electric energy to
produce 585 joules of mechanical energy?

2. How much energy is required by an electric lamp that is 18 per cent efficient and
provides 5463 J of light energy?

3. A motor is 70 percent efficient and requires 1200 J of energy. How much energy
does it provide?

4. An electric motor requires 1760 joules of electric energy to produce 1086 joules
of mechanical energy. What is the efficiency of the motor?
162

5. A flashlight uses 110 joules of chemical energy to supply 100 joules of electric
energy to the flashlight bulb. What is the efficiency of the battery?

6. How much mechanical energy will be produced by a motor that is 70 per cent
efficient and requires 1960 joules of electric energy?
163

5.9 Terminal Potential Difference

Effective voltage provided by a voltage source to the load

The potential difference measured across the resistor itself is always lower than the
actual potential difference provided by the source for the circuit, because of the
voltage source’s own internal resistance. The potential difference across the resistor
under closed circuit condition is called its terminal potential difference.

It is mathematically defined by:

V = ε − Ir

+
r
0.2ohm + -
Battery 11.881 V

Em f RL
20ohm
12V

where: ε = emf, absolute potential provided by the battery; it is


also the amount of potential a source has while in an open circuit.
r = the internal resistance of the battery
V = terminal potential difference

Electromotive Force (emf)

The force that causes a current of electricity to flow.

Potential Difference (pd)

The electrical pressure or voltage required to drive the current between


two points.

Example 5.13 The terminal voltage of a battery is 9 V when supplying a current of


4 A and 8.5 V when supplying 6 A. Find its (a) internal resistance, and (b) emf.
164

Solution:

(a) 9 = ε − 4r
8.5 = ε − 6r

0.5 = 2r

r = 0.25 

(b) ε = 9 + 4(0.25) = 10 V

Example 5.14 A battery of emf 9 V and internal resistance of 0.1  is connected to


a resistance R = 4 . Determine (a) the current in the circuit, (b) the potential drop
in the internal resistance and also across the 4- resistance, (c) the terminal voltage
of the battery, (d) the reading of a voltmeter across the battery terminals in open
circuit.

Solution:
 9
(a) I = = = 2.195 A
R+r 4+0.1

(b) Vr = Ir = 0.2195 V
VR = IR = 8.78 V

(c) V = ε − Ir = 8.78 V

(d) 9 V, since it is open circuit potential


165

Problem Set No. 10

TERMINAL POTENTIAL DIFFERENCE

1. A dry cell has an emf of 1.52 V. Its terminal potential drops to zero when a current
of 25 A passes through it. What is its internal resistance?
A. 0.0503 ohm
B. 0.0608 ohm
C. 0.0790 ohm
D. 0.0852 ohm

2. What is the emf of a battery whose internal resistance is 2  if the terminal


voltage is 16 V when a current of 2 A flows through the battery?
A. 5 V
B. 10 V
C. 20 V
D. 30 V

3. Find the internal resistance of a battery having an emf of 6 V if it supplies 2 A


under load at a terminal voltage of 1 V.
A. 1.0 ohm
B. 1.5 ohms
C. 2.5 ohms
D. 3 ohms

4. A battery has an emf of 10 V and an internal resistance of 2 . The battery is


placed in a circuit with a single external resistor. The terminal voltage across the
battery is 8 V when the circuit is closed. Find the resistance of the external
resistor.
A. 2 ohms
B. 4 ohms
C. 6 ohms
D. 8 ohms

5. A cell whose emf is 1.45 V has an internal resistance of 4 ohms. What current
will flow if this cell is connected across a 1-ohm resistor?
A. 0.12 A
B. 0.02 A
C. 0.2 A
D. 0.29 A

6. If the current from a short-circuited 1.5 V dry cell is 25 A, what is the internal
resistance of the cell?
166

A. 0.06 ohm
B. 0.02 ohm
C. 0.2 ohm
D. 2 ohms

Objective Test No. 9


TERMINAL POTENTIAL DIFFERENCE

1. A 50- V battery is connected across a 10 -  resistor and a current of 4.5 A


flows. What is the internal resistance of the battery?
A. 0 ohm
B. 1.25 ohms
C. 1.11 ohms
D. 2.05 ohms

2. A load of 10- was connected to a 12-V battery. The current drawn was 1.18
amperes. What is the internal resistance of the battery?
A. 0.169 
B. 0.265 
C. 0.390 
D. 0.375 

3. The electromotive force of a standard cell is measured with a potentiometer


that gives a reading of 1.3562 volts. When a 1-M resistor is connected across
the standard cell terminals, the potentiometer reading drops to 1.3560 volts.
What is the internal resistance of the standard cell?
A. 145.7 
B. 147.5 
C. 157.4 
D. 174.5 

4. A 6-volt lead-acid battery has an internal resistance of 0.01 ohm. How much
current will flow if the battery has a short circuit?
A. zero
B. 60 A
C. 600 A
D. infinity

5. A dry cell has internal resistance of 0.02 ohm and terminal voltage of 1.5 V on
open circuit. What will be its terminal voltage if a 0.1-ohm resistance is
connected across its terminals?
A. 1.15 V
B. 1.20 V
C. 1.25 V
167

D. 1.42 V

6. A dry cell has an internal resistance of 0.02 ohm and open circuit voltage of 1.5
V. Calculate the power delivered to a resistor of 0.6-ohm resistance.
A. 2.0 watts
B. 2.4 watts
C. 3.5 watts
D. 3.8 watts

7. A 10-ohm resistor is connected across a battery whose internal resistance is 5


ohms. If the voltage across the 10-ohm resistance is 12 V, how much is the
open circuit voltage of the battery?
A. 15 V
B. 18 V
C. 20 V
D. 24 V

8. A cell of emf 1.45 V is connected to an external circuit of resistance of 2.25


ohms and the current is found to be 0.2 A. What is the internal resistance of
the cell?
A. 3 ohms
B. 4 ohms
C. 5 ohms
D. 6 ohms

9. A battery having a total emf of 7.5 volts and a total internal resistance of 1.25
ohms. What external resistance will send a current of 2 A?
A. 1.0 ohm
B. 1.74 ohms
C. 2.0 ohms
D. 2.5 ohms

10. A battery whose internal resistance is 5 ohms is connected to an external


resistor of 10 ohms. The battery’s terminal voltage is 15 V, what is the emf of
the battery?
A. 17.5 V
B. 22.5 V
C. 25 V
D. 50 V

.
168

5.10 Power Loss in Distribution Feeder Lines

5.10.1 The Power Distribution Feeder Lines

conductor

sending end receiving end

E 412
R10.5Ω V
023.5Ω
12R2
X1 V1

Power from the sending end is transmitted to the load (receiving end) through
conductors. Power is normally reduced due to the resistance of conductors. This
power is known as power loss.

Example 5.15 A lamp with a resistance of 15 ohms is connected across a 12-V


through two pieces of wires each having a resistance of 0.5 ohm. Find the power
taken by the lamp and the power loss in the two wires.
conductor

0.5Ω
sending end receiving end
E
1 4
12 V R1312 V
2R2 15ohms

0.5Ω
169

Solution:
E
I = R+r

where R is the resistance of the load , r is the total resistance, E is the emf of
the sending end, and I is the current through the line and through the lamp.
r

E conductor
12 V
receiving
3 end
2 end 15ohms
12 V sending
1

12
I = = 0.75 Ω
15+1

The power taken by the lamp is


P = I2 R = (0.75)2 (15) = 8.44 W
The power loss in the wire
P = I2 r = (0.75)2 (1) = 0.5625 W
.
170

Assessment No. 12

POWER, ENERGY, EFFICIENCY

Name: Ybañez, Eric L. Score: _________ Rating: ______

Answer the questions in your own words:

1. What is the effect of electricity?

2. Explain the difference between power and energy.

3. What are the different types of energy conversion.

4. What is efficiency?

5. Find the power dissipated by a heater having a resistance of 250 ohms that is
connected across a 220-V supply.

6. Find the energy consumed by the heater in number 5 when it is used in 6 hours.

7. Find the efficiency of a 1-hp air-conditioning unit when it draws 3.7 A from a
220-V source.
171

QUESTIONS: (not part of Assessment No. 12)

1. What is electricity?
2. Discuss the probable changes in your lifestyle if the electric service in your home
were disconnected for one week.
3. What is power?
4. What is energy?
5. What are the forms energy into which the electric energy from a car battery is
converted?
6. What is the undesirable form of energy produced by both light bulbs and electric
motors?
7. What is efficiency?
8. In addition to decreasing costs, why is it important to increase the efficiency of
electric devices as much as it is practical?
9. Is the low efficiency of the lights in our home a greater disadvantage during the
cold months or the summer months? Why?
10. What happens to the temperature of an electric battery when it is discharging?
Why?
11. Many electric devices could be designed and constructed to operate more
efficiently. Why aren’t they?
12. Explain how energy is measured in kilowatthours.
.
172

Unit 6

SERIES CIRCUIT

A.
LEARNING
B. OUTCOMES

After completing this


unit, you are expected to:

1. identify a series circuit.


2. determine the current in a series circuit.
3. determine the total resistance in a series circuit.
4. apply Ohm’s law to find the current, voltages, and
resistances in a series circuit.
5. connect voltage sources to achieve a higher voltage.
6. apply Kirchhoff’s voltage law.
7. devise and use voltage dividers.
8. determine the total power in a series circuit.
173

6.1 The Meaning of a Series Circuit

R1

60ohm

E R2
60ohm
12V

R3

60ohm

A Series Circuit

Series - components are connected end to end.

6.2 Characteristics of a Series Circuit:

1. The same current I exist on each resistor.

+ - R1 +
0.067 A 0.067 A
-
60ohm

E
R3
12V 60ohm

R2 - +
0.067 A

60ohm

𝑉 𝑉2 𝑉𝑛
𝐼 = 𝑅1 = =. ..
1 𝑅2 𝑅𝑛

IT = I1 + I2 + I3 + . . . . In

2. The source voltage E (or VT) is the summation of the voltages across each
resistor.

E = V1 + V2 + V3 + . . . . Vn or

VT = V1 + V2 + V3 + . . . . Vn
174

+ -
4.000 V

R1

60ohm

R2 +
E 60ohm
4.000 V
-
12V

R3

60ohm

- +
4.000 V

3. The total resistance RT is the sum of the individual resistances.

R T = R1 + R 2 + R 3 + . . . . R n

R1

60ohm
XMM1
R2
60ohm

R3

60ohm

4. The total power is the summation of the powers dissipated by each resistor.

PT = P1 + P2 + P3 + . . . . Pn

5. Any change in one or more


components will affect the L1
other components.
12V_25W

V2 L2
36V 12V_25W

L3

12V_25W
175

6.3 Resistance of Two or More Equal Resistors in Series

RT = nR

where R T = total resistance


n = number of resistors
R = the resistor value

6.4 Voltage Divider

R1

50ohm

R2 +
E 75ohm 4.000 V
-
12V

R3

100ohm

Formulas
VT
V1 = x R1
RT

VT
V2 = x R2
RT

Generally
VT
Vx = x Rx
RT

where: Vx is the voltage across resistor Rx

6.5 Voltage Sources in Series

Example 6.1 Find VT


176

V1 V2 V3 V4

VT
12V 12V 12V or
12V
48V
+ -
48.000 V

VT

VT = 12 + 12 + 12 + 12

VT = 48 V

Example 6.2 Determine VT

V1 V2 V3 V4

12V
or VT
12V 12V 12V 24V

+ -
24.000 V

VT = 12 + 12 – 12 + 12

VT = 24 V

Example 6.3 Determine (a) total resistance, (b) total current, (c) the voltage
across each resistor.
R1

10ohm

V R3
30ohm
12V

R2

60ohm
Solution:

(a) R T = R1 + R 2 + R 3
177

R T = 10 + 60 + 30

R T = 100 

VT 12 V
(b) IT = = = 0.12 A or 120 mA
RT 100 

(c) V1 = I1 R1 = (0.12)(10) = 1.2 V

V2 = I2 R 2 = (0.12)(30) = 3.6 V

V3 = I3 R 3 = (0.12)(60) = 7.2 V

The figure below shows the measured current and voltage across the each resistor.

+ -
1.200 V

+ - R1
0.120 A
10ohm

V R3 +
3.600 V
30ohm -
12V

R2

60ohm
- +
7.200 V

Example 6.4 Three resistors R1 = 10 , R 2 = 20  and R 3 = 50  are


connected in series across a 120-V dc source. Calculate:

(a) The total resistance RT.


(b) The total current IT drawn by the circuit.
(c) The voltage across R3.
(d) The power dissipated in R3.
(e) The total power delivered by the source.

Solution:
178

R1 + -
1.500 A
10ohm

R2
20ohm
E
120V

R3

50ohm
- +
74.999 V

A. The total resistance RT.

R T = R1 + R 2 + R 3
= 10 + 20 + 50
R T = 80 

B. The total current IT drawn by the circuit.

VT 120
IT = =
RT 80

IT = 1.5 A

C. The voltage across R3.

V3 = IT R 3
= (1.5) (50)
= 75 volts

D. The power dissipated in R3.

P3 = IT 2 R 3 = (1.5)2 (50) = 112.5 watts or


P3 = V3 IT = (7.5)(1.5) = 112.5 watts or
P3 = V3 2 / R 3 = (75)2 /50 = 112.5 watts

E. The total power delivered by the source

PT = IT 2 R T = (1.5)2 (80) = 180 watts or


2
PT = IT R T = (120)(1.5) = 180 watts or
PT = IT 2 R T = (120)2 /80 = 180 watts
179

Example 6.5 Determine the single voltage and resistance corresponding the circuit
below.

R1 R2

50ohm 6ohm

V2
V1 30V
90V

V3
R3 R4

5ohm 11ohm
10V

Veq = 90 – 30 – 10 = 50 V

R eq = 50 + 6 + 11 + 5 = 72 
180

Assessment No. 13

SERIES CIRCUIT

Name: Ybañez, Eric L. Score: _________ Rating: ______

1. Three resistors R1, R2, and R3 , are connected in series across a 120-V DC source.
R1 = 30 , R2 = 60 , R = 90 . Find (a) the total resistance; (b) the total
current; (c) the voltage across each resistor; (d) the power taken by each
resistor; (e) the total power.

2. The resistors are connected in series across a 120-V source. The first resistor is
50 , the current through the second resistor is 0.5 A, and the voltage drop
across the third resistor is 50 V. What are the resistances of the first and third
resistors ?
181

QUESTIONS: (not part of Assessment No. 13)

1. What is series circuit?


2. What are the characteristics of a series circuit?
3. What is the purpose of connecting voltage sources in series?
182

Practical Application No. 4

Name: Ybañez, Eric L. Score: _________ Rating: ______

1. A 12-V automobile head lamp is to be used on a fishing boat with a 24-V power
system. The head lamp is rated at 50 W. A resistor is to be connected in series
with the lamp to permit it to operate on 24 V. What should be the resistance and
power rating of the resistor?

2. Three wire-wound resistors have the following values: 30 Ω, 80 Ω, and 100 Ω.


Each resistor has a voltage rating of 100 V. If these three resistors are connected
in series, can they be connected to a 240-V circuit without damage to the
resistors? Explain your answer.
183

3. You are an electrician working in an industrial plant. A circuit contains eight


incandescent lamps connected in series across 480 volts. One lamp burned out,
and you must determine which one is defective. You have available a voltmeter,
ammeter and ohmmeter. Which meter would you use to determine which lamp
is defective in the shortest possible time? Explain how you would use this meter
and why?

.
184

Problem Set No. 11

SERIES DC CIRCUIT

1. Fill in the ammeter and voltmeters with the correct readings. Write your
calculations on the space below.
+ -
0.000 V

+ - R1
0.000 A

20ohm

V1 R2 +
0.000 V
90V 30ohm
-

R3 R4

15ohm 10ohm
- + - +
0.000 V 0.000 V

A. 1.2 A, V1 = 24 V, V2 = 36 V, V3 = 12 V, V4 = 18 V
B. 1.25 A, V1 = 24 V, V2 = 26 V, V3 = 12 V, V4 = 18 V
C. 1.2 A, V1 = 24 V, V2 = 36 V, V3 = 14 V, V4 = 18 V
D. 1.2 A, V1 = 24 V, V2 = 36 V, V3 = 12 V, V4 = 28 V

2. A set of Christmas light of twenty-five 650- bulbs connected to a 220-V source.


What is the (a) total resistance of the Christmas light, (b) potential difference
across each light bulb, and (c) power dissipated by each bulb?
A. 16,350 ohms, 8.75 V, 119.14 mW
B. 16,250 ohms, 8.8 V, 119.14 mW
C. 16,250 ohms, 8.7 V, 110.14 mW
D. 12,250 ohms, 8.8 V, 119.14 mW

3. Three resistor RA, RB, and RC are connected in series and to a 120-V source. If RB
= 2RA, RC = 3RA, and the total power taken by the circuit is 200 watts, calculate
(a) the resistance of each resistor, (b) the power in each resistor.
A. RA = 14 Ω, RB = 28 Ω, RC = 38 Ω, PA =33.33 W, PB = 66.67, PC = 100 W.
B. RA = 6 Ω, RB = 12 Ω, RC = 18 Ω, PA =33.33 W, PB = 66.67, PC = 100 W.
C. RA = 12 Ω, RB = 24 Ω, RC = 36 Ω, PA =33.33 W, PB = 66.67, PC = 100 W.
D. RA = 24 Ω, RB = 24 Ω, RC = 72 Ω, PA =33.33 W, PB = 66.67, PC = 100 W.
185

4. Three resistors A, B, and C are connected in series to a 117-V source. If RA = 64


ohms, and EB = 40 volts when the current is 0.5 amp, calculate the resistances RB
and RC.
A. RB = 80 Ω, RC = 90 Ω. C. RB = 100 Ω, RC = 190 Ω.
B. RB = 90 Ω, RC = 80 Ω. D. RB = 120 Ω, RC = 240 Ω.

5. A 6-ohm load is connected to a 119.6-V source through a pair of 0.25-ohm


conductors. Calculate (a) the load current and voltage, (b) the voltage drop in the
line wires.
A. 18.4 A, 110.4 V, 9.2 V C. 18.4 A, 110.4 V, 9.8 V
B. 16.4 A, 110.4 V, 9.2 V D. 18.4 A, 100.4 V, 9.2 V

6. A 72-ohm coil of wire is connected in series with an adjustable resistor (rheostat)


whose resistance can be varied from 0 to 88 ohms. If the line potential is 115
volts, calculate (a) the range through which the coil current can be varied, (b) the
rheostat resistance when the power taken by the coil is 90 watts.
A. 218.75 mA – 1.6 A, 28.68 Ω C. 418.75 mA – 1.6 A, 30.68 Ω
B. 718.75 mA – 1.6 A, 30.68 Ω D. 618.75 mA – 1.6 A, 24.68 Ω

7. A telegraph circuit consists of a 250- relay in series with fine wires having a
resistance of 50 . If takes 72 mA to operate the relay, what voltage must be
impressed at the sending end of the circuit?
A. 1.6 V B. 11.6 V C. 12.6 V D. 21.6 V

8. To determine the resistances of three resistors A, B, and C the following procedure


is followed: RA and RB are connected in series and an emf of 21 volts is impressed
for a current of 1 A: RB and RC are next connected in series and an emf of 27 volts
is impressed for the same adjusted current; finally RC and RA are connected in
series and an emf of 24 volts is impressed for the same adjusted current. What
are the ohmic values of A, B, and C?
A. RA = 6 Ω, RB = 9 Ω, RC = 13.5 Ω
B. RA = 9 Ω, RB = 12 Ω, RC = 18 Ω
C. RA = 9 Ω, RB = 12 Ω, RC = 15 Ω
D. RA = 6 Ω, RB = 10 Ω, RC = 18 Ω

9. A generator delivers a load through a pair of wires, each of which has a resistance
of 0.06 ohm. If the load voltage and power are, respectively, 120 volts and 4.8 kW,
calculate (a) the generator voltage, (b) the power loss in the line wires.
A. 121.8 V, 192 W
B. 122.8 V, 192 W
C. 123.8 V, 192 W
D. 124.8 V, 192 W
186

Objective Test No. 10

SERIES CIRCUIT

1. When n equal resistors are connected in series to a source of emf E volts, each
having a resistance of R ohms, which of the following statements is NOT true?
A. The current through each of the resistors is the same.
B. The equivalent resistance of the circuit is equal to nR.
C. The voltage drop across one of the resistors is equal to E/n.
D. The equivalent resistance of the circuit is equal to R/n.

2. Three resistors R1, R2, and R3 are connected in series across a 100-V source. If
R2 opens, the
A. total resistance decreases
B. voltage across R2 is zero
C. voltage across R1 is 100 V
D. the voltage across R2 is 100 V

3. If four 4-ohm resistors are connected in series, the total equivalent resistance
will be
A. 1 ohm
B. 12 ohms
C. 16 ohms
D. 23 ohms

4. In a series circuit, the current is


A. zero
B. constant
C. proportional to the resistance
D. different in different resistors

5. In a series circuit, the total resistance is _______.


A. the sum of all the resistances
B. the average of all resistances
C. smallest than the smallest resistance
D. the sum of the reciprocals of all resistances

6. Five carbon-zinc cells are in series. The open circuit voltage at the output is
A. 5.5 V
B. 6.5 V
C. 7.5 V
D. 10 V
187

7. The total voltage and amperage of four 0.5-A, 1.5-V cells connected in series
are ______.
A. 1.5 V, 0.5 A
B. 1.5 V, 2 A
C. 6 V, 0.5 A
D. 6 V, 2 A

8. Two equal resistances are connected in series across a certain voltage source.
If the current drawn by the combination is equal to the applied voltage, how
much is the value of each resistance?
A. 0.25 ohm
B. 0.5 ohm
C. 0.75 ohm
D. 1 ohm

9. An arc lamp takes 10 A at 50 V. Find the value of the resistance to be placed in


series so that the lamps may burn correctly from a 110-V supply.
A. 5 ohms
B. 6 ohms
C. 10 ohms
D. 11 ohms
188

Unit 7

PARALLEL CIRCUIT

C.
LEARNING
D. OUTCOMES

After completing this


unit, you are expected to:

1. identify a parallel circuit.


2. apply Kirchhoff’s current law.
3. determine total resistance of a parallel circuit.
4. apply Ohm’s law to find voltage, currents, and resistances
in a parallel circuit.
5. describe how parallel circuits act as current divider.
6. apply the current divider principle.
7. determine the total power in a parallel circuit.
189

7.1 The Meaning of a Parallel Circuit

E
R1 R2 R3
12V
1kohm 1kohm 1kohm

Parallel - components are connected across each other.

7.2 Characteristics of a Parallel Circuit

1. The same voltage V exists across each resistor.


VT = V1 = V2 = V3 = Vn

E R2 + R3 +
R1 +
240ohm 12.000 V
240ohm 12.000 V
12V 12.000 V - -
240ohm -

2. The total current IT is the sum of individual current passing through each
resistor.

- + + +
0.480 A 0.240 A 0.160 A 0.080 A
+ - - -

E
R1 R2 R3
12V
50ohm 75ohm 150ohm

IT = I1 + I2 + I3 + . . . In

and
190

VT VT VT VT
I1 = ; I2 = ; I3 = ; In =
R1 R2 R3 Rn

3. The reciprocal of the total resistance is equal to the sum of the reciprocal of
individual resistances, that is,

1/RT = 1/R1 + 1/R2 + 1/R3 + . . . 1/Rn

1
RT = 1 1 1 1
+ + + ….
R1 R2 R3 Rn

or RT = (R1-1 + R2-1 + R3-1 + . . . + Rn-1)-1

Since 1/R = G

Therefore, GT = G11 + G2 + G3 + . . . Gn

That is the total conductance is the sum of the individual conductances.

XMM1
R1 R2 R3
50ohm 75ohm 150ohm

4. Just like in a series circuit, The total power is the summation of the powers
dissipated by each

PT = P1 + P2 + P3 + . . . .Pn

5. Each component can be controlled independently.

E L1 L2 L3
10W 12V 10W 12V 10W 12V
12V

S1 S2 S3
Key = A Key = A Key = A
191

7.3 Two Resistances in Parallel

R1 R2
75ohm 150ohm

R1 R2
RT = R
1 + R2

7.4 Equal-Value Resistors

R1 R2 R3
E 100ohm 100ohm 100ohm
120V

R
RT = n
R
RT =
n

where: RT - total resistance


R – the value of the resistor
n – number of resistors

7.5 Voltage Sources in Parallel

Ideal voltage sources are connected in parallel in order to supply a higher and at the
same time a high power to a load.

In industry paralleling of voltage sources such as generators are done in order to


sustain an increasing amount of load.

Conditions of Parallel Connection of Voltage Sources:

1. They must have the same terminal voltage.


2. They must be connected in the same polarity.
192

7.6 Current Sources in Parallel

Example 7.1 Find the equivalent current.

I1 I2 I3 +
IT
3.000 A
1A 1A 1A - or 3A

Solution:
The equivalent current is

IT = I1 + I2 + I3
IT = 1 + 1 + 1
IT = 3 A

Example 7.2 Find the equivalent current.

IT
I1 I2 I3 3A
1A 3A
or
1A

Solution:
Ieq = 1 + 3 - 1 = 3 A

7.7 Current Divider

7.7.1 Current Divider for two resistors in parallel

Current division is used to express the current through each one of the
resistors connected in parallel.
193

R1 R2
I 75ohm 150ohm
8A
I1 I2

When an ammeter is connected through each resistor, the meters read 5.333 A and
2.6667 A.

IT + +
5.333 A 2.667 A
- -

I
R1 R2
8A
75ohm 150ohm

I1 I2

Formulas

R2
I1 = IT ( )
R1 + R2

R1
I2 = IT ( )
R1 + R2

7.7.2 Current Divider for three or more resistors in parallel

IT
R1 R2 R3
E 100ohm 50ohm 150ohm
120V
194

Formula

RT
Ix = IT ( )
Rx

where Ix is the current through Rx. x represents any number of R. For example R1,
R2, etc.

Example 7.3 Find (a) the total resistance, (b) the total current, (c) the current
through each resistor using current divider formula.

IT
V1 R1 R2
I1 1kohm
I2 500ohm
12V

Solution:

R1 R2
(a) RT = R1 + R2

(1000)(500)
RT = 1000+ 500

R T = 333.333 

VT 12 V
(b) IT = = = 36 mA
RT 333.33 Ω

R2
(c) I1 = IT (R )
1 + R2

500
= 36 mA (1000+ 500)

I1 = 12 mA

R1
I2 = IT (R )
1 + R2

1000
= 36 mA (1000+ 500)

I2 = 24 mA
195

Example 7.4 Find the total resistance

V1 R1 R2 R3 R4
1kohm 1kohm 1kohm 1kohm
12V

R
Solution: RT = n

1000
RT = = 250 
4

Example 7.5 Determine the (a) total resistance, (b) current through each resistor
and (c) the total current.

R1 R2 R3
E 100ohm 50ohm 150ohm
120V

Solution:

(b) RT = (100-1 + 50-1 + 150-1)-1 = 27.2727

V1 120 V
(c) I1 = = = 1.2 A
R1 100 Ω

V2 120 V
I2 = = = 2.4 A
R2 50 Ω
V3 120 V
I3 = = = 0.8 A
R3 150 Ω

Note: E = V1 = V2 = V3

VT 120 V
(d) IT = = = 4.4 A
RT 27.2727 Ω

or IT = I1 + I2 + I3
IT = 1.2 + 2.4 + 0.8
IT = 4.4 A
196

Example 7.6 Three resistors R1 = 10 , R2 = 20  and R3 = 30  are connected


in parallel. The circuit voltage source is 60 volts. Calculate:

A. The total resistance RT.


B. The total conductance GT
C. The total current IT delivered by the source.
D. The current through R3.
E. The power dissipated in R3.
F. The total power delivered by the source

Solution:

-
11.000 A
+

VT R1 R2 R3
10ohm 20ohm 30ohm
60V +
2.000 A
-

B. The total resistance RT.


1
RT = 1 1 1
+ +
R1 R2 R3

1
RT = 1 1 1
+ +
10 20 30

RT = 5.45  or

RT = (R1-1 + R1-1 + R1-1)-1

RT = (10-1 + 20-1 + 30-1)-1

RT = 5.45 

C. The total conductance GT


1 1
GT = RT
= 5.45
197

GT = 0.18333 S

D. The total current IT.

VT 60
IT = =
RT 5.45

IT = 11 A

E. The current through R3


VT 60 V
I3 = =
R3 30 Ω
IT = 2 A

F. The power dissipated in R3

P3 = I32R3 = (2)2(30) = 120 watts

G. The total power

PT = VTRT = (60)(11) = 660 watts

Example 7.7 Find the current in the 30- resistor.

I I1 I2 R1 R2 R3 R4 R5
1A 1A 75ohm 150ohm 30ohm 50ohm 20ohm
8A

Solution:

To simplify the circuit,

The equivalent current source is

Ieq = 8 – 1 –1 = 6 A

The equivalent resistance of 75, 150, 50, and 20- resistors is 11.11 .
198

R3 Req
I 30ohm 11.11ohm
6A
I3

By current division

11.11
I1 = 6 A (30 + 11.11)

I1 = 1.62 A
199

Assessment No. 14

PARALLEL CIRCUIT

Name: Ybañez, Eric L. Score: _________ Rating: ______

1. Three resistors R1, R2, and R3 , are connected in parallel across a 120-V DC
source. R1 = 30 , R2 = 60 , R = 90 . Find (a) the total resistance; (b) the
total current; (c) the current through each resistor; (d) the power taken by
each resistor; (e) the total power.

2. The total current drawn by a circuit consisting of three resistors connected in


parallel is 12 A. The voltage drop across the first resistor is 12 V, the value of
second resistor is 3 Ω and the power dissipation of the third resistor is 24 W.
What are the resistances of the first and third resistors ?
200

QUESTIONS: (not part of Assessment No. 14)

1. What is a parallel circuit?


2. What are the characteristics of a parallel?
3. What is purpose of connecting voltage sources in parallel?
4. What are the different requirements of connecting voltage source in parallel?
5. All appliances in a household are connected in parallel. Why?
201

Practical Application No. 5

ATOMS, ELECTRICAL CHARGE, VOLTAGE, AND CURRENT

Name: Ybañez, Eric L. Score: _________ Rating: ______

1. You are employed in a large industrial plant. A 480-V, 5000-W heater is used to
melt lead in a large tank. It has been decided that the heater is not sufficient to
raise the temperature of the lead to the desired level. A second 5000-W heater is
to be installed on the same circuit. What will be the circuit current after
installation of the second heater, and what is the minimum size circuit breaker
that can be used if this is a continuous-duty circuit?

2. You are an electrician. You have been asked by a homeowner to install a lighted
mirror in a bathroom. The mirror contains eight 40-watt lamps. Upon checking
the service panel you discover that the bathroom circuit is connected to a single
120-volt, 20-ampere circuit breaker. At the present time, the circuit supplies
power to an electric wall heater rated at 1000 watts, a ceiling fan with a light kit,
and a light fixture over the mirror. The fan motor has a full-load current draw of
3.2 amperes and the light kit contains three 60-watt lamps. The homeowner
asked whether the present light fixture over the mirror can be replaced by the
lighted mirror. Assuming all loads are continuous, can the present circuit supply
the power needed to operate all the loads without overloading the circuit?
202

Problem Set No. 12

PARALLEL DC CIRCUIT

1. Find the current though each resistor and the total current drawn from the
source.

a. I1 = 0.5 A, I2 =0.25 A, IT = 0.75 A


b. I1 = 0.48 A, I2 =0.24 A, IT = 0.8 A V1 R1 R2
25ohm 50ohm
c. I1 = 0.45 A, I2 =0.21 A, IT = 0.72 A 12V
d. I1 = 0.48 A, I2 =0.24 A, IT = 0.72 A

2. Determine the voltage across the 150- resistor.

I I1 I2 R1 R2 R3 R4 R5
1A 1A 75ohm 150ohm 30ohm 50ohm 20ohm
8A

a. 45.75 V
b. 48.64 V
c. 50.36 V
d. 65.46 V

3. Three resistors A, B, and C are connected in parallel and take a total of 7.9 A.
Resistor A takes 2.5 A and has a resistance of 48 ohms; also, the current through
B is twice as much as through C. Calculate (a) IB and IC, (b) the line voltage, (c)
RB and RC .
a. IB = 3.6 A, IC = 1.8 A , VT = 120 V , RB = 33.33 Ω , RC = 66.67 Ω
b. IB = 3.5 A, IC = 1.7 A , VT = 120 V , RB = 33Ω , RC = 65 Ω
c. IB = 2.6 A, IC = 2.8 A , VT = 120 V , RB = 33.33 Ω , RC = 66.67 Ω
d. IB = 3.75 A, IC = 1.85 A , VT = 120 V , RB = 33.33 Ω , RC = 66.67 Ω

4. Two resistors RA = 1.95 ohms and RB = 0.05 ohm are connected in parallel and
take a total of 50 A. What is the current through each resistor?
a. IA = 1.26 A, IB = 48.76 A
b. IA = 1.2 A, IB = 48.7 A
c. IA = 1.35 A, IB = 48.85 A
d. IA = 1.25 A, IB = 48.75 A
203

Objective Test No. 11

PARALLEL CIRCUIT

1. A wire whose resistance is r ohms is being cut into four equal parts. If these
parts are to be connected in parallel, how much is the equivalent resistance in
ohms?
A. r/12
B. r/16
C. r/8
D. r/4

2. What resistance must be connected in parallel with a 1.0-ohm resistance to


give an equivalent resistance of 0.2 ohm?
A. 0.75 ohm
B. 0.25 ohm
C. 1.20 ohm
D. 0.50 ohm

3. Twenty resistors each having a resistance of 1000 ohms are connected in


parallel. The equivalent resistance is
A. 20,000 ohms
B. 50 ohms
C. 1,000 ohms
D. 5,000 ohms

4. Two resistors of 5 and 10 ohms respectively are connected in parallel. If the


total current to the branch is 24 A, find the current in the 5-ohm resistance?
A. 16 A
B. 15 A
C. 10 A
D. 8 A

5. Three parallel branches of 10, 20, and 30 ohms respectively, are connected
across a 60-V DC supply. How much is the total power consumed by these
resistors?
A. 600 W
B. 606 W
C. 660 W
D. 560 W
204

6. Three resistors of 100, 120, and 150 ohms are connected in parallel.
Determine the value of the current to the parallel system which will make the
current in the 150-ohm resistance equal to 1.0 A.
A. 4.00 A
B. 5.25 A
C. 3.15 A
D. 3.75 A

7. Two loads with equal resistances are connected in parallel across a certain
supply. If these loads are reconnected in series across the same supply, then
________.
A. the power drawn by each will be decreased by 100%
B. the power drawn by each will be decreased by 75%
C. the power drawn by each will be decreased by 25%
D. the power drawn by each will be decreased by 50%

8. If three equal resistances are connected in parallel, the equivalent resistance of


the combination will be
A. three times the value of one resistor
B. half the value of one resistor
C. one-third the value of one resistor
D. none of these

9. An 8-ohm resistance and a 10-ohm resistance are connected in parallel. If the


total current is 9 A, what is the current in the 8-ohm resistor?
A. 6 A
B. 5 A
C. 4 A
D. 3 A

10. In parallel circuit, the voltage across each branch is ____ the source voltage.
A. lesser than
B. greater than
C. equal to
D. all of these (dependent on size of load)

11. In a parallel circuit, the total resistance is ______.


A. the sum of all the resistances
B. the reciprocal of all the resistances
C. larger than the largest resistances in the combination
D. smaller than the smallest resistance in the combination
205

Unit 8

DC MOTOR

E.
LEARNING
F. OUTCOMES

After completing this


unit, you are expected to:

1. describe a motor.
2. differentiate motor to a generator.
3. discuss how a motor works.
4. Identify the parts of a DC motor and discuss the function of
each part.
5. identify the types of a DC motor.
6. use the principles learned to solve problems about DC
motor.
206

8.1 Motor

Motor is a machine that converts electrical energy to mechanical energy. It is


opposite of a generator.

8.2 The Basic Parts of a DC Motor

(poles and field


windings)

a. Yoke or Frame. It is cylindrical in form to which an even number of poles are


bolted. It is either made from cast iron or cast steel.
b. Armature core and winding. The core is cylindrical in form made from sheet
steel laminations with slots that carry the armature windings.
c. Poles and Field windings. They are used to generate magnetic lines of flux.
207

d. Commutator. It is cylindrical in shape and consists of hard drawn copper


conductors insulated from each other, it is also called a mechanical rectifier.
e. Brushes. Used to connected the external circuit load to the armature. They
are made from carbon particles and are held in position by spring pressures.

8.3 Types of DC Motors According to Connections:

a. Shunt Motor. The armature and the field windings are connected in parallel.

DC_MOTOR_ARMATURE

Is
Ra
250
1mH
V1 V +
Vs Shunt
Armature
L2 field Eb Rsh
A
S1
- Ia
Ish

This machine is used where almost a constant speed is required. Examples, in lathe
machines, wood working machines and other machine tools.

b. Series motor. The armature and the field windings are connected in series.

DC_MOTOR_ARMATURE
Ise

Is
Rse Ra
+
Vs Eb
A

S1 field
Shunt
250 V
1mHV1
Armature - Ia
L1
208

This machine is used where (a) the load suddenly comes and goes after some time,
(b) where constant speed is not essential. Examples, in punching machines,
presses, power hammers, lifting machines, etc.

c. Long shunt compound motor. The series field winding is connected in series
with the armature winding while the shunt field winding is connected across
the supply voltage.

DC_MOTOR_ARMATURE
Ise

Is
RaRse
+
Vs Rsh Eb

A
V1
S1
1mH
Shunt
L2field
250 V
Armature
1mH - Ia
Ish L1

d. Short shunt compound motor. The series field winding is connected in series
with the supply voltage while the shunt field winding is connected across the
armature winding.

DC_MOTOR_ARMATURE
Ise

Is
Rse
Ra +
Vs V1 field Rsh
L1
Shunt Eb
A

1mH
1mH
S1
Armature
250L2
V Ia
-
Ish

Ra = resistance of the armature winding


Rsh = resistance of the shunt field winding
Rse = resistance of the series field winding
Ia = armature current
209

Ish = shunt field current


Is = current drawn by the motor from the supply
Eb = back emf or counter emf
Vs = supply voltage

Counter or back emf. The voltage induced into the armature conductors of a DC
motor when the armature rotates.

8.4 Speed-Torque Characteristics of a DC Motor:

a. The speed (N) of a DC motor is directly proportional to the back emf (Eb)
and inversely as the flux () generated per pole.

Eb
N = kn

b. The torque (T) exerted by the DC motor is directly proportional to both the
armature current (Ia) drawn and the flux () generated per pole.

T = k t Ia 

8.5 Horsepower rating of a DC motor.

The maximum safest mechanical power it can deliver to the load.

2πNT
HP =
k

where:
HP = mechanical power in horsepower
N = speed of the motor in rpm
T = torque exerted by the motor
k = proportionality constant
= 44,760 if T is in newton-meter
= 33,000 if T is in pound-foot
QUESTIONS:

1. What is a motor?
2. What is the difference between a motor and a generator?
3. How does a motor works?
4. What are the parts of a DC motor? Discuss the function of each part.
5. What are the three types of a DC motor?
210

Practical Application No. 6

Name: Ybañez, Eric L. Score: _________ Rating: ______

1. A direct current motor is connected to a 250-volt DC supply. The armature has a


current draw of 165 amperes when operating at full load. You have been
assigned the task of connecting two resistors in the armature to provide speed
control for the motor. When both resistors are connected in the circuit, the
armature current is to be limited to 50% of the full-load current draw. When
only one resistor is connected in the circuit, the armature current is to be limited
to 85% of full-load current. Determine the ohmic value and minimum power
rating of each resistor. When both switches S1 and S2 are open (off) both
resistors are connected in the armature circuit, limiting current to 50% of its
normal value. When switch S1 is closed, it causes the current to bypass resistor
1. Resistor 2 now limits the current to 85% of the full-load current. When both
switches S1 and S2 are closed, all resistance is bypassed, and the armature is
connected to full power.

Key Key
11mΩ = L1
J3
=11mΩ
1mH
Space
1mH
R1
J2Space
DC_MOTOR_ARMATURE
R2 L2V1
S1

Resistor 1 Resistor 2 Armature Series field


A

S1 S2

Shunt field

250 V
211

Assessment No. 15

DC MOTOR

Name: Ybañez, Eric L. Score: _________ Rating: ______

1. A 220-V shunt motor has an armature resistance of 0.26 ohm. Calculate the
counter emf when the armature current is 35.4 amp.

2. A 240-V shunt motor has an armature resistance of 0.38 ohm. What current will
flow in the armature when the counter emf is 227.5 volts?

3. A 5-hp 230-V long-shunt compound motor takes 19 amp when operating at full
load. The shunt-field resistance is 115 ohms , the series field resistance is 0.20
ohms, and the armature resistance is 0.25 ohm. Calculate the value of the
counter emf.
212

Unit 9

COMBINATION CIRCUITS

G.
LEARNING
H. OUTCOMES

After completing this


unit, you are expected to:

1. identify series and parallel portions of a series-parallel


circuit and recognize the relationships of all resistors.
2. determine the total resistance of a series-parallel circuit.
3. determine the currents and voltages in a series-parallel
circuit.
4. transform delta to wye connection, vice versa.
213

9.1 Series- Parallel Circuit

R1

75ohm
R2 Rt
75ohm R3 125ohm
150ohm

R2 R3
RT = R1 + R2 + R3

(75)(150)
= 75 + 75 + 150

= 125 

9.2 Parallel-Series Circuit:

R1
75ohm
Rt
R3
75ohm
150ohm
R2
75ohm

R3 (R1 + R2 )
RT = R1 + R2 + R3

150(75 + 75)
= 75+ 75+ 150

RT = 75 

Example 9.1 From the circuit below, determine (a) the total resistance, (b) the
voltage between points a and b, (c) the total current taken by the circuit, (d) the
current in the 8- resistor.
214

R1
a
2ohm R2
4ohm
E R4
8ohm
12V
R3
6kohm

R5

10ohm
b

Solution:

a. The resistance between points a and b is

(4+6)(8)
Rab = 4+ 6+ 8

Rab = 4.44

R1
a
2ohm

E
Rab
12V 4.44ohm

R5

10ohm
b

The total resistance is

RT = R1 + Rab + R5

= 2 + 4.44 + 10

= 16.44 

b. The voltage between points a and b can be calculated using voltage divider
principle.

12(4.44)
Vab = = 3.24 V
16.44
215

c. For the total current

12 V
IT = = 729.93 mA
16.44.

d. Since the 8- resistor is connected across nodes a and b, its voltage is also
equal to Vab = 3.24 V. The current through it is

3.24 V
I8 = = 405 mA
8Ω

Example 9.2 Three resistors R2 = 12  and R3 = 6  are connected in parallel


and the combination is connected in series with R1 = 3  . if the source voltage is
100 volts, calculate:

a) The total resistance RT.


b) The total current IT.
c) The voltage across R2.
d) The current that passes through R2

Solution:

R1
-
14.286 A
3ohm
+
R2 R3
+
12ohm 6ohm 57.143 V
-
VT
100V
+
4.762 A
-

a. The total resistance RT.

R2 R3
RT = R1 + R2 + R3

(12)(6)
= 3 + 12+ 6
216

RT = 7 

b. The total current IT

VT 100 V
IT = =
RT 7Ω

IT = 14.29 A

c. The voltage across R2

The equivalent resistance across a and b is:


R2 R3
Rab = R2 + R3

(12)(6)
= 12+ 6

Rab = 4 

Using voltage divider:

V Rab (100 V)(4 Ω)


Vab = =
R1 + Rab 3Ω+4Ω

Vab = 57.14 V

Since R2 is connected across a and b, thus, V2 = Vab = 57.14 V

d. The current through the 12- resistor is:

Vab 57.14 V
I12- = = = 4.75 A
R2 12 Ω

or using current divider

R3
I12Ω = IT (R )
2 + R3

6
= 14.29 (12+ 6)
I2 = I12- = 4.75 A
217

Example 9.3 Four bulbs each rated 110 V, 60 W are connected in parallel to
a 230-V source. Calculate the resistance to be connected in series with the line
so that the voltage across the bulbs does not exceed 110 volts.

Solution:

- Rs
2.182 A
+ 55ohm

V L1 L2 L3 L4
230V 60W 110V 60W 110V 60W 110V 60W 110V

The resistance of each bulb:

V2rating 1102
Rbulb = = = 202 
P 60

For the total resistance of the four bulbs

RT = (4 x 202-1)-1 = 50.5 

The total current is

V 110 V
I = = = 2.18 A
RT 50.5 Ω

VR series = 230 - 110 = 120 volt

VRseries 120 V
Rseries = =
I 2.18 A

Rseries = 55 

9.3 Bridge Circuit

9.3.1 The Wheatstone Bridge


218

The bridge circuit is widely used in measurement devices and other applications.
We will consider the balanced bridge, which can be used to measure unknown
resistance values. This circuit, shown in Figure 10.1(a) is known as Wheatstone
bridge. Usually a Wheatstone bridge is drawn in similar manner like in Figure
10.1 (b). Resistors R1 and R3 have fixed values, R2 is a variable resistor and Rx is
unknown.
A bridge is said to be balanced when the voltage (Vout) across the
output terminals A and B is zero; that is, VAC = VAB and also VCD = VBD . To find
the unknown resistance Rx ,
Rx R1 R
= so that, R x = R 3 x R1
R3 R2 2

Note: In laboratory experiments R2 is adjusted until Vout becomes zero.

R1
Rx

1.0kΩ 2 12 V C
11.0kΩ V out B
E 1.0kΩ 3 1.0kΩ
4

R2 R3

(a) (b)

Figure 10.1. The Wheatstone Bridge

Example 9.4 If in figure10.1 R1 is 25 Ω , R3 is 20 Ω , R2 is adjusted to 8.93


Ω, find the value of Rx.

Solution:
R1
Rx = R3x
R2
200
= 100 x 350

R x = 57.14 Ω
219

Assessment No. 16

COMBINATION CIRCUITS

Name: Ybañez, Eric L. Score: _________ Rating: ______

1. Two resistances of 5 and 10 ohms respectively, are connected in series with each
other. If a 20-ohm resistance is connected across the series combination, how
much is the total current drawn by the whole circuit if connected across a 120-V
source?

2. Electric resistances of 7 and 11 ohms are connected in parallel. This


combination is then placed in series with a single resistance of 15 ohms and the
entire combination is placed across a 110-V DC mains, what current passes
through the 15-ohm resistance?
220

Problem Set No. 13

COMBINATION DC CIRCUIT

1. Find Req of the network shown below.


R2 R5 R8

100ohm 100ohm 100ohm


R1 R4 R7 R10 R11
Req 100ohm 100ohm 100ohm 100ohm 100ohm

R3 R6 R9

100ohm 100ohm 100ohm

A. 83.2 Ω
B. 73.2 Ω
C. 63.2 Ω
D. 53.2 Ω

2. Fill in the ammeters and voltmeters with their correct reading.

R1

10ohm R2
-
1.090 A 15ohm R5 +
+ 19.104 V
50ohm -

E + R3 R4
30V 8.490 V
20ohm 30ohm +
-
0.382 A
-

A. IT = 0.09 A, I5 = 0.2822 A, V5 = 19.11 V, V3 = 0.4247 A


B. IT = 1.08 A, I5 = 0.2822 A, V5 = 19.11 V, V3 = 0.6247 A
C. IT = 1.09 A, I5 = 0.3822 A, V5 = 19.11 V, V3 = 0.4247 A
D. IT = 1.02 A, I5 = 0.2822 A, V5 = 19.15 V, V3 = 0.4347 A
221

3. A 12-ohm resistor is connected in parallel with a series combination of resistors


of 8 and 16 ohms. If the drop across the 8-ohm resistor is 48 volts, determine
the total impressed emf and the total current.
A. 146 V, 15 A
B. 144 V, 12 A
C. 144 V, 18 A
D. 148 V, 14 A

4. In the circuit below, determine (a) the total resistance, (b) the total current, (c)
the current through 60-ohm resistor.

R1

10ohm
R2
30ohm
E
R3
15V 60ohm

R4
75ohm

R5

20ohm

A. 28.18 Ω, 0.22 A, 0.14 A


B. 48.18 Ω, 0.22 A, 0.14 A
C. 58.18 Ω, 0.28 A, 0.14 A
D. 68.18 Ω, 0.22 A, 0.14 A

5. Three 12-V, 25-W incandescent bulbs are connected in parallel. What is the
value of resistor that must be connected in series with the parallel bulbs so that
they can operate in a 36-V battery.

1kohm

E L1 L2 X3
36V 25W 12V 25W 12V 25W 12V

A. 1.84 Ω
B. 2.84 Ω
C. 3.74 Ω
D. 3.84 Ω
222

Objective Test No. 12

COMBINATION CIRCUIT

1. Three 120-ohm resistors are connected in series-parallel. The equivalent


resistance of the combination is __________.
A. 360 ohms
B. 80 ohms
C. 180 ohms
D. 40 ohms

2. A resistor of 3 ohms is connected in parallel with a 2-ohm resistor. If the


combination is connected in series with a 4-ohm resistor, what is the
equivalent resistance of the whole combination of three resistors?
A. 6.4 ohms
B. 5.8 ohms
C. 4.5 ohms
D. 5.2 ohms

3. A resistor of 4-ohm resistance is connected in parallel with a series


combination of two resistors; 3 Ω and 1 Ω. What is the equivalent resistance of
the whole combination?
A. 8 ohms
B. 3 ohms
C. 5 ohms
D. none of these

4. Two resistances of 5 and 10 ohms respectively, are connected in series with


each other. If a 20-ohm resistance is connected across the series combination,
how much is the total current drawn by the whole circuit if connected across a
120-V source?
A. 11.25 A
B. 10.5 A
C. 12 A
D. 14 A

5. Three resistor are to be connected in four possible type of circuit connections


namely; series, parallel, series-parallel and parallel-series. Which type of
connection will give the least amount of equivalent resistance?
A. series
B. parallel
C. series-parallel
D. parallel-series
223

6. Electric resistances of 7 and 11 ohms are connected in parallel. This


combination is then placed in series with a single resistance of 15 ohms and the
entire combination is placed across a 110-V DC mains, what current passes
through the 15-ohm resistance?
A. 5.71 A
B. 3.17 A
C. 4.27 A
D. 6.77 A
224

9.4 Delta and Wye Connections

Delta-Wye Transformation

1
1

R1

RC RA
R3

R2

3 2
RB 3 2

Formulas

RA RC
R1 =
RA + RB + RC

RA RB
R2 = RA + RB + RC

RB RC
R3 = RA + RB + RC

Wye-Delta Transformation

R1

RC RA
R3

R2

3 2
3 2
RB
225

Formulas

R1 R2 + R2 R3 + R3 R1
RA = R3

R1 R2 + R2 R3 + R3 R1
RB = R1

R1 R2 + R2 R3 + R3 R1
RC = R2

Reminder: A delta and wye connection need not to be -shape and Y-shape
respectively. They could be drawn like shown in figures below. Figure a is also
called -network and Figure b is called a T-network.

R1 R2
R1
1 2
1 2
30ohm 150ohm
30ohm

R2 R3 R3
150ohm 25ohm 25ohm

3
3

(a) (b)
226

Assessment No. 17

DELTA TO WYE AND WYE TO DELTA TRANFORMATION

Name: Ybañez, Eric L. Score: _________ Rating: ______

1. Find the total resistance between points a and b of using (a) wye to delta
transformation; (b) delta to wye transformation.

R1 R3
100ohm 100ohm

E R2

12V
100ohm

R4 R5
100ohm 100ohm

b
227

Problem Set No. 14

DELTA-WYE TRANSFORMATION

1. Find the total resistance between points a and b of each of the network below.

R1

100ohm

R2 R3
a
100ohm 100ohm

E R4
100ohm R5
12V
100ohm

A. 100 Ω
B. 50 Ω
C. 75 Ω
D. 25 Ω

2. Find the total resistance between points a and b of each of the network
below.

a
A. 0.2348R Ω R
B. 0.5833R Ω
C. 1.2984R Ω
R R
D. 1.5243R Ω R
R b
R

R
R
R

R
228

Unit 10

NETWORK RULES AND THEOREMS

I.
LEARNING
J. OUTCOMES

After completing this


unit, you are expected to:

1. apply the superposition method to evaluate circuits with


more than one source.
2. apply Thevenin’s theorem to simplify complex circuits.
3. describe the difference between Thevenin’s and Norton’s
theorem.
4. solve voltages and currents in a circuits using other
methods such as nodal analysis, mesh analysis, and
Millman’s theorem.
229

10.1 Maxwell’s (Mesh) Method

Algebraic technique or procedure is employed for writing the simultaneous


equations in terms of mesh currents in the electrical network.

A mesh is a loop which does not contain any other loops within it.

In the circuit below there are four (4) loops:

R1

1ohm
R3 V
1ohm
RL
50ohm 45V

R4 R2

4ohm 4ohm
R5 R7
4ohm 4ohm

R6

4ohm

Example 10.1 For a given shown calculate the current drawn by RL = 50 ohms
using Maxwell’s mesh method.

R1 4ohm R2 2ohm

V1 V2
RL
8V 50ohm 12V
mesh 1 mesh 2
IA IB

Note: Always assign a counterclockwise direction of loop (or mesh).

Applying KVL at mesh 1

8 – 4IA - 50IA + 50IB = 0


-54IA + 50IB = -8
230

54IA - 50IB = 8
Also, apply KVL at mesh

-12 - 50IB + 50IA - 2IB = 0

50IA - 52IB = 12

Use determinant method to solve the equations,

54IA - 50IB = 8 equation 1


50IA - 52IB = 12 equation 2

54 −50
D=[ ] = -308
50 −52
8 −50
[ ]
12 −52
IA = = -0.5974 A
−308

54 8
[ ]
50 12
IB = = -0.80519 A
−308

To find for IL

IL = IA - IB = -0.5974 - (-0.80519) = -0.20779 A or 207.79 mA

Example 10.2 For a given circuit shown calculate the current drawn by RL =
50  using Maxwell’s mesh method.

R1

1ohm

I RL
V
2A 50ohm
45V

R2

4ohm

Solution:
R1

1ohm

I RL
m1 m2 V
2A 50ohm
IA IB 45V

R2

4ohm
231

We cannot apply KVL at mesh 1(m1) because there is a current source. KVL is for
voltages (source or drop) only. Since IA passes through a current of 2 A with the
same direction IA is automatically 2 A.

For loop 2

-45 – 4IB – 50IB + 50IA - 1IB = 0

50IA - 55IB = 45

But IA = 2 A, thus

50(2) - 55IB = 45

from which

IB = 1 A

Therefore IL = IA - IB = 2 – 1 = 1 A

Example 10.3 Use mesh analysis to determine the three unknown mesh currents in
the circuit shown.

R1 R2
1ohm 2ohm
m1
R3 IA
V1
3ohm
7V V2

6V R5
m2
m2 1ohm
m3
R4
IIBB 2ohm
IC

At mesh 1 (m1)

-1IA + 1IB - 2IA - 3IA + 3IC = 0

-6IA + 1IB + 3IC = 0 equation 1

At mesh 2
232

7 - 1IB + 1IA - 6 - 2IB + 2IC = 0

1IA - 3IB + 2IC = -1 equation 2

At mesh 3

6 - 3IC + 3IA - 1Ic - 2IC + 2IB = 0

3IA + 2IB - 6IC = -6 equation 3

Using determinants method to find the values of IA, IB, and IC:

-6IA + 1IB + 3IC = 0

1IA - 3IB + 2IC = -1

3IA + 2IB - 6IC = -6

For the denominator

−5 1 3
D = [ 1 −3 2 ] = −39
3 2 −6

Solving for IA
0 1 3
[ −1 −3 2]
−6 2 −6 −78
IA = = −39 = 2 A
−39

Solving for IB
−6 0 3
[ 1 −1 2]
3 −6 −6 −117
IB = = =3A
−39 −39

and for IC
−6 1 0
[ 1 −3 −1]
3 2 −6 −117
IB = = = 3A
−39 −39
233

Example 10.4 Find the value of Vx using Mesh analysis.

+ VX -

Solution:

+ VX -

m1 m2 m3

IA IB IC

Combining meshes 1 and 2 to form a supermesh. By applying KVL at supermesh 1


and 2,

24 − 10IA − 20IB − 30IB + 30IC = 0

24 − 10IA − 50IB + 30IC = 0

where IA = IB − 2

24 − 10(IB − 2) − 50IB + 30IC = 0

−60IB + 30IC = −44 equation 1


KVL at mesh 3

−30IC + 30IB − 45IC − 48 = 0

30IB − 75IC = 48 equation 2


Equating (1) and (2)

−60IB + 30IC = −44


30IB − 75IC = 48
Therefore, IB = 0.5167 A and VX = 10.33 V
234

Assessment No. 18

MESH ANALYSIS

Name: Ybañez, Eric L. Score: _________ Rating: ______

1. Find the mesh currents for the circuit shown.

R1 R2
1Ω 2Ω

R3
E
7V 41
32 3Ω
5
I
7A R5

R4

235

2. Determine the current through RL using mesh analysis.

R1 R2
2ohm 2ohm
V1

7V
RL

3ohm
R4 R5
2ohm 2ohm

.
236

Problem Set No. 15

MESH ANALYSIS

1. Determine the current through RL using mesh analysis.

+
V1
0.1 V1
-

A. 400 mA
B. 333 mA
C. 667 mA
D. 800 mA

2. Use mesh analysis to find the current through R6 in the circuit shown.
I

R1 R3 8A

8ohm 2ohm R5 10ohm


R2 R4 R6
V 4ohm 3ohm 5ohm

100V

A. 3.38 A
B. 4.04 A
C. 5.38 A
D. 6.05 A
237

3. Find the voltage V using Mesh analysis.

A. 82.5 V
B. 84 V
C. 86.5 V
D. 88 V

V1
R2

10Ω
60 V
+
I R1 R3 I1
20Ω V 30Ω 0.4A/A
0.4 I1
4A
_

I1
238

10.2 Nodal Analysis

In this method, a solution is possible with n – 1 equations where n represents the


number of nodes.

Example 10.5 Find the current though each resistor using nodal analysis.

R1

10ohm

V R2 RL
20ohm 30ohm
12V

Step 1: Label each node (in letters or in numbers)

R1
1 2
10ohm

V R2 RL
20ohm 30ohm
12V

Step 2: Assign a reference node; usually the node with has the highest number of
branches. But for simplicity, the bottom node is assigned as the reference node. The
reference node is the ground or negative polarity. In given circuit node 4 is the
reference, hence it is negative.

Step 3: Assign a node to reference node voltage. In the circuit, V1 is the voltage
between nodes 1 and 4(the reference node); V2 is the voltage between nodes 2 and 4;
V3 is the voltage between node 3 and 4.

R1
V1 V2 V3
+ 10ohm
+ +

V R2 R3
12V 20ohm 30ohm

_
4
239

Step 4: Assume directions of current through each resistor. For a resistor (or any
component) connected to the reference node, their directions are sure to be going to
the reference node as in the case of R2 and R3.

R1 I1
V1 V2 V3
+ 10ohm
+ +

V R2 R3
12V 20ohm 30ohm

I2 I3
_
4

Step 5: Apply KCL in each node (except the reference node). Our aim here is to be
find the values of V1, V2, and V3.

 No need to apply KCL at node 1 because 12-V source is connected across


nodes 1 and 4 and its polarity is the same as V1(which is the voltage
between nodes 1 and 4), therefore,

V1 = 12 V

 Applying KCL at node 2

I1 - I2 - I3 = 0

V1 − V2
where: I1 = 10

Note: Since the direction of current through R1 goes from node 1 to node 2, the
voltage across it is V1 – V2.
V
I2 = 202

V2
I3 = 30

Thus,

(V1 − V2 ) V2 V2
− − =0
10 20 30

where V1 = 12 V;
240

(12− V2 ) V2 V2
− − =0
10 20 30

11V2 12
− + =0
60 10

V2 = 6.5455 V

V1 − V2 12−6.5455
Hence, I1 = = = 0.54545 A
10 10

V2 6.5455 V
I2 = = = 0.3273 A
20 20

V2 6.5455 V
I3 = = = 0.21818 A
30 30

Example 10.6 Find the current through RL using nodal analysis.

R1

1ohm

I RL
V
2A 50ohm
45V

R2

4ohm
Solution:

Since R1 and R2 are connected in series we can combine them and following steps 1
to 4, we have the given circuit.

R1 I1
V1 V2

5ohm

I RL V
50ohm 45V
2A
I2

3
_

By applying KCL at node 1


241

2 + I1 - I2 = 0
V2 − V1
where : I1 = ; since the current goes from node 2 to 1.
5

V1
I2 = 50
I1 - I2 = -2
V2 − V1 V1
− = -2
5 50

At node 2, A 45-V voltage source is connected across nodes 2 and 3 and its polarity
is the same as V1 so,

V2 = 45 V

From the equation


V2 − V1 V1
− = -2
5 50

45 − V1 V1
− = -2
5 50

V1 = 50 V
V1 50 V
I2 = IL = = = 1A
50 50 

Example 10.7 Find the current through 200- resistor.

Solution:
242

supernode
V1 V2

I200 I50

Reference node

In this case nodes 1 and 2 are combined to form one node. This is called a
supernode.

By applying KCL at supernode 1 and 2,

1 − I200 − I50 − 0.5 = 0

V1 V2
where I200 = , I50 =
200 50

V 1 V2
1 − 200 − − 0.5 = 0
50

V1 + 4V2 = 100 equation 1

.At supernode 1 and 2:

−V1 + V2 = 100 equation 2

Equating 1 and 2

V1 + 4V2 = 100
−V1 + V2 = 100

V2 = 40 V
𝐕𝟏 = −𝟔𝟎 𝐕

Therefore,
−𝟔𝟎
𝐈𝟐𝟎𝟎 = = −𝟎. 𝟑 𝐀
𝟐𝟎𝟎
243

Assessment No. 19

NODAL ANALYSIS

Name: Ybañez, Eric L. Score: _________ Rating: ______

1. Find the current through the 8- resistor using Nodal analysis.
R3
R1 8ohm R5

5ohm 10ohm
V2
V1 R2 R4
20ohm 40ohm 50V
100V
244

2. Determine the current through R3 using Nodal analysis.


R1 R2

2kohm 3kohm

V I R3
1kohm
40V 2A
245

Problem Set No. 16

NODAL ANALYSIS

1. Find the current, voltage, and power in RL using Nodal analysis.

R1 R2
1ohm 2ohm
RL
50ohm
V2
V1
12V 24V

A. 0.216 A, 16.79 V, 3.63 W


B. 0.316 A, 19.79 V, 6.25 W
C. 0.316 A, 15.79 V, 4.99 W
D. 0.716 A, 10.79 V, 7.73 W

2. Find the current, voltage, and power in R3 using Nodal analysis.


R1 R3

6ohm 4ohm
R2 V2
V1 3ohm 10V
42V

A. 1 A, 12 V, 12 W
B. 3 A, 6 V, 18 W
C. 4 A, 5 V, 20 W
D. 2 A, 6 V, 12 W

3. Find the current, voltage, and power in R3 using Nodal analysis.


246

R2

5ohm
R1 R3
I1 2ohm 1ohm I2
3.1A
-1.4A

A. 950 mA
B. 860 mA
C. 750 mA
D. 650 mA

4. Find the voltage V using Nodal analysis.


A. 72.5 V
B. 78.25 V
C. 80.25 V
D. 82.5 V

V1
R2

10Ω
60 V
+
I R1 R3 I1
20Ω V 30Ω 0.4A/A
0.4 I1
4A
_

I1
247

10.3 Thevenin’s Theorem

Thevenin’s theorem gives us a method for simplifying a circuit to a


standard equivalent form. The Thevenin’s equivalent form for any resistive circuit
consist of an equivalent voltage source (VTH) and an equivalent resistance (RTH),
arranged as shown in Figure 11.1. The values of the equivalent voltage and resistance
depend on the values in the original circuit. Any resistive circuit can be simplified
regardless of its complexity.
RTH

60ohm

VTH

12V

Figure 10.1. The general form of a Thevenin equivalent circuit. Any resistive circuit is
reduced to this form.

RTH + -
1.500 A
a
60ohm
VTH
RL
+
120V Any given
30.000 V
Circuit 20ohm -

RTH -
+
1.500 A
a
60ohm
VTH
Thevenin’s RL
+
120V equivalent 30.000 V
20ohm
circuit -

b
248

VTH
IL = RTH + RL

where:
VTH – the open-circuit voltage measured across terminals a and b with
RL removed.
RTH – the equivalent resistance with all voltage sources shorted and all
current sources opened, across terminals a and b with RL removed.
RL – load resistance.
IL – load current

Example 10.8 In the given circuit, calculate the current and the power drawn by RL
= 30  using Thevenin’s theorem.

R1

10ohm

V R2 RL
20ohm 30ohm
12V

Solution:

By connecting an ammeter in series with RL, the current reads 0.218 A.

R1

10ohm +
0.218 A
-

V R2 RL
20ohm 30ohm
12V

To determine the Thevenin’s equivalent circuit


249

Step 1. Remove the 30- load resistor and solve for VTH. VTH is the voltage across
open terminals a and b.

R1
a
10ohm

V R2
20ohm
12V
VTH

By voltage divider:

12 V (20Ω)
VTH = = 8 volts
10 Ω+20 Ω

Step 2. Solve for RTH. Short circuit the voltage source.

R1
a
10ohm

R2
20ohm
RTH

b
(10Ω) (20Ω)
RTH = = 6.67 
10 Ω+20 Ω

Step 3. Connect VTH and RTH in series across points a and b. Also reconnect RL = 30
 across terminals a and b.

RTH

6.67ohm
VTH RL

8V 30ohm

Step 4. Determine IL by using the equation

VTH
IL = R
TH + RL
250

8V
IL = = 0.218 A
6.67 Ω+30 Ω

PL = IL2 RL = (0.218)2(30) = 1.426 watts

Connecting an ammeter through the Thevenin’s equivalent circuit, its also


measures 0.218 A; the same current as in the original circuit.

RTH + -
0.218 A
6.67ohm
VTH
RL
+
8V 6.545 V
30ohm -

Example 10.9 For a given circuit shown, calculate the current and the power
drawn by RL = 50  using Thevenin’s theorem.

R1

1ohm

I RL
V
2A 50ohm
45V

R2

Solution: 4ohm
251

By connecting an ammeter in series with RL, the current reads 1 A.

R1

1ohm

I RL
V
2A 50ohm
45V

+
1.000 A
-
R2

4ohm

To determine the Thevenin’s equivalent circuit

Step 1. Remove the 50- load resistor and solve for VTH.

R1
a
1ohm

I
V
2A VTH
45V

R2

b 4ohm

By using Kirchhoff’s Voltage Law

VTH – 2(1) - 45 - 2(4) = 0

VTH = 2(1) + 45 + 2(4) = 55 V

Step 2. Solve for RTH. Short circuit the voltage source and leave open the current
source.
R1
a
1ohm

RTH

R2

b 4ohm
252

RTH = 5 

Step 3. Connect VTH and RTH in series across points a and b. Also reconnect RL = 50
 across terminals a and b.

RTH

5ohm
VTH
RL
55V
50ohm

Step 4. Determine IL by using the equation

VTH
IL = RTH + RL

55 V
IL = = 1A
5 + 50

PL = IL2 RL = (1)2(50) = 50 watts

Connecting an ammeter through the Thevenin’s equivalent circuit, its also


measures 0.218 A; the same current as in the original circuit.

RTH + -
1.000 A
5ohm
VTH
55V RL
50ohm

The previous examples are circuits containing independent variables. Let us


consider a circuit that contains a dependent variable.

Example 10.10 Find the equivalent Thevenin’s network across terminals A and B.
253

A
+

𝑉𝑥 𝑉𝑥
4000

Solution:
a
𝑉𝑥 +
4000
𝑉𝑥
𝑉𝑥
4000
_
b

By applying KVL around the outer loop,

V
x
4 + (4000) (2 x 103 ) − (3 x 103 )(0) − Vx = 0

Vx = 8 V = VOC = VTH

Now short-circuit the terminals a and b

a
+

𝑉𝑥
𝑉𝑥
4000

_
b

Solve for the short-circuit current, i.e., the current through a and b,
Vx
Since Vx = 0 the dependent current source is also equal to zero. Therefore,
4000
254

4V
ISC = = 0.8 mA
5 x 105

VOC 8V
Then, R TH = = = 10 k
ISC 0.8 mA

The equivalent network is

Example 10.11

Given:
40Ω 100Ω

E
R4
I13R2
1R1
20 V 200Ω 25 1.5 A/A 75Ω
R3

Find: the current through the 75-ohm resistor using Thevenin’s theorem.

Known:
1 Disconnect the load resistor.
2 Find open circuit voltage (VOC) across the terminals in which the load is
connected. This VOC is also the VTH.
3 Short-circuit the termoinals in which the load is connected and find the
short-circuit current (ISC).
VOC
4 To find RTH , divide VOC by ISC, i.e., R TH = ISC
.
255

5 Then , connect VTH and RTH in series and connect across them the load.
And find the current or any parameter through the load.

Solution:

1. Disconnect the load resistor.

40Ω 100Ω
A

E
200Ω 1R1
I1R2
2
5 1.5 A/A
20 V 3
R3

2. Find open circuit voltage (VOC) across the terminals in which the load is
connected. This VOC is also the VTH.

40Ω 100Ω
A

E
200Ω 1R1
I1R2
2
5 1.5 A/A
20 V 3
R3

3. Short-circuit the terminals in which the load is connected and find the
short-circuit current (ISC).
VOC
4. To find RTH , divide VOC by ISC, i.e., R TH = .
ISC
5. Then , connect VTH and RTH in series and connect across them the load.
And find the current or any parameter through the load.
256

Assessment No. 20

THEVENIN’S THEOREM

Name: Ybañez, Eric L. Score: _________ Rating: ______

1. Determine the Thevenin’s equivalent circuit across points A and B by


disconnecting the 75-ohm resistor. Find the power taken by the 75-ohm
resistor.

R1
A
5ohm

R3 R4
E 100ohm 75ohm
30V

R2
B
10ohm
257

2. Determine the Thevenin’s equivalent circuit across points A and B by


disconnecting the 75-ohm resistor. Find the voltage across the 75-ohm resistor.

R1 R5
A
5ohm 25ohm

R3 R4
E 50ohm 75ohm
30V

R2

10ohm
B
258

Problem Set No. 17

THEVENIN’S THEOREM

1. Determine the Thevenin’s equivalent circuit across points A and B by


disconnecting the 75-ohm resistor. Find the current through the 75-ohm
resistor.
R1 A R5

5ohm 25ohm

R3 V2
V1 75ohm
60V
30V

R2

10ohm
B

A. VTH = 41.25 V, RTH = 6.41 ohms, IL = 0.489 A


B. VTH = 41.25 V, RTH = 9.375 ohms, IL = 0.489 A
C. VTH = 41.25 V, RTH = 9.375 ohms, IL = 0.427 A
D. VTH = 21.25 V, RTH = 9.41 ohms, IL = 0.489 A

2. Determine the Thevenin’s equivalent circuit across points A and B by


disconnecting the 1-k resistor. Find the current through the 1-k resistor.

R1 R2
A
2kohm 3kohm

V I R3
1kohm
40V 2A

A. VTH = 4.04 kV, RTH = 5 k, IL = 0.273 A


B. VTH = 4.04 kV, RTH = 5 k, IL = 0.673 A
C. VTH = 4.04 kV, RTH = 5 ohms, IL = 0.673 A
D. VTH = 3.04 kV, RTH = 5 kilohms, IL = 0.573 A

3. Determine the Thevenin’s equivalent circuit across points A and B by


disconnecting the 75-ohm resistor. Find the voltage across the 75-ohm resistor.
259

V2
R1 R2
A
20ohm 10ohm
50V
V I R3
75ohm
100V 4A

B
A. VTH = 120 V, RTH = 30 ohms, VL = 92.86 V
B. VTH = 130 V, RTH = 20 ohms, VL = 92.86 V
C. VTH = 130 V, RTH = 30 ohms, VL = 62.86 V
D. VTH = 130 V, RTH = 30 ohms, VL = 92.86 V

4. Find the current through 6-ohm resistor .

A. 0.4667 A
B. 0.75 A
C. 1 A
D. 1.5 A

5. Determine the Thevenin’s equivalent circuit across points A and B by


disconnecting the RL. Find the voltage across RL.
260

R1 R2
1ohm 2ohm
V1

7V
RL
A B
3ohm
R4 R5
4ohm 5ohm

A. VTH = 0.4 V, RTH = 2.03 ohms, VL = 0.491 V


B. VTH = 0.5 V, RTH = 2.23 ohms, VL = 0.391 V
C. VTH = 0.6 V, RTH = 1.23 ohms, VL = 0.524 V
D. VTH = 0.6 V, RTH = 2.23 ohms, VL = 0.344 V

6. Determine the current through the 75- resistor.

R1 R2

40Ω 100Ω

E R3 R4
200Ω I3
0.1 Mho 75Ω
1.5 I1
20 V
I1

A. 0.154 A
B. 0.235 A
C. 0.783 mA
D. 1.275 mA
261

10.4 Maximum Power Transfer

Let r be the internal resistance of a battery with emf E; maximum power transferred
to the load RL occurs when RL = r.

r
1ohm
RL
1ohm
E
12V

Maximum Power:

E2 RL E2
Pmax = (2RL )2
= 4RL

Example 10.12 Find the value of R for maximum power transfer. Also
calculate the maximum power transferred.

r
1ohm
R

1ohm
E

12V

Solution:

R = r = 1 ohm

E2 (12)2
P max = 4RL
= 4(1)
= 36 watts

Another meaning of a maximum power transfer based on the principle of Thevenin’s


theorem

Maximum power transferred to the load RL occurs when RL = RTH.

VTH 2 RL VTH 2
P max = (2RTH )2
= 4RTH
262

RTH

1ohm

VTH RL

1ohm
12V

Example 10.13 For the given circuit calculate the value of RL that will result in
maximum power transferred to it. Also calculate the maximum power transferred.

R1

30ohm

R2 RL
E
150ohm 25ohm
120V

Solution:

For maximum power transfer,


RL = RTH

(30)(150)
where RTH = = 25 
30+150

Therefore, RL = 25 

Solving for VTH:

(120)(150)
VTH = = 100 V
30+150

From the equation,

VTH 2
P max = 4RTH

1002
P max = = 10,000 watts
4(25)
263

10.5 Norton’s Theorem

Any given circuit


RTH + -
1.500 A
60ohm
VTH
RL
+
120V 30.000 V
20ohm -

Norton’s Equivalent Circuit


+ -
1.500 A

I1
2A RL
RTH +
30.000 V
60ohm 20ohm -

Formula:
RN
IL = IN (R )
N + RL

where:

IN - Norton’s equivalent current; the short circuit current passing


through terminals a and b with RL removed and terminals a and b short-circuited.

RN – Norton’s equivalent resistance; the equivalent resistance with all


voltage source shorted and all current sources opened, across terminals a and b
with RL removed. This is the same as RTH in Thevenin’s theorem.

RL – load resistance; the resistance at which the voltage, current, or


power is required.

IL – load current
264

Example 10.14 In the given circuit, calculate the current and the power drawn
by RL = 30  using Norton’s theorem.

R1

10ohm

V R2 RL
20ohm 30ohm
12V

Solution:

By connecting an ammeter in series with the load, the measured current is 0.218 A.

By connecting an ammeter in series with RL, the current reads 0.218


A.
R1

10ohm +
0.218 A
-

V R2 RL
20ohm 30ohm
12V

Now, to determine the Norton’s equivalent circuit,

Step 1. Remove the 30- load resistor and solve for IN or ISC. IN is the short circuit
current passing through terminals a and b.

R1

10ohm

R2
V 10ohm
12V
IN

By using Ohm’s law


12 V
IN = = 1.2 A
10 
265

Step 2. To solve for RTH, short-circuit the voltage source. Note: RN is similar to RTH
in Thevenin’s Theorem

R1
a
10ohm
R2
20ohm RN

(10)(20)
R TH = = 6.67 
10+20

Step 3. Connect IN and RN in parallel across points a and b. Also reconnect RL = 30


 across terminals a and b.

Step 4. Determine IL by using the equation

RN
IL = IN (R )
N + RL

6.67
IL = 1.2 (6.67+ 30)

= 0.218 A

PL = IL2 RL = (0.218)2(30) = 1.426 watts

Connecting an ammeter through the Norton’s equivalent circuit, its also


measures 0.218 A; the same current as in the original circuit.
266

+ -
0.218 A

RN
IN 6.67ohm RL
1.2A 30ohm

Example 10.15 For a given circuit shown, calculate the current and the power
drawn by RL = 50  using Norton’s theorem.

R1

1ohm

I RL
V
2A 50ohm
45V

R2

4ohm
Solution:
By connecting an ammeter in series with RL, the current reads 1 A.

R1

1ohm

I RL
V
2A 50ohm
45V

+
1.000 A
-
R2

4ohm

To determine the Norton’s equivalent circuit

Step 1. Remove the 50- load resistor and solve for IN.
267

R1 I1

1ohm

I V
2A IN
45V

R2

4ohm

For the current from the 45-V source,


45 V
I1 = 5  = 9 A
IN = I1 + 2 = 9 + 2 = 11 A

Step 2. Solve for RN. Short circuit the voltage source and leave open the current
source.

R1
a
1ohm

R2
b
4ohm

RN = 5 

Step 3. Connect N and RN in parallel across points a and b. Also reconnect RL = 50


 across terminals a and b.

I RN RL
5ohm 50ohm
11A

Step 4. Determine IL by using the equation


268

RN
IL = IN (R )
N + RL

5
IL = 11 (5 + 50) = 1 A

PL = IL2 RL = (1)2(50) = 50 watts

Connecting an ammeter through the load, it measures 1 A; the same current


as in the original circuit.
+ -
1.000 A

I RN RL
5ohm 50ohm
11A
269

Assessment No. 21

NORTON’S THEOREM

Name: Ybañez, Eric L. Score: _________ Rating: ______

1. Determine the Norton’s equivalent circuit across points A and B by


disconnecting the 75-ohm resistor. Find the power taken by the 75-ohm
resistor.

R1
A
5ohm

R3 R4
E 100ohm 75ohm
30V

R2

10ohm
B
270

2. Determine the Thevenin’s equivalent circuit across points A and B by


disconnecting the 75-ohm resistor. Find the voltage across the 75-ohm
resistor.

R1 R5
A
5ohm 25ohm

R3 R4
E 50ohm 75ohm
30V

R2

10ohm
B

.
271

Problem Set No. 18

NORTON’S THEOREM

1. Determine the Norton’s equivalent circuit across points A and B by


disconnecting the 75-ohm resistor. Find the current through the 75-ohm
resistor.
R1 A R5

5ohm 25ohm

R3 V2
V1 75ohm
60V
30V

R2

10ohm
B

A. IN = 4.18 A, RN = 6.751 ohms, IL = 0.489 A


B. IN = 4.38 A, RN = 9.41 ohms, IL = 0.489 A
C. IN = 4.08 A, RN = 8.41 ohms, IL = 0.489 A
D. IN = 4.68 A, RN = 9.41 ohms, IL = 0.439 A

2. Determine the Norton’s equivalent circuit across points A and B by


disconnecting the 75-ohm resistor. Find the current through the 75-ohm
resistor.

R1 R2
A
2kohm 3kohm

V I R3
1kohm
40V 2A

A. IN = 0.408 A, RN =4 kilohms, IL = 0.673 A


B. IN = 0.808 A, RN = 5 kilohms, IL = 0.673 A
C. IN = 0.708 A, RN = 8kilohms, IL = 0.673 A
D. IN = 0.608 A, RN = 3 kilohms, IL = 0.673 A

3. Determine the Norton’s equivalent circuit across points A and B by


disconnecting the 75-ohm resistor. Find the voltage across the 75-ohm resistor.
272

V2
R1 R2
A
20ohm 10ohm
50V
V I R3
75ohm
100V 4A

B
a. IN = 5.33 A, RN = 20 ohms, VL = 92.86 V
b. IN = 3.33 A, RN = 40 ohms, VL = 92.86 V
c. IN = 2.33 A, RN = 50 ohms, VL = 92.86 V
d. IN = 4.33 A, RN = 30 ohms, VL = 92.86 V

4. Determine the current through RL using Norton’s theorem.

R1 R2
1ohm 2ohm
V1

7V
RL
A B
3ohm
R4 R5
4ohm 5ohm

A. 114.72 mA
B. 134.76 mA
C. 156.98 mA
D. 163.67 mA
273

10.6 Source Transformation

RTH
a a
2ohm
VTH IN RN
2ohm
12V
6A

b
b

A series voltage source and resistance can be transformed in parallel current source
and resistance.

RN = RTH

VTH = RN IN

VTH
IN = RTH

Reminder: Take note of the direction of current source and the polarity of the
voltage source. Consider the figures below:

V1
I1
12V 1.2A

V1 I1
12V 1.2A

Example 10.16 Transform to parallel current source and resistance

R
a a
5ohm
V R
I 5ohm
25V
5A

b b
274

25 V
IS = 5  = 5 A
Example 10.17 Find the current through R3 by source transformation.

R1

5ohm

V R2 R3
30ohm 60ohm
25V

Transform to parallel current source and resistance the 25-V source and R1.

I R1 R2 R3
5ohm 30ohm 60ohm
5A

(5)(30)
R5-30 = = 4.2857 
5+30

R3
I R5and30 60ohm
4.2857ohm
5A

By current division
5 x 4.2857
I3 = = 333.33 mA
4.2857 + 60

Example 10.18 Find the value of I in the 8- resistor.

R1 R3 R5

5ohm 8ohm 10ohm


V2
V1 50V
R2 R4
100V
20ohm 40ohm
275

Solution:

Transforming the 50-V source in series with the 10- resistor and also transforming
the 100-V source in series with the 5- resistor yields

R3 8ohm

I2
I1 R1 R2 R4 R5
5ohm 20ohm 40ohm 10ohm 5A
20A

Getting the equivalent resistance of R1 and R2 and the equivalent resistance of R4


and R5

(5)(20)
R1-2 = = 4 ohms
5 + 20

(40)(10)
R4-5 = = 8 ohms
40 + 10

R3 8ohm

I8
I2
I1 R12 R45
4ohm 8ohm 5A
20A

Transform I1 and R12 to series voltage and resistance and also I2 and R45.

VS1 = (20)(4) = 80 V
VS2 = (5)(8) = 40 V
R12 R3 8ohm R45

4ohm 8ohm Vs2


Vs1
I8 40V
80V

80−40
I8 = = 2A
4+8+8
276

Assessment No. 22

SOURCE TRANSFORMATION

Name: Ybañez, Eric L. Score: _________ Rating: ______

1. Find the current through R1 using source transformation.

R1

2ohm
V I
R2
6V 8ohm 4A
277

2. Find the current through R4 using source transformation.

V1 V2 V3
40 V 20 V 30 V
R4
50Ω
R1 R2 R3
5Ω 2Ω 3Ω
278

10.7 Superposition Theorem

In any electrical multi-source network consisting of several elements (resistors), the


voltage across or current passing through any element is equal to the algebraic sum
of the individual voltages produced in that element by each source acting
independently.

R1 R2

4ohm 2ohm
V1
RL V2
6V
I1 50ohm
I2
8V

IL

Short-circuit V2 and solve for the current due to V1

R1 R2

4ohm 2ohm
V1
RL
6V I1’ 50ohm I2’

IL’

Short-circuit V1 and solve for the currents due to V2

R1 R2

4ohm 2ohm

I1” RL I2” V2
50ohm 8V

IL”

Thus, the currents are solved as:

I1 = I1’ - I1”
I2 = I2” – I2’
IL = IL’ + IL”

Example 10.19 Solve for the current IL using superposition theorem.


279

R1 4ohm R2 2ohm

V1 V2
I1 RL I2
8V 50ohm 12V

IL

Solution:

Let IL’ be the current in RL due to 8-V source. The 12-V source is short-circuited.

R1 4ohm R2 2ohm

V1
I T’ RL
8V 50ohm

IL’

VT 8
IT’ = R R = (50)(2) = 1.35 A
R1 + L 2 4+
RL + R2 50 + 2

1.35 x 2
IL’ = = 51.9 mA
50+2

Let IL” be the current in RL due to 12-V source. The 8-V source is short-circuited.
R1 4ohm R2 2ohm

RL IT” V2
50ohm
12V
IL”

VT 12
IT’’ = R R = (50)(4) = 2.10 A
R2 + L 1 2+
RL + R1 50 + 4

2.10 x 4
IL’ = = 155.56 mA
50+4
Adding algebraically,

IL = IL’ + IL” = 51.9 + 155.56 = 207.46 mA


280

Example 10.20 Find the current through RL using superposition theorem.


R1

1ohm

I RL
V
2A 50ohm
45V

R2

4ohm
Solution:

Let IL’ be the current in RL due to the 2-A current source. The 45-V source is short-
circuited.
R1

1ohm

I RL
50ohm
2A IL’
R2

4ohm

By current division,
2x5
IL’ = = 181.818 mA
50+5

Let IL” be the current in RL due to 45-V source. The 2-A current source is open-
circuited.

Note: If the other sources are not used, the voltage sources are short-circuited; the
currents sources are open-circuited.
R1

1ohm
V
RL
50ohm 45V

IL” R2

4ohm
45
IL” = = 818.182 mA
55

Solving for IL add algebraically, add algebraically IL’ and IL”

IL = IL’ + IL” = 181.818 mA + 818.182 = 1000 mA or 1 A


281

Assessment No. 23

SUPERPOSITION THEOREM

Name: Ybañez, Eric L. Score: _________ Rating: ______

1. Determine the current through, voltage across, and power taken by the 75-
resistor using superposition theorem.

R1 R5

5ohm 25ohm

R3 V2
V1 75ohm
60V
30V

R2

10ohm
282

2. Determine the current through, voltage across, and power taken by the 1-k
resistor using superposition theorem.

R1 R2

2kohm 3kohm

V I R3
1kohm
40V 2A

.
283

Problem Set No. 19

SUPERPOSITION THEOREM

1. Determine the current through, voltage across, and power taken by the 75-
resistor using superposition theorem.
V2
R1 R2

20ohm 10ohm
50V
V I R3
75ohm
100V 4A

A. 5.238 A, 92.857 V, 486.38 W


B. 3.238 A, 92.887 V, 300.77 W
C. 1.238 A, 92.857 V, 114.96 W
D. 0.238 A, 92.897 V, 22.11 W
284

10.8 Millman’s Theorem

Millman’s theorem is applicable to parallel voltage sources with given internal


resistances connected across a given load such as shown below, or to any circuit
similar to the similar to this.

R1 R2
1ohm 2ohm
RL
50ohm
V2
V1
12V 24V

(Let R1 be the internal resistance of V1 and R2 for V2)

Formula

 I’s summation of currents


Vab = =
 G’s summation of conductances

For the circuit shown above

V1 V2
+
R1 R2
Vab = 1 1 1
+ +
R1 R2 R3

If the polarity of V2 is reversed

R1 R2
1ohm 2ohm
RL
50ohm

V1 V2
12V 24V

Vab becomes,
285

V1 V
− 2
R1 R2
Vab = 1 1 1
+ +
R1 R2 R3

Example 10.21 Find the current, voltage, and power for the circuit shown.
a

R1 R2
1ohm 2ohm
RL
50ohm
V2
V1
12V 24V

b
Solution:
V1 V2 12 24
+ +
R1 R2 1 2
Vab = 1 1 1 = 1 1 1 = 15.789 V
+ + + +
R1 R2 R3 1 2 50

The voltage across the 50-ohm resistor is 15.789 V,


a

Vab RL
50ohm
15.789V

Vab 15.789 V
IL = = = 0.31578 A
RL 50 

Example 10.22 Determine the current through RL

R1 R2
1ohm 2ohm
RL
50ohm

V1 V2
12V 24V

b
286

Solution:

V1 V
− 2
R1 R2
Vab = 1 1 1
+ +
R1 R2 R3

12 24

Vab = 11 1 1
2
= -7.8947 V
+ +
1 2 50

Since Vab is negative, its polarity must be reversed. The voltage


across the 50-ohm resistor
is,
a

RL
Vab
50ohm
7.8947V

Vab 7.8947 V
IL = = = 0.15789 A or 157.89 mA
RL 50 
287

Assessment No. 24

MILLMAN’S THEOREM

Name: Ybañez, Eric L. Score: _________ Rating: ______

1. Determine the current through RL by using Millman’s theorem

R1

10ohm

V R2 RL
20ohm 30ohm
12V
288

2. Determine the current through, voltage across, and power taken by the 75-
resistor using Millman’s theorem.

R1 R5

5ohm 25ohm

R3 R4
E 50ohm 75ohm
30V

R2

10ohm

.
289

Problem Set No. 20

MILLMAN’S THEOREM

1. Determine the current through, voltage across, and power taken by the 75-
resistor using Millman’s theorem.

A. 0.6393 A, 47.95 V, 30.65 W


B. 0.6393 A, 47.95 V, 30.65 W
C. 0.6393 A, 47.95 V, 30.65 W
D. 0.6393 A, 47.95 V, 30.65 W

R1 R5

5ohm 25ohm

R3 V2
V1 75ohm
60V
30V

R2

10ohm

2. Determine the current through, voltage across, and power taken by the 1-k
resistor using Millman’s theorem.

R1 R2

2kohm 3kohm

V I R3
1kohm
40V 2A

A. 0.6533 A, 662.3 V, 432.68 W


B. 0.6133 A, 679.3 V, 416.61 W
C. 0.6733 A, 673.3 V, 453.33 W
D. 0.2733 A, 573.3 V, 156.68 W
290

Unit 11

CAPACITOR AND CAPACITANCE

K.
LEARNING
L. OUTCOMES

After completing this


unit, you are expected to:

1. describe the basic construction of a capacitor.


2. define capacitance and tell how it is measured.
3. explain how a capacitor stores energy.
4. state Coulomb’s law and discuss how it relates to an
electric field and the storage of energy.
5. illustrate the charging and discharging of a capacitor.
6. relate various physical parameters to capacitance value.
7. determine total series capacitance.
8. determine total parallel capacitance.
9. define time constant as related to a capacitive circuit.
10. relate the charging and discharging of a capacitor to the
time constant.
11. explain why a capacitor blocks dc.
12. explain why a capacitor produces no energy loss.
13. describe some common capacitor applications.
14. check out a capacitor with an ohmmeter.
291

Important Terms

dielectric voltage rating


charging dielectric strength
discharging temperature coefficient
capacitance leakage
farad dielectric constant
Coulomb’s law time constant
energy storage permittivity

11.1 Basic Construction

In its simplest form, a capacitor is an electrical device constructed of two parallel


conductive plates separated by an insulating material called the dielectric.
Dielectric

Connecting leads

Conductive plates

Figure 11.1. The Basic Capacitor

11.2 How Does a Capacitor Stores Charges?

In the neutral state, both plates of a capacitor have an equal number of free electrons,
as indicated in Figure 11.2. When the capacitor is connected to a voltage source
through a resistor, electrons (negative charge) are removed from plate A, and an
equal number are deposited on plate B. As plate A loses electrons , plate B gains
electrons, plate A becomes positive with respect to plate B. During the charging
process, electrons flow only through the connecting leads and the source. No
electrons flow through the dielectric of the capacitor because it is an insulator. The
movement of electrons ceases when the voltage across the capacitor equals the
292

source voltage. If the capacitor is disconnected from the source retains the stored
charge for a long period of time (the length depends upon the type of capacitor) and
still has the voltage across it. Actually charged capacitor can be considered as a
temporary battery.

Figure 11.2. Illustration of a capacitor storing charge.

11.3 Capacitance

The amount of charge per unit of voltage that a capacitor can store is its capacitance,
designated C. That is, capacitance is a measure of capacitor’s ability to store
charges. The more charge per unit of voltage that a capacitor can store, the greater
its capacitance, as expressed by the following formula:

Q
C =V

where C is the capacitance, Q is charge, ad V is voltage

The Unit of Capacitance


293

The farad (F) is the basic unit of capacitance. By definition,

One farad is the amount of capacitance when one coulomb of charge is stored
with one volt across the plates.

Most capacitors that you will use in electronics work have capacitance values in
microfarads (F) and picrofarads (pF).

Example 11.1 What is the capacitance of that stores a charge of 20 C at a voltage


of 50 V?

Solution:
Q 20 C
C =V = = 0.4 F
50 V

Example 11.2 What is the voltage across a 20-F capacitor that is charged to 4000
C?

Solution:

Q 4000 C
V=C = = 200 V
20 F

11.4 How a Capacitor Stores Energy

A capacitor stores energy in the form of an electric field that is established by the
opposite charges on the two plates. The electric field is represented by lines of force
between the positive and negative charges and concentrated within the dielectric.

Lines of force
Q1 Q2

An electric field exists between the plates of a charged capacitor.


294

Q1 F Q2

A force exists between a charged bodies.

11.5 Coulomb’s law

A force exists between two charged bodies that is directly proportional to the
product of the twp charges and inversely proportional to the square of the distance
between the bodies.

This relationship is expressed as

kQ1 Q2
F = d2

where F is the force in newtons, Q1 and Q2 are the charges in coulombs, d is the
distance between the charges in meters, and k is a proportionality constant equal to
9 x 109.

.
11.6 The Energy Stored in a Capacitor

The formula for the energy stored by a capacitor is as follows:

W = ½ CV 2

where the energy, W, is in joules when C is in farads and V is in volts.

Example 11.3 What is the energy stored in a 2.0-µC capacitor with a 50 V across it?

Solution:
W = ½ CV 2
= ½ (2.0 x10-6)(50)2
= 2.5 mJ

Example 11.4 The energy stored in a 0.125 F capacitor is 50 J, solve for the charge
accumulated.

Solution:
Q Q2
Substitute V = C to the formula W = ½ CV2 , so that W = ½ C
295

To find Q, Q = √2CW

= √2(0.125 x 10−6 )(50)

= 3.54 mF

11.7 Voltage Rating

Every capacitor has a limit on the amount of voltage that it can withstand across its
plates. The voltage rating specifies the maximum dc voltage that can be applied
without risk of damage to the device. If this maximum voltage, commonly called the
breakdown voltage or working voltage, is exceeded, permanent damage to the
capacitor can result.
Both the capacitance and the voltage rating must be taken into consideration
before a capacitor is used in a circuit application. The choice of capacitance value is
based on particular circuit requirements (and or factors that are studied later). The
voltage rating should always be well above the maximum voltage expected in a
particular application.

11.8 Dielectric Strength

The breakdown voltage of a capacitor is determined by a dielectric strength of the


dielectric material used. The dielectric strength is expressed in volts/mil (1 mil =
0.001 in.) Table 11.1 below shows typical values for several materials. Exact values
vary depending on the specific composition of the material.

Table 11.1
Some Common Dielectric Materials and their Dielectric Strengths.

Material Dielectric Strength


(volts/mil)
Air 80
Oil 375
Ceramic 1000
Paper 1200
Teflon 1500
Mica 1500
Glass 2000

11.9 Temperature Coefficient


296

The temperature coefficient indicates the amount and direction of a change in


capacitance value with temperature. A positive temperature coefficient means that
th capacitance increases with an increase in temperature or decreases in
temperature. A negative coefficient means that the capacitance decreases with an
increases in temperature or increases with a decreases in temperature.
Temperature coefficients typically are specified in parts per million per degree
Celsius (ppm/C). For example, a negative temperature coefficient of 150 ppm/C
for a 1-F capacitor means that for every degree rise in temperature, the capacitance
decreases by 150 pF (there are one million picofarads in one microfarad).

11.10 Leakage

No insulating material is perfect. The dielectric of any of any capacitor will conduct
some very small amount of current. Thus, the charge on a capacitor will eventually
leak off. Some types of capacitors have higher leakages than others. An equivalent
circuit for a nonideal capacitor is shown in Figure 11.3. The parallel resistor
represents the extremely high resistance of the dielectric material through which
leakage current flows.

C R leak

Figure 11.3. Equivalent Circuit for a Nonideal Capacitor

11.11 Physical Characteristics of a Capacitor

The following parameters are important in establishing the capacitance and the
voltage rating of a capacitor: plate area, plate separation, and dielectric constant.

 Plate Area

Capacitance is directly proportional to the physical size of the plates as determined


by the plate area.
297

 Plate Separation

Capacitance is inversely proportional to the distance between the plates.


Dielectric Constant

As you know, the insulating material between the plates of a capacitor is called the
dielectric. Every dielectric material has the ability to concentrate the lines of force of
the electric field existing between the oppositely charged plates of a capacitor and
thus increase the capacity for energy storage. The measure of a material’s ability to
establish an electric field is called the dielectric constant or relative permittivity ,
symbolized by r (the Greek letter epsilon).
Capacitance is directly proportional to the dielectric constant. The dielectric
constant (relative permittivity) is dimensionless, because it is a relative measure and
is a ratio of the absolutely permittivity, , of a material to the absolute permittivity, o,
of a vacuum, as expressed by the formula:

r = o

The value of o is 8.85 x 10-12 (farads per meter).

Table 11.2
Some Common Dielectric Materials and their Dielectric Constants

Material Typical r values


Air (vacuum) 1.0
Teflon 2.0
Paper (paraffined) 2.5
Oil 4.0
Mica 5.0
Glass 7.5
Ceramic 1200

11.12 Formula for Capacitance in Terms of physical Parameters

An exact formula for calculating the capacitance in terms of the three quantities
mentioned is as follows:

A r (8.85 x 10−12 F/m)


C = d

where A is in square meters (m2), d is in meters (m), C is in farads (F) and r is the
relative permittivity.
298

Example 11.5 Determine the capacitance of a parallel plate capacitor having a plate
area of 0.01 m2 and a plate separation of 0.02 m. The dielectric is mica, which has a
dielectric constant of 5.0.

Solution:
A r (8.85 x 10−12 F/m)
C = d

(0.01 m2 )(5.0)(8.85 x 10−12 F/m)


= 0.02 m

= 22.13 pF

11.13 Types of Capacitor

 Mica Capacitors

 Ceramic Capacitors

 Paper/Plastic capacitors

 Electrolytic Capacitors

 Variable Capacitors

 Air Capacitors

 Trimmers and Padders

 Varactors

11.14 Series Capacitors

C1 C2 C3

Vs
299

While charging, I = Q/t is the same at all points so that all capacitors store the same
amount of charge (QT = Q1 = Q2 = Q3)

By Kirchhoff’s voltage law,

Vs = V1 + V2 + V3

Using the fact that V = Q/C, we can substitute into the formula for Kirchhoff’s law
and get he following relationship (where Q = QT = Q1 = Q2 = Q3):

Q Q Q Q
= + +
CT C1 C2 C3

Canceling out Q we have,


1 1 1 1
= + +
CT C1 C2 C3

Taking the reciprocal of both sides gives the formula for the total capacitance:

1
CT = 1 1 1
+ +
C1 C2 C3

Voltage Division in Series Capacitors

CT
Vx = x Vs
Cx

where Vx is the voltage across Cx which is any capacitor, such as C1, C2, and so on.

Example 11.6 Three capacitors of C1 = 50 µF, C2 = 100 µF and C3 = 200 µF are


connected in series. What is the total capacitance?

Solution:
1
CT = 1 1 1 = 28.57 µF
+ +
50 µC 100 µC 200 µC

Example 11.7 What is the voltage each capacitor in example 10.6 if they are
connected across a 120-V DC source?
300

Solution:
CT 28.57 µF
V1 = x Vs = x 120 V = 68.57 V
C1 50 µF

CT 28.57 µF
V2 = x Vs = x 120 V = 34.28 V
C2 100 µF

CT 28.57 µF
V3 = x Vs = x 120 V = 17.14 V
C3 200 µF

11.15 Parallel Capacitors

Vs C1 C2 C3

The charged stored by the capacitors together equals the total charge that was
delivered from the source:

QT = Q1 + Q2 + Q3

Using the fact that Q = CV, we can substitute into the preceding formula and get the
following relationship:

CTVs = C1V1 + C2V2 + C3V3

Since Vs = V1 = V2 = V3, they can be canceled, leaving

CT = C1 + C2 + C3, etc.

Example 11.8 Three capacitors of C1 = 50 µF, C2 = 100 µF and C3 = 200 µF are


connected in parallel across a 120-V DC source. (a) What is the total capacitance?
(b) What is the charge in each capacitor?
301

Solution:

(a) CT = 50 µF + 100 µF + 200 µF = 350 µF

(b) Q1 = C1V1 = (50 µC)(120V) = 6,000 µC

Q2 = C2V2 = (100 µC)(120V) = 12,000 µC

Q3= C3V3 = (200 µC)(120V) = 24,000 µC

11.16 Capacitor in DC Circuit

In this section, the response during charging and discharging of a simple capacitive
circuit with a dc source is examined. Figure 11.4 shows a capacitor connected in
series with a resistor and a switch to a dc voltage source. Initially, the switch is open
and the capacitor is uncharged with zero volts across its plates. At the instant the
switch is closed, the current jumps to its maximum value and the capacitor begins to
charge. The current is maximum initially because the capacitor has zero volts across
it and therefore, appears as a short; thus, the current is limited only by the resistance.
S time passed and the capacitor charges, the current decreases and the voltage VC
across the capacitor increases. The resistor voltage is proportional to the current
during this charging period.
302

Figure 11.4 Charging and discharging of a capacitor.

After a certain period of time, the capacitor reaches full charge. At this point,
the current is zero and the capacitor voltage is equal to the dc source voltage, as
shown in figure. If the switch were opened now, the capacitor would retain its full
charge (neglecting any leakage).
In figure, the voltage source has been removed. When the switch is closed, the
capacitor begins to discharge. Initially, the current jumps to a maximum but in a
direction opposite to its direction during charging. As time passes, the current and
capacitor voltage decrease. The resistor voltage is always proportional to the current.
When the capacitor has fully discharged, the current and the capacitor voltage are
zero.
303

Remember the following about capacitors in dc circuits:

1. Voltage across a capacitor cannot change instantaneously.


2. Current in a capacitive circuit can change instantaneously.
3. A fully charged capacitor appears as an open to nonchanging current.
4. An uncharged capacitor appears as a short to an instantaneous change in
current.

11.17 The RC Time Constant

As you have seen, when a capacitor charges or discharges through a resistance, a


certain time is required for the capacitor to charge fully or discharge fully. The
voltage across a capacitor cannot change instantaneously, because a finite time is
required to move charge from one point to another. The rate at which the capacitor
charges or discharges is determined by the time constant of the circuit.

The time constant of a series RC circuit is a time interval that equals the
product of the resistance and the capacitance.

The time constant is symbolized by  , and the formula is as follows

 = RC

Recall that I = Q/t. The current is the amount of charge moved in a given time.
When the resistance is increased, the charging current is reduced, thus increasing
the charging time of the capacitor. When the capacitance is increased, the amount of
charge increases; thus, for the same current, more time is required to charge the
capacitor.

Example 11.9 A series RC circuit has a resistance of 1 M and a capacitance of 5 F.


What is the time constant?

Solution:

 = RC = (1 M)(5 F) = 5s
304

11.18 The Charging Curve

120%

100%
100%

80%
Curremt

60%

37%
40%

20% 14%
5%
2% 1%
0%
0 1 2 3 4 5 6
Time Constant (τ)

Current in a Capacitor during charging

120

100

80
Voltage

60

40

20

0
0 1 2 3 4 5 6
Time Constant (τ)

Voltage in a Capacitor during charging


305

Voltage and Current in a Capacitor during charging and discharging

A. Electrical Transient Analysis

R ER
12 V
E 1.0kΩ
1µF
C EC
306

Let ER be the voltage across the resistor


EC be the voltage across the capacitor

dQ
ER = RI = R dt
Q
EC =
C

By KVL

ER + EC = E

dQ Q
R dt + = E
C

Divide the C

dQ Q 𝐸
+ =𝑅 With the initial values Q= 0 when t = 0,
dt RC

The particular solution is


t
Q = EC (1 − e− RC )

To find ER and EC at a given time


t

d[EC (1− e RC )]
ER = R dt
Q
EL =
C

QUESTIONS:
1. Describe the basic construction of a capacitor.

2. What is capacitance? What are the factors that affect capacitance?

3. Explain how a capacitor stores energy.

4. Explain Coulomb’s law and discuss how it relates to an electric field and the
storage of energy.

5. How does capacitor charges and discharges?

6. How do you determine total series capacitance?


307

7. How do you determine total parallel capacitance?

8. What is time constant as related to a capacitive circuit.

9. Relate the charging and discharging of a capacitor to the time constant.

10. Explain why a capacitor blocks dc.

11. Explain why a capacitor produces no energy loss.

12. Explain the significance of reactive power in a capacitive circuit.

13. What are some common capacitor applications?

14. How do you check out a capacitor with an ohmmeter?

Assessment No. 25

CAPACITOR AND CAPACITANCE

Name: Ybañez, Eric L. Score: _________ Rating: ______

1. Derive the formula for the energy stored in a capacitor.

2. Derive the formula for the inductance.


308

3. Derive the formula for the total capacitance of series capacitors.

4. Derive the formula for the total capacitance of parallel capacitors

5. Convert the following to microfarad:


a. 0.0000003456 F b. 0.00000456 F
309

Problem Set No. 21

CAPACITOR AND CAPACITANCE

1. Two capacitors connected in parallel across a 250-V mains have charges of


3,000 µC and 5,000 C, respectively. Find the total capacitance of the
combination.
A. 32 µF
B. 45 µF
C. 56 µF
D. 76 µF

2. A 0.4-F capacitor has a charge of 20 C. How much is the voltage across it?
A. 20 V
B. 30 V
C. 45 V
D. 50 V

3. The equivalent capacitance of two capacitors in series is 2.4 F. If one of the
capacitors has a capacitance of 4 F, what is the capacitance of the other?
A. 2 µF
B. 4 µF
C. 6 µF
D. 8µF

4. Three capacitors having capacitance of 4 F , 6 F and 8 F respectively are


connected in series. Find the equivalent capacitance of the combination.
A. 1.08 µF
B. 2.84 µF
C. 1.84 µF
D. 4.84 µF

5. The energy stored in a 0.125 F capacitor is 50 J, solve for the charge


accumulated.
A. 3.54 mC
B. 5.34 mC
C. 6.23 mC
D. 8,45 mC
310

6. A certain capacitor is charged at 48 volts after its stored energy is 5.76 x 10-2
joules. What is the capacitance of the capacitor?
A. 25 µF
B. 50 µF
C. 75 µF
D. 89 µF

7. Calculate the capacitance between two plates each of which is 100 cm2 and 2 mm
apart in air.
A. 44.27 pF
B. 48.90 pF
C. 56.84 nF
D. 76.43 nF

8. A capacitor whose plates is 20 cm x 3.0 cm and is separated by a 1.0-mm air gap


is connected across a 12-V battery. Determine the charge accumulated on each
plate after a long time.
A. 438.9 pC
B. 563.2 pC
C. 637.5 pC
D. 895.6 pC

9. Three capacitors A, B, and C are charged as follows: A: 10F, 100 volts; B: 15


F, 150 volts; C: 25 F, 200 volts. They are then connected in parallel with
terminals of like polarity together. What is the voltage across the combination?
A. 165 V
B. 175 V
C. 185 V
D. 195 V

10. A given capacitor has a capacitance of 100 F. Calculate its elastance.
A. 1,000 D
B. 10,000 D
C. 100,000 D
D. 1,000,000 D
11. Three capacitors of 5 F , 10 F and 15 F respectively are connected in series
across a 100-V supply. Solve for the voltage across the 15-F capacitor.
A. 18.2 V
B. 19.6 V
C. 20.7 V
D. 25.4 V
311

Objective Test No. 13

CAPACITOR AND CAPACITANCE

1. Which of the following statement(s) accurately describes a capacitor?

A. The plates are conductive.


B. The dielectric is an insulator between the plates.
C. Constant dc flows through a fully charged capacitor.
D. A practical capacitor stores charge indefinitely when disconnected from the
source.

2. The capacity of a condenser is proportional to ___________.


A. area of its plates
B. volume of its plates
C. the specific resistance of the plate material
D. the temperature coefficient of the plate material

3. The capacity of the capacitor is inversely proportional to ________.


A. the temperature of the dielectric
B. the material of the dielectric
C. the thickness of the dielectric
D. the permeability of the material and inductance

4. The capacitors in series have the same _______.


A. voltage
B. capacity
C. charge
D. energy loss

5. The capacitors are named according to the _______ used.


A. material of the plate
B. dielectric used
C. enclosures used
D. voltage

6. When one of the following statements is true?


A. There is current through the dielectric of a charging capacitor.
B. When a capacitor is connected to a DC voltage source, it will charge to the
value of the source.
312

C. An ideal capacitor can be discharged by disconnecting it from the voltage


source.

7. A capacitance of 0.01 F is larger than


A. 0.00001 F
B. 100,000 pF
C. 1000 pF
D. all of the above

8. When the voltage across a capacitor is increased, the stored charge


A. increases
B. decreases
C. remains constant
D. fluctuates

9. When the voltage across a capacitor is doubled, the stored charge


A. stays the same
B. is halved
C. increases by four
D. doubles

10. The voltage rating of a capacitor is increased by


A. decreasing plate area
B. increasing plate separation
C. increasing the plate area
D. a and b

11. The capacitance value is increased by


A. decreasing plate area
B. increasing plate separation
C. decreasing plate separation
D. increasing plate area

12. An uncharged capacitor and a resistor are connected in series with a switch and
a 12-V battery. At the instant the switch is closed, the voltage across the
capacitor is
A. 12 V
B. 6 V
C. 24 V
D. 0 V

13. In Question 12, the voltage across the capacitor when it is fully charged is
313

A. 12 V
B. 6V
C. 24 V
D. -6 V

14. An ohmmeter is connected across a discharged capacitor and the needle


stabilizes at approximately 50 k. The capacitor is
A. good
B. charged
C. too large
D. leaky

15. A good capacitor has a __________ resistance.


A. negligible
B. very high
C. negative
D. none of these

16. A capacitor opposes any change in __________.


A. current
B. voltage
C. Resistance
D. flux

17. The capacitance of a capacitor is directly proportional to


A. area of its plate
B. thickness of dielectric

18. Capacitors are used to


A. filter AC currents and pass DC currents
B. filter AC and DC currents
C. filter DC currents and pass AC currents
D. pass AC and DC currents

19. A capacitor consists of two _________


A. insulators separated by a conductor.
B. conductors separated by an insulator
C. conductors
D. insulators
314

20. Capacitors designed to be used in places where a high dielectric breakdown


voltage is important .
A. paper capacitors
B. ceramic capacitors
C. electrolytic capacitors
D. mica capacitors

21. Reciprocal of capacitance.


A. Inductance
B. Elastance
C. Reluctance
D. Daraf

22. The capacitance of a capacitor is NOT affected by _____.


A. type of dielectric material
B. distance between plates
C. area of the plates
D. type of material used in the plates

23. The capacitor stores the electricity in the shape of _____.


A. dynamic charge
B. static charge
C. current electricity
D. molecules

24. Capacitors are used in electric circuits to ____________.


A. store energy
B. introduce a voltage drop
C. produce a low opposition path to high frequencies
D. all of these
315

Unit 12

INDUCTOR AND INDUCTANCE

M.
LEARNING
N. OUTCOMES

After completing this unit,


you are expected to:

1. describe the basic construction of an inductor.


2. define inductance.
3. explain how an inductor stores energy.
4. relate various physical parameters to inductance value.
5. explain why practical inductors have both resistance and
capacitance.
6. state Lenz’s law and Faraday’s law.
7. identify various types of inductors.
8. determine total series inductance.
9. determine total parallel inductance.
10. define time constant as related to an inductive circuit.
11. explain how an inductor causes a phase shift between
current and voltage.
12. define inductive reactance and determine its value in a
circuit.
13. describe some common inductor applications.
14. check out an inductor with an ohmmeter.
316

Important Terms

inductor winding resistance


electromagnetic field winding capacitance
self-inductance Faraday’s law
henry Lenz’s law
induced voltage permeability
energy storage time constant
core

12.1 The Inductor

When a length of wire is formed into a coil, as shown in Figure 12.1, it becomes a basic
inductor. Current through coil produces an electromagnetic field. The magnetic lines
of force form a strong magnetic field within and around a coil.. The net direction of
the total magnetic field creates a north and a south pole, as indicated.

Figure 12.1. A coil of wire forms an inductor. When current flows through it, a three-
dimensional electromagnetic field is created, surrounding the coil in all directions.

12.2 Self-Inductance

When there is current through an inductor, an electromagnetic field is established.


When the current changes, the electromagnetic field also changes. An increase in
317

current expands the field, and a decrease in current reduces it. Therefore, a changing
current produces a changing electromagnetic field around the inductor (coil). In
turn, the changing electromagnetic field produces a voltage across the coil in a
direction to oppose the change in current. This property is called self-inductance, but
it is usually referred to as simply inductance. Inductance is symbolized by L.

Inductance is a measure of a coil’s ability to establish an induced voltage as a


result of a change in its current and that induced voltage is in direction to oppose that
change in current.

The Unit of Inductance

The henry, symbolized by H, is the basic unit of inductance. By definition, the


inductance is one henry when current through the coil, changing at the rate of one
ampere per second, induces one volt across the coil. In many practical application,
millihenries (mH) and microhenries (H) are the most common units. A common
schematic symbol for the inductor is shown in Figure 12.2.
L

Figure 12.2. Symnol of an inductor

12.3 The Induced Voltage in an Inductor

A changing current in an inductor causes a changing magnetic field though it. Since
according to Faraday’s law a changing magnetic field results to the induction of
voltage across the inductor.

The formula for the induced emf (or voltage) across the coil or inductor is,

di
EL = L dt

where EL = the induced emf across a coil or inductor in volts (V)


L = the inductance in henry (H)
di
= rate of change of current in amp/sec.
dt

Example 12.1 A group of electromagnets that create a flux in a dc generator – the


field circuit – has an inductance of 15 henrys. If the 2.6 amp excitation is
318

interrupted in 0.04 sec by the opening of the field switch, what average voltage is
induced in the winding?
2.6
Solution: EL = 15 x 0.04 = 975 volts

12.4 Energy Storage

An inductor stores energy in the magnetic field created by the current. The energy
stored is expressed as follows
1
W = LI2
2

As you can see, the energy stored is proportional to the inductance and the square of
the current. When I is in amperes and L is in henries, the energy is in joules.

12.5 Physical Characteristics

The following characteristics are important in establishing the inductance of a coil,


the core material, the number of turns of wire, the length, and the cross-sectional
area.

 Core Material

As discussed earlier, an inductor is basically a coil of wire. The material around which
the coil is formed is called the core. Coils are wound on either nonmagnetic or
magnetic materials. Examples of nonmagnetic materials are air, wood, copper,
plastic, and glass. The permeabilities of these material are the same as for a vacuum.
Examples of magnetic materials are iron, nickel, steel, cobalt, or alloys. These
materials have permeabilities that are hundreds or thousands of times greater than
that of a vacuum and are classified as ferromagnetic. A ferromagnetic core provides
a better path for the magnetic lines of force and thus permits a stronger magnetic
field.
The permeability () of the core material determines how easily a magnetic
field can be established. The inductance is directly proportional to the permeability
of the core material.

 Parameters

As indicated in the Figure 12.3, the number of turns of wire, the length, and the cross
sectional area of the core are factors in setting the value of inductance. The
inductance is inversely proportional to the length of the core and directly
proportional to the cross-sectional area. Also, the inductance is directly related to the
number of turns squared.
319

Figure 12.3. Factors that determine the inductance of a coil.

This relationship is as follows:

N2  A
L= l

where L is the inductance in henries, N is the number of turns,  is the permeability,


A is the cross-sectional area in meters squared, and l is the core length in meters.

Example 12.2 Determine the inductance of the coil below. The permeability of the
core is 0.25 x 10-3.

0.01 m
0.1 m2
N=4

Solution:

N2  A (4)2 (0.25 x 10−3 )(0.1)


L= = = 40 mH
l 0.01

12.6 Winding Resistance

When a coil is made of a certain material, for example, insulated copper wire, that
wire has a certain resistance per unit of length. When many turns of wire are used to
construct a coil, the total resistance may be significant. This inherent resistance is
called the dc resistance of the winding resistance (Rw). Although this resistance is
distributed along the length of the wire, it effectively appears in series with the
320

inductance of the coil, as shown in Figure 12.4. In many applications, the winding
resistance can be ignored and the coil considered as an ideal inductor. In other cases,
the resistance must be considered.

Rw L

(a) The wire has resistance (b) Equivalent circuit

Figure 12.4. Winding resistance of a coil.

12.7 Winding Capacitance

When two conductors are placed side by side, there is always some capacitance
between them. Thus, when many turns of wire are placed close together in a coil, a
certain amount of stray capacitance is a natural side effect. In many applications, this
stray capacitance is very small and has no significant effect. In other cases,
particularly at high frequencies, ti may become quite important.
The equivalent circuit for an inductor with both its winding resistance (R w)
and its winding capacitance (Cw) is shown in Figure 12.5. The capacitance effectively
acts in parallel.

Cw

Rw L

(a) Stray capacitance between each (b) Equivalent circuit


loop appears as a total parallel
Capacitance.

Figure 12.5. Winding capacitance of a coil.

12.8 Faraday’s Law

Faraday found that by moving a magnet through a coil of wire, a voltage was
introduced across the coil, and that when a complete path was provided, the
induced voltage an induced current.
The amount of induced voltage is directly proportional to the rate of change of
the magnetic field with respect to the coil.
321

This principle is illustrated in the Figure 12.6, where a bar magnet is moved through
a coil of wire. An induced voltage is indicated by the voltmeter connected across the
coil. The faster the magnet is moved, the greater is the induced voltage.

When a wire is formed into a certain number of loops or turns and is exposed
to a changing magnetic field, a voltage is induced across the coil. The induced
voltage is proportional to the number of turns of wire in the coil, N, and to the rate
at which the magnetic field changes.

Figure 12.6. Induced voltage is created by a changing magnetic field.

12.9 Lenz’s Law

Lenz’s law adds to Faraday’s law by defining the direction of induced voltage as
follows:

When the current through a coil changes and an induced voltage is created as a
result of the changing magnetic field, the direction of the induced voltage is such that it
always opposes the change in current.

In Figure 12.7 (a), the current is constant and is limited by R1. There is no
induced voltage because the magnetic field is unchanging. In part (b), the switch
suddenly is closed, placing R2 in parallel with R1 and thus reducing the resistance.
Naturally, the current tries to increases and the magnetic field begins to expand, but
the induced voltage opposes this attempted increases in current for an instant.
322

In part (c), the induced voltage gradually decreases, allowing the current to
increase. In part (d), the current has reached a constant value as determined by the
parallel resistors, and the induced voltage is zero. In part (e), the switch has been
suddenly opened, and, for an instant, the induced voltage prevents any decreases in
current. In part (f), the induced voltage gradually decreases, allowing the current to
decreases back to a value determined by R1. Notice that the induced voltage has a
polarity that opposes any current change. The polarity of the induced voltage is
opposite that of the battery voltage for an increases in current and aids the battery

voltage for a decreases in current.

(a) Switch open: Constant (b) At instant of switch closure:


current and constant Expanding magnetic field
magnetic field; no induced induces voltage, which
voltage. prevents increase in total
current.

(c) Right after switch closure: The rate (d) Switch remains closed: Current
of expansion of the magnetic field and magnetic field reach
decreases, allowing the current to constant value.
increase as induced voltage
decreases.
323

(e) At instant of switch opening: (f) After switch opening: Rate of


Magnetic field begins to collapse, collapse of magnetic field decrease,
creating an induced voltage, which allowing current to decrease back to
prevents decrease in current. original value.

Figure 12.7. Demonstration of Lenz’s law: When the current tries to change
suddenly, the electromagnetic field changes and induces a voltage in a direction that
opposes that change in current.

12.10 Classifications of Inductor

Inductors are made in a variety of shapes and sizes. Basically, they fall into two
general categories: fixed and variable.

(a) Fixed (b) Variable

Both fixed and variable inductors can be classified according to the type of core
material. Three common types are the air core, the iron core, and the ferrite core.
Each has a unique symbol, as shown.

(a) Air core (b) Iron core (c) Ferrite core


324

Adjustable (variable) inductors usually have a screw-type adjustment that


moves a sliding core in and out, thus changing the changing the inductance.

12.11 Types of Inductor

(a) fixed molded inductors


(b) variable coils
(c) toroid inductor

13.12 Series Inductors

When inductors are connected in series, the total inductance, LT, is the sum of the
individual inductances. The formula for LT is expressed in the following equation for
the general case of n inductors in series:

LT = L 1 + L 2 + L 3 + . . . + L n

Notice that the formula for inductance in series is similar to the formula for
resistance in series.
L1 L2 L3 Ln

Figure 12.8 Inductors in Series

12.13 Parallel Inductors

When inductors are connected in parallel, the total inductance is less than the
smallest inductance. The formula for total inductance in parallel is similar to that
for total parallel resistance.
1 1 1 1 1
= + + + …+
LT L1 L2 L3 Ln

Figure 12.9 Inductors in parallel


325

The general formula states that the reciprocal of the total inductance is equal to the
sum of the reciprocals of the individual inductances. LT can be found by taking the
reciprocals of both sides of the equation.
1
LT = 1 1 1 1
+ + + ….+
L1 L2 L3 Ln
or
LT = (L1-1 + L2-1 + L3-1 + . . . + Ln-1)-1

12.14 Inductors in DC Circuits

When there is constant direct current in an inductor, there is no induced voltage.


There is, however, a voltage drop due to the winding resistance of the coil. The
inductance itself appears as a short to dc. Energy is stored in the magnetic field
according to the formula W = ½ LI2. The only energy loss occurs in the winding
resistance (P = I2Rw).

This condition is illustrated in Figure 12.10.

Figure 12.10. Energy storage and loss in an inductor. The only dc voltage drop across
the coil is due to the winding resistance.

Remember the following about inductors in DC circuits:

1. Current through an inductor cannot change instantaneously.


2. Voltage in an inductive circuit can change instantaneously.
3. An inductor appears as a short to non-changing magnetic field as in the case
of DC.

13.15 Time Constant


326

Because the inductor’s basic action is to oppose a change in its current, it follows that
current cannot change instantaneously in an inductor. A certain time is required for
the current to make a change from one value to another. The rate at which the current
changes is determined by the time constant. The time constant for a series RL circuit
is
L
 = R

where  is in seconds when L is in henries and R is in ohms.

Example 12.3 A series RL circuit has a resistance of 1 k and an inductance of 1 mH.


What is the time constant?

Solution:
L 1 mH
 = = = 1 s
R 1 k

12.16 Energizing Current in an Inductor

In a series RL circuit, the current will increase to 63% of its value in one time constant
interval after the switch is closed. The buildup of current is analogous to the buildup
of capacitor voltage during the charging in an RC circuit; they both follow an
exponential curve and reach the approximate percentages of final value as indicated
in the Figure 12.11.

1.2

98% 99%
1 95%
86%

0.8
63%
Current

0.6

0.4

0.2

0
0
0 1 2 3 4 5 6
Time Constant (τ)
327

Figure 12.11. Illustration of the exponential buildup of current in an inductor. The current
increases another 63% during each time constant interval. A winding resistance of 10 is
assumed. A voltage (VL) is induced in the coil that tends to oppose the increase in current.

The change in current over five time constant intervals is illustrated in Figure 12.11.
When the current reaches its final value at approximately 5, it ceases to change. At
this time, the inductor acts as a short (except for winding resistance) to the constant
current. The final value of the current is Vs/Rw = 10 V/10  = 1 A.

120%

100%
100%

80%
Voltage

60%

37%
40%

20% 14%
5%
2% 1%
0%
0 1 2 3 4 5 6
Time Constant (τ)
328

Example 12.4 Calculate the time constant for the circuit shown below. Then
determine the current and the time at each time constant interval, measured from
the instant the switch is closed.
R
100 ohms

L
20 V 50 mH

Solution:

V 20 V
Ifinal = = = 0.2 A
R 100

L 50 mH
 = = = 0.5 ms
R 100

At 1 = 0.5 ms: i = 0.63(0.2 A) = 0. 126 A


At 2 = 1.0 ms: i = 0.86(0.2 A) = 0. 172 A
329

At 3 = 1.5 ms: i = 0.95(0.2 A) = 0. 190 A


At 4 = 2.0 ms: i = 0.98(0.2 A) = 0. 196 A
At 5 = 2.5 ms: i = 0.99(0.2 A) = 0. 198 A  0.2 A

1.16 Electrical Transient Analysis

ER
R
E EL
Key2J1
12
3 1Space
1.0kΩ
=V
4
1.0mH

Let ER be the voltage across the winding resistance.


EL be the voltage induced due to the inductance of the inductor.

ER = RI
dI
EL = L
dt

By KVL

ER + EL = E

dI
RI + L dt = E

Rearranging the equation.

dI
L dt + RI = E

Divide the L

dI 𝑅 𝐸
+ 𝐿I = 𝐿 With the initial values I = 0 when t = 0,
dt

The particular solution is


R
E
I= (1 − e− L t )
R
330

To find ER and EL in a given time

ER = RI
R
E − t
d[ (1− e L )] R
dI R
EL = L dt = L = Ee− L t
dt

12.17 Inductor Applications

Power Supply Filter

rf Choke

Tuned Circuits

12.18 Testing Inductors

The most common failure in an inductor is an open coil. To check for an open, remove
the coil from the circuit. If there is an open, an ohmmeter check will indicate infinite
resistance, as shown in Figure 12.12. If the coil is good, the ohmmeter will show the
winding resistance. The value of winding resistance depends on the wire size and
length of the coil. It can be anywhere from one ohm to several hundred ohms.

Occasionally, when an inductor is overheated with excessive current, the wire


insulation will melt, and some coil will short together. This produces a reduction in
the inductance by reducing the effective number of runs and a corresponding
reduction in winding resistance.

Figure 12.12. Checking a coil by measuring the resistance.


331

Assessment No. 26

INDUCTOR AND INDUCTANCE

Name: Ybañez, Eric L. Score: _________ Rating: ______

1. Derive the formula for the energy stored in an inductor.

2. Derive the formula for the inductance.

3. Derive the formula for the total inductance of series inductors.


332

4. Derive the formula for the total inductance of parallel inductors.

5. Convert the following to millihenries:


a. 1 H b. 250 H c. 10 H d. 0.0005 H
333

Problem Set No. 22

INDUCTOR AND INDUCTANCE

1. How many turns are required to produce 30 mH with a coil wound on a


cylindrical coil having a cross-sectional area of 10 x 10-5 m2 and a length of 0.05
m? The core has a permeability of 1.2 x 10-6?
A. 3536 turns
B. 3679 turns
C. 3987 turns
D. 4502 turns

2. A 12-V battery is connected across a coil with a winding resistance of 12 .


How much current is there in the coil?
A. 1 A
B. 2 A
C. 3 A
D. 4 A

3. How much energy is stored by a 100-mH inductor with a current of 1 A?


A. 0.02 J
B. 0.04 J
C. 0.05 J
D. 0.09 J

4. The current through a 100-mH coil is changing at a rate of 200 mA/s. How much
voltage is induced across the coil?
A. 0.02 V
B. 0.04 V
C. 0.01 V
D. 0.08 V

5. Suppose that you require a total inductance of 50 mH. You have available a 10-
mH coil and a 22-mH coil. How much additional inductance do you need?
A. 16 mH
B. 18 mH
C. 20 mH
D. 36 mH

6. Determine the total parallel inductance for the following coils in parallel: 75 H,
50 H, 25 H, and 15 H.
A. 7.14 µH
B. 8.90 µH
C. 9.28 µH
D. 10.67 µH
334

7. You have a 12-mH inductor, and it is your smallest value. You need an
inductance of 8 mH. What value can you use in parallel with the 12-mH to obtain
8 mH?
A. 12 mH
B. 18 mH
C. 20 mH
D. 24 mH
335

Objective Test No. 14

INDUCTOR AND INDUCTANCE

1. When the current though an inductor increases, the amount of energy stored in
the electromagnetic field
A. Decreases
B. remains constant
C. increases
D. doubles

2. When the current though an inductor doubles, the stored energy


A. Doubles
B. Quadruples
C. is halved
D. does not change

3. The winding resistance of a coil can be decreased by


A. reducing the number of turns
B. using a larger wire
C. changing the core material

4. The inductance of an iron-core coil increases if


B. the number of turns is increased
C. the iron core is removed
D. the length of the core is increased
E. larger wire is used

5. An inductor, a resistor, and a switch are connected in series to a 12-V battery. At


the instant the switch is closed, the inductor voltage is
A. 0 V
B. 12 V
C. 6 V
D. 4 V

6. An ohmmeter is connected across an inductor and the pointer indicates an


infinite value. The inductor is
A. Good
B. Open
C. shorted
D. resistive

7. The property that opposes any change in current


A. mutual inductance
336

B. friction
C. self-inductance
D. losses

8. An open coil has


A. infinite resistance and inductance
B. zero resistance and inductance
C. zero resistance and infinite inductance
D. infinite resistance and zero inductance

9. If the number of turns in an inductor is increased, its inductance will


A. Vary
B. Decrease
C. increase
D. remain the same

10. At DC steady state, an inductor acts like ___________.


A. an open circuit
B. a short circuit
C. a capacitor
D. an insulator

11. Unit of inductance


A. Farad
B. Ohm
C. Henry
D. siemen
337

Unit 13

ELECTRICAL TRANSIENTS

O.
LEARNING OUTCOMES

After completing this


unit, you are expected to:

1. to define electrical transients.


2. to analyze electrical transients in RL, RC, and RLC series DC
circuits.
338

13.1 Transient Period

When a circuit is to be switched on, it will not directly attain its steady state
response but instead passes through a transition period wherein the currents or
voltages adjust themselves to their steady state response. This transition period is
called the transient period.

Example 13.1 A graph of the current growth in a given RL DC circuit.

𝑖
Steady state response

Transient response

𝑡
Transient period

13.2 Series RL DC Circuit

E R
i= + ke− L t
R

E R
i= (1 − e− L t )
R

Note: “k” is an arbitrary constant whose value is found by substituting the initial
conditions of the circuit.

E
If the initial value of the current in the circuit at t = 0 is zero, then k = − R

where i = instantaneous current (ampere)


E = supply voltage (volt)
339

R = resistance of the resistor (ohm)


L = inductance of the coil (henry)

14.2.1 Instantaneous Voltage Drops

R R
di
ER = Ri = E (1 − e− L t ) EL = L dt = E (e− L t )

where: ER = voltage drop across the resistor


EL = voltage drop across the inductor

13.2.2 Instantaneous Powers (watt)

E2 R 2
− t
PR = ER i = (1 − e L )
R

E 2 − Rt 2R 2
PL = EL i = (e L − e− L t )
R

E2 R
PT = PR + PL = E(i) = (1 − e− L t )
R

where: PR = power dissipated by the resistor


PL = power dissipated by the inductor
PT = total power dissipated in the circuit

Example 13.1 A 1-H inductor whose resistance is 20 ohms is connected across a


200-V DC source. When t = 0, i = 0, find i when t = 0.02 second.

Solution:
R
E
Using the formula, i= (1 − e− L t )
R

where E = 200 V
R = 20 ohms
L = 1H
t = 0.02 sec
20
200
i= [1 − e− 1 (0.02) ] = 3.296 A
20
340

Example 13.2 A 500-ohm relay coil has an inductance of 10 mH. This relay
requires a current of 40 mA in order to trigger. How long after the switch is
closed will the relay trigger if a 24-V DC source is applied across its
terminals. Assume at t = 0, i = 0.

Solution:
R
E
From the formula , i= (1 − e− L t ) to find for t when i = 40 mA.
R

R 500(0.040)
ln (1 − E i) ln (1 − )
24
i= = = 35.83 μs
R 500
−L − 0.010

13.3 Series RC DC Circuit

t
i = ke−RC

Note: “k” is arbitrary constant whose value is found by substituting the initial
conditions of the circuit.

13.3.1 Instantaneous currents:

t
 Capacitor has no initial charge: i = ke−RC
341

Q
E± 0 t
 Capacitor has an initial charge of Q0: i = [ C
] e−RC
R

where: + (sign) = if capacitor is discharging


- (sign) = if capacitor is cgarging
i = instantaneous current
E = supply voltage
R = resistance
C = capacitance of the capacitor
Q0 = initial charge of the capacitor (coulomb)
t = time in seconds (s)

14.3.2 Instantaneous Voltage Drops (with Q0 = 0)


t t
1
ER = Ri = Ee−RC EC = C ∫ i(t)dt = E (1 − e−RC )

where: ER = voltage across the resistor


EC = voltage across the capacitor

13.3.3 Instantaneous Powers (watt)

E 2 − 2t
PR = ER i = (e RC )
R

E2 − 1 2t
PC = EC i = (e RC − e−RC )
R

E2 − t
PT = PR + PC = E(i) = (e RC )
R

where: PR = power dissipated by the resistor


PC = power dissipated by the capacitor
PT = total power dissipated in the circuit

13.3.4 Instantaneous Charge (coulomb) on the capacitor:


t
 Capacitor has no initial charge: Q = CE (1 − e−RC )

t
 Capacitor has an initial charge of Q0: Q = CE + (Q0 − CE)e−RC
342

Example 13.3 A resistance of 100 k is connected in series with a 100-F


capacitor. If the combination is suddenly connected across a 125-V DC source,
determine the current one second after the switch is closed.
Solution:

t
E
Using i= ( e−RC )
R

where: R = 100 k , C = 100 F , E = 125 V

Finding i when t = 1 sec


1
125
i= [e(100,000)(100 x 10−6) ] = 1.13 mA
100,000

Example 13.4 A series RC circuit consists of R = 2 M and an uncharged capacitor C


= 5 F. The circuit is connected across a 100-V DC source at t = 0. Determine the
voltage across the resistor 5 seconds later.
t
Solution: ER = Ee−RC

Substitute t = 5
1
(2 x 10−6 )(5 x 10−6 )
ER = 100e = 60.65 V

13.4 Series RLC DC Circuit


343

R 1
m2 + m+ =0 (Auxiliary equation)
L LC

 Overdamped case – when the roots of the auxiliary equation are real and
distinct.

R 2 4
(L) − >0 i = C1 em1 t + C2 em2 t
LC

where: m1 and m1 are the roots of the auxiliary equation

 Critically damped case – when the roots of the auxiliary equation are
repeated

R 2 4 R
(L) − =0 m = − 2L i = (C1 + C2 t)emt
LC

where: m is the repeated root

 Underdamped case – when the roots of the auxiliary equation contain a real
and an imaginary part.

2
√(R) − 4
R 2 4 R L LC
(L) − <0 α = − 2L β=
LC 2
i = eαt (C1 cosβt + C1 sinβt)

Note: C1 + C2 can be obtained using the initial conditions of the circuits.

Example 13.5 A series RLC circuit with R = 1 k , L = 1 H and C = 6.25 F is


suddenly connected across a 24-V DC source. At t = 0, i = 0 , and q = 0. Determine
the current after 0.01 sec.

Solution:
344

Solving for the roots

2 1000 1000 2 4
R R 4 − 1 ± √( 1 ) −
− L ± √( L ) − LC (1)(6. 25 x 10−6 )
m= =
2 2
−1000 ± 600
m=
2
−1000+ 600
(+) m1 = = −200
2
−1000− 600
(-) m2 = = −800
2

Note: Since the roots m1 and m2 are real and unequal, the circuit is an
“overdamped case”

i = C1 em1 t + C2 em2 t
i = C1 e−200t + C2 e−800t equation 1
di
= −200C1 e−200t − 800C2 e−800t equation 2
dt

At t = 0, i = 0 , substitute in equation 1

0 = C1 e−200(0) + C2 e−800(0)
C1 = − C2 equation 3

At t = 0, q = 0;
di E 24
= = = 24 A/s
dt L 1

Substitute di/dt = 24 , t = 0, and equation 3 in eq. 2

24 = −200(−C2 )e−200(0) − 800C2 e−800(0)


C2 = −0.04
C1 = 0.04

And in equation 1

i = C1 e−200t + C2 e−800t
i = 0.04e−200(0.01) − 0.04e−800(0.01)
i = 5.4 mA

Example 13.6 A series RLC circuit has R = 200  , L = 0.1 H, and a capacitor C = 10
F. If a 100-V DC source is connected across the end terminals of the series circuit
at t = 0, determine the current after 1 ms. Assume zero initial conditions.

Solution:
345

Solving for the roots


2
200 √(200) −
2 4
R √ R 4 − ±
− L ± ( L ) − LC 0.1 0.1 (0.1)(10 x 10−6 )
m= =
2 2
−2000 ± 0
m= = −1000
2
Note: Since there is only one real root, the circuit is a “critically damped case”

i = (C1 + C2 t)emt
i = (C1 + C2 t)e−1000t
C1 = 0 equation 1

Substitute t = 0 and i = 0 in eq 1

i = C1 e−1000(0) + C2 (0)e−1000(0)

Substitute C1 = 0 in eq. 1

i = C2 te−1000t eq. 2
di −1000t −1000t
= C2 [e (1) + te (−1000)]
dt

At t = 0, q = 0

di E 100
= = = 1000 A/s
dt L 0.1
1000 = C2 [e−1000(0) (1) + (0)e−1000(0) (−1000)]
C2 = 1000

Substitute t = 0.001 in eq. 2

i = C2 te−1000t = (1000)(0.01)e−1000(0.001) = 0.368 A

Example 13.7 A series RLC circuit has R = 40 ohms, L = 100 mH, and C = 50 F.
the circuit is connected across a 100-V DC source at t = 0 through a switch.
Determine the current 0.02 second after the switch is closed. Assume all initial
conditions to be zero.

Solution:
Solving for the roots

2 40 40 2 4
R R 4 − 0.1 ± √(0.1) −
− L ± √( L ) − LC (0.1)(50 x 10−6 )
m= =
2 2
346

Note: Since the roots m1 and m2 are real and unequal, the circuit is an
“overdamped case”

i = eαt (C1 cosβt + C2 sinβt)


R 40
 = − 2L = − 2(0.1) = −200

2 40 2 4
√( ) −
√(R) − 4 0.1 (0.1)(50 x 10−6 )
 = L LC
= = 400
2 2
−200t
i= e (C1 cos400t + C2 sin400t) eq. 1

Substitute t = 0, i = 0 in equation 1

0 = e−200(0) [C1 cos400(0) + C2 sin400(0)]


C1 = 0
i = e−200t (C2 sin400t)
di
= C2 (sin 400t e−200t (−200) + e−200t cos400t (400) equation 2
dt
di E 100
=L= = 1000 A/s
dt 0.1

Substitute di/dt = 1000 , t = 0, in eq. 2

1000 = C2 [sin 400(0) e−200(0) (−200) + e−200(0) 400cos400(0)


C1 = 2.5

Substitute t = 0.02

i = 2.5e−200t sin400t
i = 2.5e−200(0.02) sin400(0.02)
i = 6.4 mA
347

Assessment No. 26

ELECTRICAL TRANSIENTS

Name: Ybañez, Eric L. Score: _________ Rating: ______

1. In an RL circuit, E = 200 volts, R = 20 ohms, L = 1 H. If i = 0 when t = 0, find i


when t = 0.02 second. Find also the voltage across the resistor and inductor
when t = 0.02 sec.
348

2. A 6.0-H coil whose resistance is 12 ohms is connected in series with a 24-ohm


resistor and to a 144-V battery and a switch. The switch is closed at t = 0.
Determine the time constant of the circuit. Find also the current, voltage across
the resistor and inductor when t = 0.15 sec.
349

3. A 500-ohm relay coil has an inductance of 10 mH. This relay requires a current
of 40 mA in order to trigger. How long after the switch is closed will the relay
trigger if a 24-V DC source is applied across its terminals. Assume t = 0 , i = 0.
350

4. A coil having a resistance of 10  and inductance of 10 H is connected in series


with a 15- resistor. A 200-V DC source is connected to the circuit at t = 0.
Determine the voltage across the coil at t = 0.05 sec.
351

5. A resistance of 100 k is connected in series with a 100-F capacitor. If the


combination is suddenly connected across a 125-V DC source, determine the
current one second after the switch is closed.
352

6. A series RC circuit consist of R = 2 M and an uncharged capacitor C = 5 F.


The circuit is connected across a 100-V DC source at t = 0. Determine the
voltage across the resistor 5 seconds later.
353

SUPPLEMENTARY PROBLEMS

1. How many coulombs are represented by these amounts of electrons?


(a) 1.24 x 1018 (b) 1.628 x 1020
2. Determine the current flowing through an element if the charge flow is
(a) q(t) = (3t + 8) mC (b) q(t) = (3e-t – 5e-2t) nC
3. Determine the total charge transferred over the time interval of 0  t  10 s
1
when i(t) = 2t .
4. A lightning bolt with 10 kA strikes an object for 15 µs. How much charge is
deposited on the object?
5. A rechargeable flashlight is capable of delivering 90 mA for about 12 h. How
much charge can it release at that rate? If its terminal voltage is 1.5 V, how
much energy can the battery deliver?
6. If the current flowing through an element is given by

3t A, 0 ≤t <6𝑠
18 A, 6 ≤ t < 10 𝑠
i(t) = {
−12 A, 10 ≤ t < 15 𝑠
0, t ≥ 15 s

Find the charge stored in the element over 0 < 𝑡 < 20 𝑠


7. The charge entering the positive terminal of an element is
q = 5 sin 4t mC

while the voltage across the element (plus to minus) is


v = 3 cos 4t V
(a) Find the power delivered to the element at t = 0.3 s.
(b) Calculate the energy delivered to the element between 0 and 0.6 s.
8. The voltage v across a device and the current i through it are
v(t) = 10 cos 2t V, i(t) = 20(1 – e=0.5t) mA
Calculate:
(a) The total charge in the device at t = 1 s.
(b) The power consumed by the device at t = 1 s.
9. The current entering the positive terminal of a device is i(t) = 6e-2t mA and
di
the voltage across the device is v(t) = 10 V.
dt
(a) Find the charge delivered to the device between t = 0 and t = 2 s.
(b) Calculate the power absorbed.
(c) Determine the energy absorbed in 3 s.
354

APPLICATIONS
10. A 60-W incandescent bulb operates at 120 V. How many electrons and
coulombs flow through the bulb in one day?
11. A lightning bolt strikes an airplane with 40 kA for 1.7 ms. How many
coulombs of charge are deposited on the plane?
12. A 1.8-kW electric heater takes 15 min to boil a quantity of water. If this is
done once a day and power costs 10 cents/kWh, what is the cost of its
operation for 30 days?
13. A utility company charges 8.2 cents/kWh. If a consumer operates a 60-W
light bulb continuously for one day, how much is the consumer charged?
14. A 1.5-kW toaster takes roughly 3.5 minutes to heat four slices of bread. Find
the cost of operating the toaster once per day for 1 month (30 days). Assume
energy costs 8.2 cents/kWh.
15. A flashlight battery has a rating of 0.8 ampere-hours (Ah) and a lifetime of
10 hours.
(a) How much current can it deliver?
(b) How much power can it give if its terminal voltage is 6 V?
(c) How much energy is stored in the battery in Wh?
16. A constant current of 3 A for 4 hours is required to charge an automotive
battery. If the terminal voltage is 10 + t/2 V, where t is in hours,
(a) how much charge is transported as a result of the charging?
(b) how much energy is expended?
(c) how much does the charging cost? Assume electricity costs 9 cents/kWh.
17. A 60-W incandescent lamp is connected to a 120-V source and is left burning
continuously in an otherwise dark staircase. Determine:
(d) the current through the lamp.
(e) the cost of operating the light for one non-leap year if electricity costs 9.5
cents per kWh.
18. An electric stove with four burners and an oven is used in preparing a meal
as follows.
Burner 1: 20 minutes Burner 2: 40 minutes
Burner 3: 15 minutes Burner 4: 45 minutes
Oven: 30 minutes
If each burner is rated at 1.2 kW and the oven at 1.8 kW, and electricity costs
12 cents per kWh, calculate the cost of electricity used in preparing the meal.
19. Reliant Energy (the electric company in Houston, Texas) charges customers
as follows:
Monthly charge $6
First 250 kWh at $0.02/kWh
All additional kWh at $0.07/kWh
355

If a customer uses 2,436 kWh in one month, how much will Reliant Energy
charge?
20. Calculate the resistance of 100 m length of a wire having a uniform cross-
sectional area of 0.1 mm2 if the wire is made of manganin having a resistivity
of 50 × 10−8 Ω-m. If the wire is drawn out to three times its original length,
by how many times would you expect its resistance to be increased ? [500 Ω;
9 times]
21. A cube of a material of side 1 cm has a resistance of 0.001 Ω between its
opposite faces. If the same volume of the material has a length of 8 cm and a
uniform cross-section, what will be the resistance of this length ? [0.064 Ω]
22. A lead wire and an iron wire are connected in parallel. Their respective
specific resistances are in the ratio 49 : 24. The former carries 80 per cent
more current than the latter and the latter is 47 per cent longer than the
former. Determine the ratio of their cross-sectional area. [2.5 : 1]
23. A rectangular metal strip has the following dimensions :
x = 10 cm, y = 0.5 cm, z = 0.2 cm
Determine the ratio of resistances Rx, Ry, and Rz between the respective pairs of
opposite faces. [Rx : Ry : Rz : 10,000 : 25 : 4] (Elect. Engg. A.M.Ae. S.I.)
24. The resistance of a conductor 1 mm2 in cross-section and 20 m long is 0.346
Ω. Determine the specific resistance of the conducting material. [1.73 × 10−8
Ω-m] (Elect. Circuits-1, Bangalore Univ. 1991)
25. When a current of 2 A flows for 3 micro-seconds in a coper wire, estimate the
number of electrons crossing the cross-section of the wire. (Bombay
University, 2000)
Hint : With 2 A for 3 μ Sec, charge transferred = 6 μ-coulombs
Number of electrons crossed = 6 × 10−6/(1.6 × 10−19) = 3.75 × 10+ 13
26. It is found that the resistance of a coil of wire increases from 40 ohm at 15°C
to 50 ohm at 60°C. Calculate the resistance temperature coefficient at 0°C of
the conductor material. [1/165 per °C] (Elect. Technology, Indore Univ.)
27. A tungsten lamp filament has a temperature of 2,050°C and a resistance of
500 Ω when taking normal working current. Calculate the resistance of the
filament when it has a temperature of 25°C. Temperature coefficient at 0°C is
0.005/°C. [50 Ω] (Elect. Technology, Indore Univ.)
28. An armature has a resistance of 0.2 Ω at 150°C and the armature Cu loss is to
be limited to 600 watts with a temperature rise of 55°C. If α0 for Cu is
0.0043/°C, what is the maximum current that can be passed through the
armature ? [50.8 A]
29. A d.c. shunt motor after running for several hours on constant voltage mains
of 400 V takes a field current of 1.6 A. If the temperature rise is known to be
40°C, what value of extra circuit resistance is required to adjust the field
current to 1.6 A when starting from cold at 20°C ? Temperature coefficient =
0.0043/°C at 20°C. [36.69 Ω]
30. In a test to determine the resistance of a single-core cable, an applied voltage
of 2.5 V was necessary to produce a current of 2 A in it at 15°C.
(a) Calculate the cable resistance at 55°C if the temperature coefficient of
356

resistance of copper at 0°C is 1/235 per°C.


(b) If the cable under working conditions carries a current of 10 A at this
temperature, calculate the power dissipated in the cable. [(a) 1.45 Ω (b) 145 W]
31. An electric radiator is required to dissipate 1 kW when connected to a 230 V
supply. If the coils of the radiator are of wire 0.5 mm in diameter having
resistivity of 60 μ Ω-cm, calculate the necessary length of the wire. [1732 cm]
32. An electric heating element to dissipate 450 watts on 250 V mains is to be
made from nichrome ribbon of width 1 mm and thickness 0.05 mm. Calculate
the length of the ribbon required (the resistivity of nichrome is 110 × 10−8
Ω-m). [631 m]
33. When burning normally, the temperature of the filament in a 230 V, 150 W
gas-filled tungsten lamp is 2,750°C. Assuming a room temperature of 16°C,
calculate (a) the normal current taken by the lamp (b) the current taken at
the moment of switching on. Temperature coefficient of tungsten is 0.0047
Ω/Ω°C at 0°C. [(a) 0.652 A (b) 8.45 A] (Elect. Engg. Madras Univ.)
34. An aluminium wire 5 m long and 2 mm diameter is connected in parallel with
a wire 3 m long. The total current is 4 A and that in the aluminium wire is 2.5
A. Find the diameter of the copper wire. The respective resistivities of copper
and aluminium are 1.7 and 2.6 μΩ-m. [0.97 mm]
35. The field winding of d.c. motor connected across 230 V supply takes 1.15 A at
room temp. of 20°C. After working for some hours the current falls to 0.26 A,
the supply voltage remaining constant. Calculate the final working
temperature of field winding. Resistance temperature coefficient of copper at
20°C is 1/254.5. [70.4°C] (Elect. Engg. Pune Univ.)
36. It is required to construct a resistance of 100 Ω having a temperature
coefficient of 0.001 per°C. Wires of two materials of suitable cross-sectional
area are available. For material A, the resistance is 97 Ω per 100 metres and
for material B, the resistance is 40 Ω per 100 meters. The temperature
coefficient of resistance for material A is 0.003 per °C and for material B is
0.0005 per °C. Determine suitable lengths of wires of materials A and B. [A :
19.4 m, B : 200 m]
37. The resistance of the shunt winding of a d.c. machine is measured before and
after a run of several hours. The average values are 55 ohms and 63 ohms.
Calculate the rise in temperature of the winding. (Temperature coefficient of
resistance of copper is 0.00428 ohm per ohm per °C). [36°C] (London Univ.)
38. A piece of resistance wire, 15.6 m long and of cross-sectional area 12 mm2 at
a temperature of 0°C, passes a current of 7.9 A when connected to d.c. supply
at 240 V. Calculate (a) resistivity of the wire (b) the current which will flow
when the temperature rises to 55°C. The temperature coefficient of the
resistance wire is 0.00029 Ω/Ω/°C. [(a) 23.37 μΩ-m (b) 7.78 A] (London
Univ.)
39. A coil is connected to a constant d.c. supply of 100 V. At start, when it was at
the room temperature of 25°C, it drew a current of 13 A. After sometime, its
temperature was 70°C and the current reduced to 8.5 A. Find the current it
will draw when its temperature increases further to 80°C. Also, find the
357

temperature coefficient of resistance of the coil material at 25°C. [7.9 A;


0.01176°C−1] (F.Y. Engg. Univ.)
40. The resistance of the filed coils with copper conductors of a dynamo is 120 Ω
at 25°C. After working for 6 hours on full load, the resistance of the coil
increases to 140 Ω. Calculate the mean temperature rise of the field coil. Take
the temperature coefficient of the conductor material as 0.0042 at 0°C.
[43.8°C] (Elements of Elec. Engg. Banglore Univ.)
41. The total current drawn by a circuit consisting of three resistors connected in
parallel is 12 A. The voltage drop across the first resistor is 12 V, the value of
second resistor is 3 Ω and the power dissipation of the third resistor is 24 W.
What are the resistances of the first and third resistors ? [2Ω; 6Ω]
42. Three parallel connected resistors when connected across a d.c. voltage
source dissipate a total power of 72 W. The total current drawn is 6 A, the
current flowing through the first resistor is 3 A and the second and third
resistors have equal value. What are the resistances of the three resistors ? [4
Ω; 8 Ω; 8 Ω]
43. A bulb rated 110 V, 60 watts is connected with another bulb rated 110-V, 100
W across a 220 V mains. Calculate the resistance which should be joined in
parallel with the first bulb so that both the bulbs may take their rated power.
[302.5 Ω]
44. Two coils connected in parallel across 100 V supply mains take 10 A from the
line. The power dissipated in one coil is 600 W. What is the resistance of the
other coil ? [25 Ω]
45. An electric lamp whose resistance, when in use, is 2 Ω is connected to the
terminals of a dry cell whose e.m.f. is 1.5 V. If the current through the lamp is
0.5 A, calculate the internal resistance of the cell and the potential difference
between the terminals of the lamp. If two such cells are connected in parallel,
find the resistance which must be connected in series with the arrangement
to keep the current the same as before. [1 Ω ; 1 V ; 0.5 Ω] (Elect. Technology,
Indore Univ.)

References

“Electrical Power Systems Technology”. Stephen W. Fardo, Dale R. Patrick

“Electrical Engineering”, Chester Dawes

“Electric Circuits”, Charles Siskind

“Understanding AC Circuits”, Dale R. Patrick

”Electric Circuit Fundamentals”, Thomas L. Floyd

“1001 Solved Problems in Electrical Engineering”, Romeo Rojas


358

“Textbook-Reviewer in Electrical Engineering”, Marcialito M. Valenzona

“Electricity: Principles and Application”, Richard J. Fowier

“A Textbook in Electrical Technology”. Volume 1. Basic Electrical Engineering in SI


System of Units. B.L. Theraja, A.K. Theraja

You might also like